16-30

Réussis tes devoirs et examens dès maintenant avec Quizwiz!

Sarah is a nursing graduate of 5 years who is very confident in her clinical skills. She has taken some certificate courses in leadership and management and has considered beginning a graduate degree with this focus. She is excited about being able to use her knowledge and interest by being hired as a nurse manager. Before beginning her new position, Sarah spends time with her nurse executive to clarify the executive's expectations of her and of the unit that she has been hired to manage. After a year, Sarah successfully transitions into the nurse manager role and considers taking the examination to become a certified nurse executive. Amy advises her that this is not possible because Sarah will need:

24 months of experience.

11. As a new manager, you are shocked to learn that your unit is still using heparin in heparin locks. You are aware of evidence related to this practice and want to change this practice as quickly as possible on your unit. You are in which stage of Lewin's stages of change? a. Unfreezing b. Experiencing the change c. Moving d. Refreezing

: A Although you may be at a higher level of change in relation to your individual practice and knowledge of the use of change, in this situation, you are recognizing the need for change in relation to practice on the unit that you are managing. This phase is the initial phase in first-order change and will involve listening to staff to see if they perceive a similar problem. REF: Page 308 TOP: AONE competency: Leadership

23. Once evidence related to the use of prompted voiding in patients with cognitive impairment has been appraised and integrated with practice, it is important to: a. Consider whether patients' families see this as necessary for the well-being of family members. b. Search large databases such as CINAHL to amass further evidence. c. Clarify the clinical practice question. d. Solicit input regarding integration with practice.

: A EBP is the integration of best research evidence with clinical expertise and the patients' unique values and circumstances. In this situation, the family's values and preferences would be considered. Search of databases, clarification of the question, and gaining staff input would occur in the process. REF: Page 384 | Page 385 TOP: AONE competency: Knowledge of the Health Care Environment

1. Planning is a process designed to achieve goals in dynamic, competitive environments. As a new manager, what is the first step you will undertake to develop a strategic plan of action for a congestive heart failure program? a. Search the environment to determine changes that may affect the organization. b. Appraise the organization's strengths and weaknesses. c. Identify the major opportunities for and threats to the organization. d. Identify and evaluate the various strategies available to the organization.

: A Assessment of the external environment is the first step in the strategic planning process. REF: Page 293 TOP: AONE competency: Business Skills 2. A community-based pain management program is being planned for your region. As a coordinator of a home care agency, you have been requested to develop a conceptual framework, mission statement, philosophy, and objectives for the program. In what order will you pursue this endeavor? a. Objectives, philosophy, and mission statement b. Philosophy, objectives, and mission statement c. Philosophy, mission statement, and objectives d. Mission statement, philosophy, and objectives ANS: D The development of the mission statement is the first step in defining the purpose and direction of the organization after an assessment has been completed. REF: Page 294 TOP: AONE competency: Business Skills

17. Sarah determines, in partnership with her patient, that current medications are not enabling her patient, a married account executive with fibromyalgia, to continue with her employment and family responsibilities. After searching for additional information on fibromyalgia, Sarah finds nonpharmacologic interventions that are supported through credible evidence. Sarah suggests that the patient, her physician, and she meet to discuss the medications and possible options and a plan of care for the patient's discharge. This action exemplifies which of the four historical concepts identified by Lewis and Batey? a. Authority b. Responsibility c. Communication of conflict d. Autonomy

: A Authority refers to the use of professional status and power to act in the patient's best interests. In this example, Sarah is using her professional status and power to set up a conference in which her patient, the prescribing physician, and she can discuss what is not working for the patient and potential options. REF: Page 349 TOP: AONE competency: Professionalism

6. A Magnet™ hospital surveys the staff about job satisfaction. This type of environment, in which nurses have authority and autonomy, is linked with: a. Client satisfaction with the healthcare organization. b. Organizations with a limited number of nurse managers. c. Private, specialty organizations in urban areas. d. Sophisticated academic health sciences universities.

: A Autonomy and authority in decision making that is consistent with scope of practice are linked both to higher job satisfaction and to higher patient satisfaction with care. Job satisfaction is an important indicator of the quality of patient care. REF: Page 350 TOP: AONE competency: Business Skills

3. The nurse manager frequently interacts with staff and other hospice facility employees. Communication is purposeful because the manager assesses current issues, such as specific satisfactions and dissatisfactions with the newly implemented computerized documentation system. Informally, the manager gathers available staff members to address similar learning needs. Many times, staff members are found coaching other staff about improving use of the new system. According to Senge (1990), the activities demonstrated in this example are: a. Dialogue, team learning. b. Resilience, personal mastery. c. Shared vision, systems thinking. d. Mental models, teachable moments.

: A Building shared vision occurs when leaders involve all members in moving personal visions of the future into a consolidated vision common to members and leaders. REF: Page 314 TOP: AONE competency: Leadership

3. The Emergency Department staff decides to use a collective bargaining model for negotiation rather than a traditional trade union model. A traditional trade union model is characterized by: a. Positional conflict. b. Management support of labor's initiatives. c. A spirit of trust between management and labor. d. An ability to resolve complaints.

: A Collective bargaining encompasses management support of labor's initiative, a spirit of trust between labor and management, and resolution of problems. It replaces the positional conflict that has been associated with traditional trade unions. Models such as the interest-based problem solving (IBPS) model seek to avoid positional conflicts such as those between labor and management that do not take into account the opposing party in any way. REF: Page 354 | Page 357 TOP: AONE competency: Business Skills

7. The state of being emotionally impelled, demonstrated by a sense of passion and dedication to a project or event, describes: a. Commitment. b. Control. c. Willingness to cooperate. d. Communication.

: A Commitment is described as having a sense of passion and commitment to a project without necessarily having a need to control. REF: Page 329 TOP: AONE competency: Professionalism

16. Hospital Magnet™ decides against creating a separate department to lead and monitor quality activities because: a. Total organizational involvement is critical to QI. b. Data generated by a single, separate department are generally flawed. c. Monitoring and commitment to QI can come only from senior-level managers. d. Staff resent suggestions for improvement that originate outside of their unit.

: A Decentralized approaches are effective in developing unit-level solutions, as well as commitment to strategies and implementation of changes. REF: Page 362 TOP: AONE competency: Knowledge of the Health Care Environment

6. The chief nursing officer has to write a strategic plan. The most difficult stage in the strategic planning process is: a. Assessment of the external and internal environment. b. Review of mission statement, goals, and objectives. c. Identification of strategies. d. Implementation of strategies.

: A Determination of strengths, weaknesses, opportunities, and threats requires openness to what is being said and observed and is critical to setting the stage for relevant responses in the latter phases of strategic planning. REF: Page 293 TOP: AONE competency: Business Skills

7. The nurse manager of an ICU wants to implement the revised policy and procedure on central line catheter care. What would be the most effective method of getting the staff nurses to incorporate a new evidence-based practice into their care? a. Conducting an interactive educational workshop b. Distributing educational materials on clinical units c. Sharing the results of a chart audit with staff d. Providing staff with a short DVD on the topic

: A Dobbins et al. suggest that effective strategies for promoting behavioral change in health professionals include active involvement strategies such as face-to-face information sessions in small groups and one-to-one interactions. Other approaches have mixed or few results. REF: Page 394 TOP: AONE competency: Communication and Relationship-Building

4. The clinic nurse understands that the advantages of planning include: a. Assisting staff in critical thinking and improving decisions. b. Orienting people to react instead of act. c. Forcing managers to be cost-efficient and effective. d. Focusing on activities, not results.

: A Effective planning means that everyone in the organization manages his own work and sees how it relates to organizational goals. Being proactive means "aggressive planning" that ensures that decisions are improved and are oriented toward organizational goals. REF: Page 292 TOP: AONE competency: Leadership

2. The chief nursing officer at a local hospital seeking Magnet™ status creates staff development classes about incorporating evidence-based practice in nursing. What best describes evidence-based practice? a. Applying best research evidence to care of patients b. Using research-based information to develop practice guidelines c. Conducting a randomized control trial to determine effectiveness of handwashing techniques d. Developing standards for practice

: A Evidence-based medicine is derived from evidence-based medicine and involves integration of the best research evidence with clinical expertise and the patient's unique values and circumstances in making decisions about the care of individual patients. It is focused on searching for, appraising, and synthesizing the best evidence to address a particular clinical practice problem. REF: Page 385 TOP: AONE competency: Knowledge of the Health Care Environment

20. Through the QI process, the need to transform and change the admissions process across administrative and patient care units is identified. In this particular situation, what method of data organization will be most effective? a. Flowchart b. Histogram c. Narrative d. Line graphs

: A Flowcharts are useful in identifying and visualizing sequential steps, such as the admissions process. REF: Page 369 TOP: AONE competency: Business Skills

6. The mediator noticed that tension was still evident between the nurse manager and staff members. He informed the chief nursing officer that to begin team-building, it would be important that everyone: a. Work together in a respectful, civil manner. b. Use avoidance techniques when confronted with a conflict. c. Develop a personal friendship with each other. d. Socialize frequently outside of work.

: A For team-building to occur, team members need to be able to listen actively and respect one another's opinions, while feeling comfortable in openly expressing their own. REF: Page 338 TOP: AONE competency: Communication and Relationship-Building

21. Which of the following represents an effective goal statement? a. Involve patients and families in planning health care. b. To revise clinical guidelines for incontinence care from evidence within 6 months c. Patients and families are partners first in care planning. d. To acquire $4 million in additional funding within 12 months to support clinical education programs for dementia

: A Goal statements formalize targets for an organization. Objectives include the word to and an action word, and specify dates for attainment. Mission statements convey values and beliefs of the organization. REF: Page 295 TOP: AONE competency: Business Skills

1. A nursing unit is interested in refining its self-medication processes. In beginning this process, the team is interested in how frequently errors occur with different patients. To assist with visualizing this question, which organizational tool is most appropriate? a. Histogram b. Flowchart c. Fishbone diagram d. Pareto chart

: A Histograms are bar graphs that are useful in outlining and identifying frequency. REF: Page 369 TOP: AONE competency: Knowledge of the Health Care Environment

5. The nurse manager wants to use evidence-based recommendations to prevent ventilator-associated pneumonia. In order to effectively gather evidence to guide practice, what is the critical first step? a. Develop the clinical question. b. Identify the databases to be used. c. Appraise the evidence. d. Integrate available evidence with unit expertise.

: A Identifying the question may be the most challenging part of the process. Once the clinical question has been identified, writing it down will help in moving on to the next step of gathering evidence. REF: Page 395 | Page 396 TOP: AONE competency: Leadership

23. Which of the following would not be a characteristic of an effective team nurse leader? a. An autocratic perspective b. Excellent communication skills c. Awareness of everyone's abilities d. A genuine interest in team members

: A In ineffective teams, leadership tends to be autocratic and rigid, and the team's communication style may be overly stiff and formal. Members tend to be uncomfortable with conflict or disagreement, avoiding and suppressing it rather than using it as a catalyst for change. When criticism is offered, it may be destructive, personal, and hurtful rather than constructive and problem-centered. Team members may begin to hide their feelings of resentment or disagreement, sensing that they are "dangerous." This creates the potential for later eruptions and discord. REF: Page 323 TOP: AONE competency: Communication and Relationship-Building

12. To engage your staff in awareness of their current practice and how it is affirmed or not by evidence, you plan a short series of learning presentations on evidence and use of heparin and saline to maintain IV patency. You meet with the educator to plan out the goals for each session with the overall purpose of increasing knowledge and awareness of staff in readiness to consider questions related to the IV practice. This learning approach is an example of which change management approach? a. Linear b. First-order c. Facilitative d. Integrative

: A In the second stage, the moving or changing stage of Lewin's first-order, planned change process, planned interventions and strategies are executed to support the implementation of the change. One commonly used method is educating staff about the need for the change. REF: Page 307 TOP: AONE competency: Leadership

25. Sandra, an RN on the surgery unit, is assisting with a procedure in the patient examination room. The physician orders a medication to be given through IV. Sandra questions the order, based on her knowledge of the patient's history and of other medications that the patient has been given. The physician reiterates the order and Sandra refuses to give it. In this instance, Sandra is demonstrating: a. Autonomy. b. Accountability. c. Authority. d. Best practice.

: A In this situation, Sandra is exemplifying autonomy, which is the act of making independent decisions in the best interests of the patient, based on her knowledge and experience. This is analogous to the example in the text where the workers on the manufacturing floor have the independence to say "Stop the line" when something is wrong. Key to the concept of autonomy is decision making and the level of independence that is given. Accountability refers to achievement of outcomes, and authority refers to the capacity to make decisions. REF: Page 349 TOP: AONE competency: Knowledge of the Health Care Environment

12. Hospital ABCD is a Magnet™ hospital. This designation has been applied to Hospital ABCD because it: a. Facilitates active staff participation in decision making related to quality nursing care. b. Has implemented a graduate nurse orientation program. c. Espouses commitment to excellence in patient care. d. Is establishing career ladders for nurses.

: A Magnet™ hospitals are particularly successful in implementing excellence in patient care through use of standards, evidence, and participatory decision making in quality improvement. Organizations that cannot pursue Magnet™ status can implement strategies such as career ladders. REF: Page 362 TOP: AONE competency: Knowledge of the Health Care Environment

24. You are hired as a new manager. When the offer of employment is made, you agree to at-will employment. Later, you become very concerned about the policies and practices of your organization and their impact on patient care. You speak with your supervisor several times about your concerns, but no action is taken. In considering your next steps, you: a. Consider your increased vulnerability under the terms of your employment. b. Recognize that your supervisor is more vulnerable than you are because of her more senior position. c. Are more likely as a leader to take action because you are well protected from repercussions by federal and state regulations. d. Contact your union to discuss your concerns and review your options.

: A Managers of at-will employees have greater latitude in selecting disciplinary measures for specific infractions. State and federal laws do provide a level of protection; however, an at-will employee may be terminated at any time for any reason except discrimination. At-will employees, in essence, work at the will of the employer. Nurses in these positions need to know their rights and accountability. REF: Page 356 TOP: AONE competency: Business Skills

15. To meet the social and human needs of a target market, it is critical to: a. Assess the needs and priorities of the target market. b. Revise the organization's mission statement to reflect external needs. c. Assess the internal culture and needs. d. Develop highly sophisticated media.

: A Marketing emphasizes the identification and meeting of human and social needs, which can only be known through assessment of the needs of the consumer (as opposed to the needs of the organization). REF: Page 298 TOP: AONE competency: Business Skills

22. Which of the following is a meta-analysis? a. Review of 35 studies on nurse work satisfaction to determine the significance of the aggregated research findings b. Review of multiple chart audits to determine which errors are being reduced through implementation of evidence-based guidelines c. RCT comparing the effectiveness of a local anesthetic in reducing the pain of venipuncture in young children d. Analysis of factors contributing to nurse burnout and dissatisfaction at emergency room sites

: A Meta-analysis statistically combines the results of several similar studies to determine whether aggregate findings are significant. REF: Page 393 TOP: AONE competency: Knowledge of the Health Care Environment

10. As a new nurse manager who has "inherited" a unit with high nurse turnover and complaints of patient dissatisfaction, your first course of action would be to: a. Determine levels of nurse engagement on the unit. b. Review the personnel files of nurses who have resigned. c. Interview upper management about their vision for the . unit. d. Meet with your staff to clarify your vision for the unit.

: A Multiple studies demonstrate that a healthcare organization that provides a climate in which nurses have authority and autonomy has better patient outcomes, retains nurses at a higher rate, is more cost-effective, and has evidence of greater patient satisfaction than an organization in which such a climate does not exist (Aiken, Clarke, Sloane, Sochalski, & Silber, 2002; Dunton, Gajewski, Klaus, & Pierson, 2007). Organizational assessment assists in identifying the reasons for high nurse turnover and patient complaints. REF: Page 349 | Page 350 TOP: AONE competency: Knowledge of the Health Care Environment

13. A nursing-led classification system that has led to greater reliability and standardization in data utilized for QI processes is: a. NANDA. b. AHRQ. c. NIOSH. d. Nursing process.

: A NANDA has been developed by nurses and uses standardized terminology that enables study of health problems across populations, settings, and caregivers. REF: Page 373 TOP: AONE competency: Knowledge of the Health Care Environment

3. Northwestern Hospital has decided to implement peer review. As a clinical leader in the Emergency Department, you have overall responsibility for ensuring that this is in place by next month. The most important step in this process is to: a. Provide an educational session for staff on peer review. b. Revise the mission statement, objectives, and performance standards. c. Develop objectives and performance standards with employees. d. Implement objectives and performance standards.

: A Once a strategic plan has been developed, the next step is open communication and execution of the specific plan, which, in this instance, is peer review. REF: Page 296 | Page 298 TOP: AONE competency: Leadership

18. An example of an effective patient outcome statement is: a. Eighty percent of all patients admitted to the Emergency Department will be seen by a nurse practitioner within 3 hours of presentation in the Emergency Department. b. Patients with cardiac diagnoses will be referred to cardiac rehabilitation programs. c. The hospital will reduce costs by 3% through the annual budget process. d. Quality is a desired element in patient transactions.

: A Patient outcomes must be measurable, specific, and patient-centered. REF: Page 367 TOP: AONE competency: Knowledge of the Health Care Environment

3. A mediator suggested that the nurse manager and staff members decide on a method to resolve conflicts. It is important to have agreements about how team members will work together because: a. If there are no agreements, each member will make up rules about how to handle disagreements and relationships. b. People are naturally difficult and will not work well together without such agreements. c. People will naturally ask for agreements about how to be together. d. A way to eliminate nonproductive team members must be available.

: A People must agree on the goals and mission with which they are involved. They have to reach some understanding of how they will exist together. Tenets or agreements such as "I will respectfully speak promptly with any team member with whom I have a problem" go a long way to avoid gossiping, backbiting, bickering, and misinterpreting others. Without agreement, people have implicit permission to behave in any manner they choose toward one another, including angry, hostile, hurtful, and acting-out behavior. REF: Page 333 TOP: AONE competency: Communication and Relationship-Building

18. From the information presented in this chapter, which of the following statements best defines an accomplished team? Effective teams: a. Can create a form of synergism in which the outcome is greater than the sum of the individual performances. b. Do not necessarily need goals, objectives, vision, and a clearly stated purpose. c. Do not always have effective communication patterns. d. May or may not have a clear plan that is followed and revisited and has an ongoing evaluation scheme.

: A Research on team-building over many years has established that high-functioning teams are characterized by synergy that takes the team from a collection of individuals to an outcome that is greater than the sum of the parts. REF: Page 335 TOP: AONE competency: Communication and Relationship-Building

18. Elizabeth, an RN with approximately 15 years of service on your unit, walks away from one of the learning sessions on IV care and you overhear her telling a colleague that she thought the session was a waste of time because "the unit has been using heparin for years and there has never been any adverse effects." You follow up with Elizabeth and discover that she is really quite angry about the information sessions because she feels that you are implying that "what she has been doing all these years means that she is incompetent and doesn't care about her patients." After speaking with Elizabeth a few days later you discover that she is now fine with the change but is concerned that other areas of the organization might resist the change because of perceptions related to patient safety and cost. She suggests that it is important to bring pharmacy on board as they have had previous concerns about the use of heparin. In relation to change theory, this is indicative of: a. Systems level thinking. b. Linear thinking. c. Interprofessional collaboration. d. First-order change.

: A Senge's complexity theory, Bevan's Seven Change Factors, and general systems theory all highlight connectivity and the idea that changes are not isolated events. REF: Page 307 | Page 308 | Page 314 | Page 316 TOP: AONE competency: Leadership

5. Complex change situations require that the change leader promote ongoing visioning among staff members. One strategy is to: a. Consciously evaluate invisible mental models. b. Allow for individual outcomes. c. Encourage cooperative activities. d. Operate between order and disorder.

: A Senge's theory on change suggests that each individual or organization bases activities on a set of assumptions, or a set of beliefs, or mental pictures about the way that the world should work. When these invisible models are uncovered and consciously evaluated, it is possible to determine their influences on work accomplishment. REF: Page 314 TOP: AONE competency: Leadership

22. Senior executives at Hospital A determine that the hospital will engage in a strategic planning process after changes in healthcare funding and concerns expressed in the community about care that is being delivered at the hospital. The senior executives decide on a participatory process in which staff are widely consulted regarding input about the organization and the external environment and are actively invited to be part of decisions related to the mission statement, goals, and objectives. For true shared governance to be seen as part of this approach: a. It must be evident in the outcomes of the process that staff and senior executives have partnered on the decisions. b. Stakeholders must be assured of the value of their input even though final decisions rest with senior executives. c. Publications must clearly outline how staff input was solicited and obtained. d. Staff must be reassured that significant concerns will be kept in mind even if they have not been addressed in planning documents.

: A Shared governance demands participation in decision making. When partnership, equity, and ownership are not involved, then shared governance has not occurred, and publication and expressions of appreciation for input will not be seen as representative of shared governance. REF: Page 351 | Page 352 TOP: AONE competency: Knowledge of the Health Care Environment

14. In determining the relationship between injury-producing falls and proposed preventive measures as part of the QI process, a QI team might turn to which of the following for confirmatory evidence? a. NDNQI b. NANDA c. NIOSH d. AHRQ

: A The National Database of Nursing Quality Indicators is a national, nursing quality measurement program from the American Nurses Association that provides hospitals with unit-level performance reports with comparisons to national averages and rankings. REF: Page 372 | Page 373 TOP: AONE competency: Knowledge of the Health Care Environment

25. What is not a key concept in a well-functioning team? a. Absence of disagreement or conflict b. Special work that is understood and supported by all c. Willingness to work together respectfully d. Dedication to a mission

: A The challenge in teams is not to eliminate disagreement or conflict but to recognize when a breakdown in communication occurs. Singleness of mission, willingness to cooperate, and commitment are all key concepts in a well-functioning team. REF: Page 328 | Page 333 TOP: AONE competency: Communication and Relationship-Building

5. Nursing labor management partnerships: a. Engage nurses at all levels in problem solving for better patient care. b. Require unions and management to negotiate in good faith regarding hours of work and wages. c. Have been shown to have negligible effects on nurse turnover and patient outcomes. d. Have typically resulted in increased polarization of nurses and management, leading to formation of collective bargaining units.

: A The development of a nursing labor management partnership is an approach that can be used in most professional nursing environments. This process recognizes nurses as leaders on all levels and provides formal and informal mechanisms for professional nurses to work together to achieve shared goals through collaboration and shared decision making or decentralized decision making. A study of a nursing labor management partnership suggested that nurse satisfaction was higher, turnover was lower, and more time was available for patient care. REF: Page 355 TOP: AONE competency: Business Skills

7. The reasons cited by nursing administrators for not planning in a systematic manner include: a. Lack of understanding of the planning process. b. Lack of knowledge regarding the internal and external operations of the organization. c. Not enough hours in the day for both day-to-day operations and planning. d. Delegation of the task to subordinates.

: A The nurse manager should participate in strategic planning and quality initiatives with the governing body. Lack of knowledge is a common reason for nonparticipation in planning initiatives. REF: Page 293 TOP: AONE competency: Knowledge of the Health Care Environment

16. The implementation of saline flushes for capped angiocatheters is an example of: a. How multilevel and interprofessional application of a procedure can slow adoption of EBP. b. How competition among disciplines can lead to negative patient outcomes. c. The reluctance of hospital administrators to act on recommendations from EBP. d. How a safe, well-known practice outweighs the benefits of adopting a newer practice.

: A The translation of research into practice operates at four levels: The individual healthcare professional, healthcare groups or teams, organizations, and the larger healthcare system or environment. The adoption of saline flushes illustrates the challenges of communicating EBP to other disciplines and organizations and of the involvement of different levels. This particular innovation needed endorsement by nurses, physicians, and pharmacists, as well as by administrators who needed evidence of lost savings to support adoption. REF: Page 395 | Page 401 TOP: AONE competency: Leadership

20. Trust is an important aspect of helping relationships, therapeutic communications, and the positive communications model. Which statement does not involve or define trust? Trust: a. Involves decisions to manipulate situations to gain advantage over another. b. Is the basis by which leaders facilitate the activities and progress of a team. c. Is low among members and leaders in poorly performing teams. d. Involves what we say and not necessarily what we do.

: A Trust is high in high-performing teams and involves not consciously taking advantage of others and behaving in a way that inspires trust. It is the basis by which leaders facilitate the activities and progress of a team. REF: Page 333 | Page 334 TOP: AONE competency: Communication and Relationship-Building

23. Nurses in an Emergency Department, in an inner-city neighborhood characterized by high levels of violence, are concerned with low levels of security presence in their department. Security levels have recently been decreased and the nurses question why this has occurred. An appropriate action would be to: a. Provide nurses with information about rationale for recent changes in security staffing. b. Refer the matter to the head of security and let her deal with it. c. Provide mentors who can help nurses diffuse aggressiveness. d. Accept the security levels as a consequence of funding realities.

: A Workplace advocacy is an umbrella term encompassing activities within the practice setting. Workplace advocacy includes an array of activities undertaken to address the challenges faced by nurses in their practice settings. The focus of these activities is on career development, employment opportunities, terms and conditions of employment, employment rights and protections, control of practice, labor-management relations, occupational health and safety, and employee assistance. The objective of workplace advocacy is to equip nurses to practice in a rapidly changing environment. One manifestation of workplace advocacy is ensuring that relevant information is shared about decisions that affect practice so that further data gathering and decision making (in this instance about security levels and nurse safety) is informed. REF: Page 352 TOP: AONE competency: Communication and Relationship-Building

1. Martin, the unit manager, receives complaints from community agencies that patients who have been discharged from his unit seem to lack understanding about their disorder and immediate strategies for managing elements of their care. Martin checks the patient teaching sheets and notes that the sheets are initialed by staff. He calls the agencies and indicates that teaching has been done. Potential outcomes of Martin's actions include (select all that apply): a. Poor morale on the unit. b. Disruption in community relationships. c. Corruption of patient-staff relationships. d. Patient outcomes for quality care are met.

: A, B, C Kupperschmidt (2004) points out that when accountability is not accepted, then relationships suffer, professional practice is diminished, and self-esteem suffers. REF: Page 350 TOP: AONE competency: Professionalism Chapter 20: Managing Quality and Risk Yoder-Wise: Leading and Managing in Nursing, 6th Edition MULTIPLE CHOICE

1. Which of the following are examples of application of the Leadership Rounding Tool? (Select all that apply.) a. "What is working well for you during bedside reporting?" b. "What has not worked for you today?" c. "Is there someone on your team who deserves special recognition for her efforts in the implementation?" d. "Did you have a good vacation?"

: A, B, C, D The Leadership Rounding Tool suggests establishing and maintaining rapport and asking what is working well, what was a barrier, and who should be recognized, as well as answering tough questions. REF: Page 316 TOP: AONE competency: Communication and Relationship-Building

1. A nurse manager introduces prompted voiding into nursing practice on a unit, which is supported by clinical guidelines based on evidence-based practice. The nurses on the unit resist implementation, indicating that the bathroom facilities are too far away for efficient implementation of the guidelines, and that resources are too few to accomplish the initial voiding observations. For the nurse manager in this situation, it is important to have further discussion with the staff regarding (select all that apply): a. Compatibility of this intervention with the values of staff on the unit. b. Advantages of prompted voiding over incontinence products and catheterizations. c. Usefulness of prompted voiding with the particular population of patients on the unit. d. Feasibility of the program with respect to unit design.

: A, B, C, D Various theories related to the translation of evidence into practice point to strategies for success in introducing innovation and EBP on units, including determination of how well the innovation fits with the values of the staff; benefits of this practice or innovation over current practice; appropriateness of the innovation or practice for the target group for which the practice is intended; and the feasibility of the innovation from a variety of perspectives, including the physical design or layout of a unit. REF: Page 386 | Page 387 TOP: AONE competency: Leadership Chapter 16: Strategic Planning, Goal-Setting, and Marketing Yoder-Wise: Leading and Managing in Nursing, 6th Edition MULTIPLE CHOICE

1. Examples of sentinel events include (select all that apply): a. Forceps left in an abdominal cavity. b. Patient fall, with injury. c. Short staffing. d. Administration of morphine overdose. e. Death of patient related to postpartum hemorrhage.

: A, B, D, E Sentinel events are serious, unexpected occurrences involving death or physical or psychological harm. REF: Page 376 | Page 377 TOP: AONE competency: Knowledge of the Health Care Environment

. Before beginning a continuous quality improvement project, a nurse should determine the minimal safety level of care by referring to the: a. Procedure manual. b. Nursing care standards. c. Litigation rate of unsafe practice. d. Job descriptions of the organization.

: B Standards establish the minimal safety level of care. Procedure manuals provide information about how standards are to be achieved. REF: Page 367 TOP: AONE competency: Knowledge of the Health Care Environment

1. Mobilizing others to accomplish extraordinary things requires what leadership behaviors? (Select all that apply.) a. Celebrating the successes of others b. Demonstrating exceptional technical skills c. Imagining possibilities d. Establishing a sense of "being in this together"

: A, C, D Leaders who inspire teams to accomplish extraordinary things or to display synergy model the way, inspire shared vision, challenge the status quo, and encourage the heart by celebration of success. REF: Page 324 | Page 325 TOP: AONE competency: Leadership

11. In assessing the internal environment, the strategic planning team at Pacific Hospital discovers that 90% of staff members have no idea what the current mission statement means. This is most likely related to: a. The level of abstraction inherent in mission statements. b. Development of the statement by the administration team with no other staff involvement. c. Outdated goals and objectives contained in the mission statement. d. Failure to provide action statements within the mission statement.

: B A mission statement reflects the beliefs of the organization in relation to those it serves and services it provides, and it communicates the direction of an organization. Covey (1990) believes that the process of developing the mission statement is vital to an organization's success and should involve everyone. REF: Page 294 | Page 295 TOP: AONE competency: Business Skills

6. Martin, the unit manager, receives complaints from community agencies that patients who have been discharged from his unit seem to lack understanding about their disorder and immediate strategies for managing elements of their care. Martin checks the patient teaching sheets and notes that the sheets are initialed by staff. He calls the agencies and indicates that teaching has been done. Martin's follow-up to complaints from the community is: a. Appropriate and indicates that he has assumed accountability for the actions of his staff. b. Indicative that he does not clearly understand the concept of accountability. c. Indicative of strong support for his staff and their autonomy. d. Important in clarifying the difference between his accountability and that of the community in patient care.

: B Accountability refers to the achievement of desired outcomes. If community agencies are noticing that limited or no change in patient behavior has occurred despite teaching on the unit, then the staff has not achieved accountability, and he is not holding his unit responsible for the outcomes. Martin is also demonstrating lack of accountability. REF: Page 350 TOP: AONE competency: Professionalism

9. With the rise of violence in the psychiatric department, the nurse manager decides that she should work with the risk manager in violence prevention. The nurse manager should: a. Request all staff to accept new risk management practices. b. Hold staff accountable for safe practices. c. Document inappropriate behavior. d. Hire more police security.

: B Active involvement of staff in risk management activities is key to prevention of adverse events. Nursing has a primary role in leadership in optimizing patient outcomes, preventing patient care issues, and mitigating adverse events. Accountability for safety can be one aspect of performance evaluations. REF: Page 362 TOP: AONE competency: Knowledge of the Health Care Environment

4. By following a shared leadership model, the nurse manager believes that staff members will learn to function synergistically. Some teams function synergistically because members: a. Do not volunteer unwanted information. b. Actively listen to each other. c. Listen to the person who believes he or she is an expert. d. Do not speak unless they are absolutely sure they are correct in their views.

: B Active listening in a group creates synergy in that team members really hear one another's ideas and share in decision making. REF: Page 335 TOP: AONE competency: Professionalism

8. Before implementation of the new policy and procedure on central line catheter care, the nurse manager uses an appraisal system to evaluate the evidence. What is important in using an appraisal system to evaluate the evidence gathered in preparation for development of a new protocol? a. Limiting the search to randomized clinical trials b. Matching the appraisal tool to the type of evidence c. Eliminating qualitative research studies d. Using only pre-processed evidence

: B Appraisal tools are specific to the number of studies, as well as to the study design (type of evidence), type of review, and strategy for determining the applicability of evidence to your practice. REF: Page 397 TOP: AONE competency: Knowledge of the Health Care Environment

16. Assessment of the needs of the consumer or client (in developing a strategic marketing plan) focuses on: a. Present needs only. b. Present and future needs. c. The development of surveys. d. Secondary analysis of existing data.

: B Assessment of needs takes into account needs of the consumer now and for the future, using a variety of data-gathering techniques. Future orientation aids in anticipating how marketing and programs will need to shift to meet those needs. REF: Page 299 | Page 300 TOP: AONE competency: Business Skills

15. You are excited by evidence supporting the use of PDAs at the bedside to improve documentation and patient outcomes. You have disseminated the information through discussions and e-mails and are now ready to begin the process of considering implementation on the unit. To develop positive attitudes toward the use and implementation of the technology, you would discuss your ideas with (Rogers' diffusion of innovations theory): a. Harvey, RN, a technology guru, who enthusiastically tries on all new software. b. Berta, RN, who thoughtfully considers evidence and regularly uses it to try new approaches in her practice. c. Carol, LPN, who is positive about new ideas but looks to her friends for their ideas about whether or not to try something new. d. Ben, a 10-year veteran of the unit, who wonders why technology should be used at all. He says that he will use it when there is no chance of security breaks.

: B Berta is the one with whom you should now have informed conversations, because she is an early adopter who is respected for her thoughtful acquisition and critique of knowledge and application of knowledge to practice. Berta, an early adopter (Rogers' characteristics of innovation adopters), is more effective in this stage than Harvey, an innovator who may be seen as open to all new ideas regardless of merit. REF: Page 392 TOP: AONE competency: Communication and Relationship-Building

19. Patient perceptions are useful in: a. Determining disciplinary actions in QI. b. Establishing the competitive advantage of QI decisions. c. Providing one source of data for QI initiatives. d. Establishing blame for poor-quality care.

: C Customers define quality and patient dissatisfaction as useful indicators of which areas are of greatest concern to patients and of what matters then to nurses and organizations. Patient perceptions guide areas of inquiry; however, they do not establish what disciplinary decisions will be made. REF: Page 366 | Page 367 TOP: AONE competency: Knowledge of the Health Care Environment

11. Healthcare organization X is committed to improving patient outcomes and, as part of the QI process, examines its executive structure and organizational design. This approach recognizes: a. The importance of decentralized structure in QA. b. That structure influences nurse burnout and participation in quality improvement initiatives. c. The need to ensure sufficient supervisory staff to respond in a corrective manner when mistakes occur. d. That a narrow hierarchy ensures accountability for errors and outcomes.

: B Common organizational characteristics of Magnet™ hospitals include structure factors (e.g., decentralized organizational structure, participative management style, and influential nurse executives) and process factors (e.g., professional autonomy and decision making, ongoing professional development/education, active quality improvement initiatives). ANCC Magnet™ designated hospitals and other high-reliability organizations in the United States and Europe generally have lower burnout rates, higher levels of job satisfaction, and provide higher levels of quality care resulting in greater levels of patient satisfaction (Aiken et al., 2012; Kelly, McHugh, & Aiken, 2011). REF: Page 362 TOP: AONE competency: Knowledge of the Health Care Environment

4. As a nurse manager, you trial a new pain scale on your unit that is supported by numerous research studies. You compare the patient outcomes with the new scale against the existing scale. Feedback from staff suggests that the new scale is too difficult for patients who have limited language skills and who are already under duress to understand. The difficulty in implementing the new scale refers to testing: a. Efficacy. b. Effectiveness. c. Practice failure. d. Comparative error.

: B Comparing the effectiveness of interventions can help to address the needs of clinicians in determining best practices for their patients. Comparative effectiveness research (CER) is the "generation and synthesis of evidence that compares the benefits and harms of alternative methods to prevent, diagnose, treat, and monitor a clinical condition or to improve the delivery of care" (Institute of Medicine, 2009, p. 29). Efficacy is testing an intervention or treatment in a traditional randomized clinical trial under carefully controlled conditions and is used to determine whether an intervention or treatment works, whereas effectiveness is testing whether the intervention or treatment works in the real world of practice. REF: Page 389 TOP: AONE competency: Knowledge of the Health Care Environment

19. Based on Elizabeth's insights and suggestions, you involve pharmacy, only to discover that the change in practice involves practice committees, a medical practice committee, and concerns from administration about potential costs and safety of the proposed change to the IV protocols. The change process at this point is: a. Linear. b. Nonlinear. c. Sabotaged. d. Neutralized.

: B Complex change involves nonlinear processes and a variety of strategies to negotiate influences on change. Complexity theories alter the traditional systems thinking approach by asserting that system behavior is unpredictable. This theory views change as emergent, nonlinear, and highly influenced by all individuals and subsystems in an organization. REF: Page 309 TOP: AONE competency: Leadership

10. An example of one strategy used to improve participation in the change process by staff fitting the behavioral description of innovators and early adopters is to: a. Repeat the benefits of the change. b. Share change experiences early in the process. c. Initiate frequent interactions among staff. d. Provide select information to the staff.

: B Connecting innovators and early adopters to new ideas and with new peers keeps them at the cutting edge. REF: Page 311 TOP: AONE competency: Professionalism

19. Government and third-party payers announce reduction of compensation for the delivery of patient services. Hospital STV has a flat organizational structure. After the funding announcements, senior officials at the hospital meet and make decisions regarding cost containment of new revenue streams. This action is consistent with: a. The practice of leaving financial decisions with senior officials who understand the total context of funding. b. A tendency to concentrate decision making during economic downturns at the top administrative level. c. A need to make expedient decisions that are likely to be poorly received by staff. d. Ensuring that decisions with regard to cost are made equitably across all departments.

: B During times of economic downturn, decisions tend to become very centralized to avoid risk. History demonstrates that increasing the breadth of input during these times is more effective than narrowing it. REF: Page 351 TOP: AONE competency: Knowledge of the Health Care Environment

12. In accomplishing the goal of breakfast for children in elementary school, Leanne is particularly effective in approaching businesses with the needs that the group has determined and articulating the ways that the group has found for businesses to participate. Leanne is exemplifying: a. Leadership. b. Followership. c. Professionalism. d. Knowledge of context.

: B Effective followership involves active and loyal involvement in an agenda that has been established. In this role, Leanne is supporting and operationalizing the agenda and strategies that have been established within the group. REF: Page 349 TOP: AONE competency: Communication and Relationship-Building

21. On Unit 62, the nurses and the unit manager have been involved in shared decision making related to the model of nursing care delivery that the unit will adopt. All individuals have participated and been involved in decision making and implementation of changes. When issues arise during implementation, it is expected that: a. Accountability resides entirely with the unit manager. b. Individual expertise will be utilized to provide solutions, but that responsibility for the change is shared. c. No one really has any accountability or responsibility for the changes. d. This will contribute to widespread skepticism among the staff about the probability of success.

: B High-performing organizations provide for participation by all stakeholders, and each stakeholder shares responsibility and risk. This kind of environment is more satisfying for nurses and is characterized by optimism and trust. REF: Pages 350-352 TOP: AONE competency: Knowledge of the Health Care Environment

10. A new RN staff member asks you about the difference between QA and QI. You explain the difference by giving an example of QI. a. "Last year, the management team established new outcomes that addressed issues such as medication errors." b. "At a staff meeting last year, two of our staff commented on the number of recent falls and asked, 'What can we do about it?'" c. "A process audit was done recently to determine how much time was being spent on patient documentation." d. "Errors are reported on our new computerized forms, and I follow up with staff to make sure that they understand the seriousness of their error."

: B In QI, followers invest in the process by continually asking "What makes this indicator important to measure?" "What has been done to improve it?" "What can I do to improve it?" REF: Page 365 TOP: AONE competency: Knowledge of the Health Care Environment

. The nurse educator of the pediatric unit determines that vital signs are frequently not being documented when children return from surgery. According to quality improvement (QI), to correct the problem, the educator, in consultation with the patient care manager, would initially do which of the following? a. Talk to the staff individually to determine why this is occurring. b. Call a meeting of all staff to discuss this issue. c. Have a group of staff nurses review the established standards of care for postoperative patients. d. Document which staff members are not recording vital signs, and write them up.

: B Leadership must identify safety shortcomings and must locate resources at patient care levels to identify and reduce risks. One method of doing this is to invite all staff into a discussion related to solutions to an identified concern. This approach encourages teamwork. REF: Page 364 TOP: AONE competency: Knowledge of the Health Care Environment

15. You are charged with developing a new nursing curriculum and are committed to developing a curriculum that reflects the needs of the profession and of the workplace. To address deficits that may already be present in nursing curricula related to the workplace, you include more content and skills development related to: a. Therapeutic communication with patients. b. Effective communication in the workplace. c. Increased emphasis on sender-receiver dyads. d. Generational differences in communication.

: B Nursing programs teach therapeutic communications with patients and their families. Little focus, however, is placed on effective communication in the workplace, although communication is essential to building and maintaining smoothly functioning teams. REF: Page 325 TOP: AONE competency: Communication and Relationship-Building

22. The outcome statement "Patients will experience a ten percent reduction in urinary tract infections as a result of enhanced staff training related to catheterization and prompted voiding" is: a. Physician-sensitive and nonmeasurable. b. Measurable and nursing-sensitive. c. Precise, measurable, and physician-sensitive. d. Patient care-centered and nonmeasurable.

: B Nursing-sensitive outcomes refer to outcomes that are affected by nursing activity and are precise, measurable, and patient-centered. REF: Page 367 | Page 368 TOP: AONE competency: Knowledge of the Health Care Environment

15. Nurses on the dialysis unit notice that changes in labeling of fluids have meant several alarming near misses in terms of wrong administration of fluids. They take this concern to the unit manager. As an advocate of nurse autonomy, the most appropriate response in this situation would be to: a. Ensure that the nurses are aware of the reasons for the change and how the decision was made about the new labels. b. Discuss concerns about the labels and develop potential solutions that take into account changes that can be made at the local level and those that need system intervention. c. Suggest that the staff wait until they have become more familiar with the labels before taking further action. d. Tell the staff that you will notify the pharmacy about these concerns and leave it up to the pharmacy to decide what should be done.

: B Participation in decision making regarding one's practice is an appropriate expectation for professional nurses, provides for greater autonomy and authority over practice decisions, contributes to supporting the professional nurse, and is a major component of job satisfaction (Kramer et al., 2008; Pittman, 2007). Autonomy is encouraged through supportive management and through unit-level support of changes without the need for complex, multilevel approval of changes that can be made locally. REF: Page 348 | Page 349 TOP: AONE competency: Leadership

10. What is a strategy that can be used by a small community hospital with limited resources to develop an evidence-based nursing practice program? a. Hiring a nurse researcher b. Partnering with nurse researchers at a local university c. Subscribing to journals devoted to evidence-based nursing d. Including research competencies in managers' job descriptions

: B Partnering with nurse researchers assists in providing nurse researcher expertise and leadership to organizations that do not have the size or the resources to hire nurse researchers. REF: Page 400 TOP: AONE competency: Communication and Relationship-Building

3. A nurse manager wants to decrease the number of medication errors that occur in her department. The manager arranges a meeting with the staff to discuss the issue. The manager conveys a total quality management philosophy by: a. Explaining to the staff that disciplinary action will be taken in cases of additional errors. b. Recommending that a multidisciplinary team should assess the root cause of errors in medication. c. Suggesting that the pharmacy department should explore its role in the problem. d. Changing the unit policy to allow a certain number of medication errors per year without penalty.

: B Quality management stresses improving the system, and the detection of staff errors is not stressed. If errors occur, reeducation of staff is emphasized rather than imposition of punitive measures such as disciplinary action or blaming. REF: Page 365 | Page 366 TOP: AONE competency: Knowledge of the Health Care Environment

13. To engage your staff in awareness of their current practice and how it is affirmed or not by evidence, you plan a short series of learning presentations on evidence and use of heparin and saline to maintain IV patency. You meet with the educator to plan out the goals for each session with the overall purpose of increasing knowledge and awareness of staff in readiness to consider questions related to the IV practice. Staff nurses who gain information on current IV therapy practices are engaging in which phase of Rogers' decision-making process? a. Persuasion b. Knowledge c. Confirmation d. Decision

: B Rogers' innovation-decision process involves five stages for change in individuals, the first of which is knowledge. REF: Page 310 TOP: AONE competency: Leadership

6. A strategic goal developed by the chief nursing officer is to implement an evidence-based practice program. What is an appropriate strategy that can be used by a nurse manager who is beginning to implement an evidence-based practice program? a. Conducting a review of adverse events and incident reports b. Soliciting input from staff members c. Reviewing specialty organization guidelines d. Identifying patients with extended lengths of stay

: B Stakeholders need to be involved early, and staff members need to be involved when initiatives involve direct patient care. Involvement assists in understanding issues and concerns, motivations, and unmet needs. REF: Page 398 TOP: AONE competency: Communication and Relationship-Building

11. Tara, the unit manager, is telling her colleague about her recent project, which involves seeking the most effective approaches to incontinence care, with the intention of adopting evidence-supported approaches on her dementia care unit. Her colleague suggests that translation of research into practice is: a. Less important than knowledge-generating research, which is required to advance the nursing profession. b. A priority of all healthcare practitioners to improve patient care. c. Characterized by lack of knowledge about how to use evidence to guide practice. d. So difficult that it is useless to begin the query in the first place.

: B The National Institutes of Health identified translational research, or getting research into the hands of practitioners to improve patient care, as a priority. REF: Page 385 TOP: AONE competency: Knowledge of the Health Care Environment

13. The clinical guidelines for management of incontinence developed by the Registered Nurses Association of Ontario (RNAO): a. Reflect practice that is fiscally directed and sound. b. Articulate practice recommendations developed from synthesis and review of evidence. c. Are intended to increase awareness of issues in incontinence management. d. Reflect a compilation of information from a variety and range of sources related to incontinence.

: B The evidence-based practice (EBP) movement has grown exponentially with scientific publications, establishment of collaboration centers, resources on the Web, and grants focused specifically on translating of research into practice. A number of evidence-based nursing centers have been established around the world. These centers have teams of researchers who critically appraise evidence and then disseminate protocols for the use of evidence in practice. In this example, clinical guidelines have been developed by a professional nursing organization on the basis of best possible evidence on incontinence management. Although issues may be raised in the recommendations, the purpose is to guide practice for the purposes of better patient care. REF: Page 388 TOP: AONE competency: Knowledge of the Health Care Environment

19. Shifts such as changes in the percentage of seniors in the population, governmental initiatives in health care, and the influence of income on health status are factors in the __________ environment. a. Public b. Macro c. Competitive d. Social

: B The external environment in strategic marketing planning includes the public, competitive, and macro environments. Demographic, social, and political factors are included in the macro environments. REF: Page 299 TOP: AONE competency: Business Skills

8. The mediator suggested to the unit staff that a group agreement needed to be made so meetings could become productive. For example, the group agreement, "We will speak supportively," prevents: a. Expression of opposing ideas. b. Gossip and making negative comments about absent team members. c. Efforts to ensure that everyone thinks alike. d. Votes that oppose motions.

: B This is an example of a rule that a team can implement to prevent certain negative behaviors such as gossip, backbiting, and bickering that undermine the productivity and functioning of a group. REF: Page 323 | Page 324 | Page 333 TOP: AONE competency: Professionalism

5. As a nursing leadership student, you have had the opportunity to develop a palliative care manual that will be utilized by the palliative care network (PCN) in your region for teaching healthcare professionals. The PCN has requested that you provide a prioritized plan of action for marketing the manual on a regional and state basis. What is the first priority? a. Motivate the target market. b. Research the target market. c. Communicate benefits to the target group. d. Package the product.

: B To put a strategic plan into place for marketing, it is necessary to conduct an external assessment. REF: Page 299 TOP: AONE competency: Business Skills

1. When goals/outcomes are somewhat unclear in early preparation for a complex change, the manager and the change management team develop several acceptable goals/outcomes. This change in management approach is termed: a. Unfreezing. b. Nonlinear. c. Cybernetic. d. Linear.

: B While Lewin's theory was designed to describe planned or first-order changes, many scholars think the theory is too simplistic to address how unplanned or second-order change occurs. In complex situations with an uncertain change environment, a nonlinear approach that involves flexibility improves overall outcomes. Linear change is appropriate to stable, less complex, and more predictable situations. REF: Page 309 TOP: AONE competency: Leadership

9. The Emergency Department staff members are concerned that working long hours without rest puts patient safety at risk. One staff member decides that she will risk her job and become a whistleblower. Whistle-blowing is an appropriate recourse when management: a. Disregards due process when disciplining a nurse. b. Delays responding to repeated efforts to provide safe care. c. Hires nurses who are not a part of the union during a strike. d. Refuses to bargain in good faith with the elected bargaining agent.

: B Whistle-blowing is often a result of organizational failure, including failure of the organization to respond to serious danger or wrongdoing created within the environment, which, in this instance, involves conditions that put the patient at risk. REF: Page 349 TOP: AONE competency: Business Skills

17. Which of the following would be most effective in implementing the findings of Dobbins et al. on treating problems associated with bowel motility? a. Lecture by a nurse practitioner b. Workshop for surgical nurses that involves discussion of case studies and application of evidence c. Discussion of the findings on the bulletin boards at the workstation d. Education of unit opinion leaders regarding the evidence presented in the studies

: B Work by Dobbins et al. suggests that translation of research into practice is best facilitated through interactive learning such as workshops. Least effective strategies included didactic learning and distribution of learning materials. REF: Page 394 | Page 395 TOP: AONE competency: Communication and Relationship-Building

4. The chief nursing officer utilizes the hospital's workplace advocacy to help the overwhelmed Emergency Department staff. Workplace Advocacy is designed to assist nurses by: a. Creating professional practice climates in their institutions. b. Equipping them to practice in a rapidly changing environment. c. Negotiating employment contracts. d. Representing them in labor-management disputes.

: B Workplace advocacy encompasses a number of activities that enable nurses to control the practice of nursing and to address challenges that they face in the practice setting. These activities include career development, employment rights, employment opportunities, and the labor-management relationship. The aim of workplace advocacy is to proactively equip nurses to practice within a rapidly changing environment, rather than to negotiate contracts or provide representation in employment disputes. REF: Page 352 TOP: AONE competency: Business Skills

24. As a nurse manager, you notice that one of your new nurses has provided exceptional care for a patient with especially complex needs. What would be the MOST effective way of recognizing the nurse's performance? a. At the next performance review, note specifically what the nurse did to make the patient comfortable. b. To avoid embarrassing the nurse in front of others, find a way to compliment the nurse in private. c. When the nurse comes out of the room, tell the nurse specifically what you appreciated about the care that was provided. d. Encourage the patient to note the care on the patient feedback form so that the institution can recognize the nurse's efforts.

: C Acknowledgement is most effective when it is specific, timely, given in public, sincere, and on an eye-to-eye basis. The more time that elapses between the event and acknowledgement, the less effective it becomes. REF: Page 336 TOP: AONE competency: Communication and Relationship-Building

12. Pacific Hospital's team develops a process to include staff in development of the mission statement. Which of the following statements would indicate that the staff has been able to develop an effective mission statement? a. Pacific Hospital will increase its sustainability by increasing revenue streams within the next three years. Revenue streams are important to our hospital and are a driving force in our sustainability in the face of insurer and government funding changes. b. Pacific Hospital has a desire to relate to its community to enable appropriate identification of programming needs by July 2012. c. Pacific Hospital will provide family-centered care to the community of Pacific Rim. It is committed to delivering a full spectrum of services that foster collaborative partnerships among clients, families, and the hospital, and it utilizes evidence-based practice to guide planning and interventions. d. To provide health promotion programs for school-aged children

: C An effective mission statement reflects the values and direction of the organization. Included in the statement are beliefs about individuals, health, practitioners, and the relationship of the organization to practice, research, professionalism, and/or education. REF: Page 294 | Page 295 TOP: AONE competency: Business Skills

18. Nurses are valuable partners in marketing strategies because they: a. Are more likely than other members of the organization to use marketing tools. b. Have a strong background in using evidence to support decisions. c. Have close involvement with patients, who are the target group for marketing. d. Constitute the largest staff group in most healthcare organizations.

: C As nurses are directly involved with patients who use services and who are the target group for marketing efforts, nurses have excellent insights into patient needs. REF: Page 299 TOP: AONE competency: Communication and Relationship-Building

10. To conduct assessment of the internal environment, the strategic planning team for Pacific Hospital: a. Invites community members and staff of the hospital to an evening focus group session. b. Discusses what it sees as the primary threats and opportunities in demographic shifts affecting the hospital. c. Invites all levels of staff to focus groups on the effectiveness of the hospital environment, including information systems and staffing. d. Asks the board to provide a summary of major opportunities for the future.

: C Assessment of the internal environment should include all levels of staff and facilitates feedback on effectiveness of strengths and weaknesses of systems and processes within the institution. REF: Page 294 TOP: AONE competency: Business Skills

1. A new graduate is asked to serve on the hospital's quality improvement (QI) committee. The nurse understands that the first step in quality improvement is to: a. Collect data to determine whether standards are being met. b. Implement a plan to correct the problem. c. Identify the standard. d. Determine whether the findings warrant correction.

: C Before further action (data collection, decision making related to correction, and implementation of a plan) can occur, it is necessary to identify the standards against which data collection and decision making will occur. Institutions may or may not adopt standards that are already established by organizations such as the ANA. REF: Page 369 TOP: AONE competency: Knowledge of the Health Care Environment

23. Your institution has identified a recent rise in postsurgical infection rates. As part of your QI analysis, you are interested in determining how your infection rates compare with those of institutions of similar size and patient demographics. This is known as: a. Quality assurance. b. Sentinel data. c. Benchmarking. d. Statistical analysis.

: C Benchmarking is a widespread search to identify the best performance against which to measure practices and processes. REF: Page 371 TOP: AONE competency: Knowledge of the Health Care Environment

20. At an organizational level, which of the following strategies would assist in ensuring that EBP is incorporated into nursing care? a. Formation of nursing-only implementation teams b. Restriction of evidence to RCTs c. Formation of a network of individuals doing research and/or interested in research utilization d. Avoidance of partnership with experienced researchers

: C Collaboration, partnerships, and consideration of a variety of evidence appropriate to the clinical questions are important in ensuring translation of evidence into practice. Collaboration is considered particularly critical and can occur through practice-based networks. REF: Page 401 TOP: AONE competency: Knowledge of the Health Care Environment

11. In an inner-city area, a group of nurses meet and develop a plan to negotiate with local businesses to support a breakfast program for young elementary schoolchildren. This is an example of: a. Community development. b. Collective bargaining. c. Collective action. d. Shared governance.

: C Collective action refers to activities undertaken by a group of people with common interests and, in this example, by a group of nurses who are interested in the welfare of children in their community. REF: Page 347 TOP: AONE competency: Communication and Relationship-Building

9. The mediator asked each staff member to reflect on his or her communication style. Which of the following best describes communication? Communication: a. Is a reflection of self-analysis. b. Is a result of thoughtful consideration. c. Consists of thoughts, ideas, opinions, emotions, and feelings. d. Focuses on the sender of the message.

: C Communication involves both senders and receivers and may or may not be a reflection of self-analysis and thoughtful consideration. It always, however, involves thoughts, ideas, opinions, emotions, and feelings. REF: Page 325 TOP: AONE competency: Communication and Relationship-Building

16. Elizabeth, an RN with approximately 15 years of service on your unit, walks away from one of the learning sessions on IV care and you overhear her telling a colleague that she thought the session was a waste of time because "the unit has been using heparin for years and there has never been any adverse effects." You follow up with Elizabeth and discover that she is really quite angry about the information sessions because she feels that you are implying that A"what she has been doing all these years means that she is incompetent and doesn't care about her patients." Which of the following would be the most effective response to Elizabeth? a. "I understand how you feel, but you are going to have to change." b. "It is unfortunate that you feel this way. Others seem quite excited about the new information." c. "It is difficult sometimes to change what we know very well. Sometimes it can be frightening." d. "Perhaps I can arrange some more information sessions for you, so you can see just how important this change is to patient safety."

: C Dialogue can reveal areas where individuals feel inept or overwhelmed, providing the leader with an understanding of what programs need to be developed to increase personal ability to change and what educational initiatives need to be implemented to support change. To promote dialogue, leaders must serve as facilitators, promoting the sharing of ideas, fears, and honest reactions to the change proposal. REF: Page 315 TOP: AONE competency: Communication and Relationship-Building

5. The chief nursing officer decided that the nurse managers need a series of staff development programs on team-building through communication and partnerships. She understood that the nurse managers needed to build confidence in ways of handling various situations. The greatest deterrent to confidence is: a. Lack of clarity in the mission. b. Lack of control of the environment. c. Fear that one can't handle the consequences. d. Fear that the boss will not like one's work.

: C Fear of not being able to manage consequences undermines confidence and a sense of competency. REF: Page 341 TOP: AONE competency: Professionalism

23. You anticipate that your region will soon move towards an e-health record system. You begin to discuss this with your staff and are disappointed that you receive little positive response from the staff about this possibility. One staff member, in particular, seems to sum it up by saying "e-health? Won't happen in my working life! There are too many problems with it, like privacy issues." This response is most likely motivated by: a. Lack of urgency regarding the need to change. b. Lack of evidence to support importance of technology. c. Deficits in education and experience. d. Lack of organizational support for change.

: C For leaders to inspire change, they must have intimate knowledge of what matters to the people they manage. Kotter (2012) characterizes this as establishing a sense of urgency, and this involves overcoming complacency. This is especially hard when there doesn't seem to be any visible crisis, or the crisis seems irrelevant to the people being asked to change (Kotter, 2012). REF: Page 314 | Page 315 TOP: AONE competency: Leadership

2. The Emergency Department nurses' decision to organize for the purpose of collective bargaining is being driven by a desire to: a. Establish the staffing pattern that will be used. b. Determine the hours that one is willing to work. c. Create a professional practice environment. d. Protect against arbitrary discipline and termination.

: C Historically, nurses were reluctant to unionize. However, concern with safety of care and quality of care, especially when tension is present in a work environment, makes unionization more desirable. U.S. Supreme Court rulings have provided for RN-only units and protection to practice according to what the profession and licensure status require nurses to do. REF: Page 356 TOP: AONE competency: Business Skills

. The clinical coordinator expects the position description of the new wound care specialist to change nurses' responsibilities in caring for clients with skin integrity problems. The best approach to address this need for change, yet to have the best outcomes for clients, staff nurses, and the organization, is to: a. Select one of the change models. b. Use Lewin's model and principles of change. c. Apply both planned and complexity theory approaches. d. Form a task force of nursing staff and wound care specialists.

: C In the second stage, the moving or changing stage of Lewin's theory, planned interventions and strategies, such as education, vision building, and incremental steps towards the change, are executed to support the implementation of the change. This situation potentially also involves complexity theories that recognize that change involves engagement of individuals and subsystems throughout the unit and organization. REF: Page 308 | Page 309 TOP: AONE competency: Leadership

8. The nurse manager is concerned about the negative ratings her unit has received on patient satisfaction surveys. The first step in addressing this issue from the point of view of quality improvement is to: a. Assemble a team. b. Establish a benchmark. c. Identify a clinical activity for review. d. Establish outcomes.

: C In theory, any and all aspects of clinical activity could be improved through the QI process. However, QI efforts should be concentrated on changes to patient care that will have the greatest effect. REF: Page 364 | Page 366 | Page 380 TOP: AONE competency: Knowledge of the Health Care Environment

21. As the unit manager on the unit that is leading changes to heparin locks, you find that Elizabeth is very valuable in terms of her observations about other units and her knowledge of organizational processes, and now in discussing the new procedure with others. Elizabeth might be considered an: a. Engager. b. Innovator. c. Informal change agent. d. Informant.

: C Informal change agents are those who do not have formal, positional power but who have credibility through expertise and can model the new way of thinking, or who offer suggestions, ideas, and concerns. REF: Page 315 TOP: AONE competency: Leadership

20. In Hospital STV, senior administration is strongly oriented toward fiscal and social conservatism. The nursing department is deeply concerned with the provision of quality to the community, which includes a high number of poor and unemployed. To accomplish the goals of the nursing department, resources need to be allocated that administration is not able to allocate. Nursing and administration: a. Are engaged in shared governance. b. Are involved in an irreconcilable conflict of interests. c. Represent separate subcultures in the institution. d. Represent union and nonunion conflict.

: C Institutions can have several subcultures, which are represented by unique features and distinct ideologies. Subcultures can be congruent and can support healthy relationships in the organization, or they may be separated and characterized by tensions that may be irreconcilable and destructive. From the information given in this scenario, it is not evident that shared governance, union presence, or irreconcilable differences are present. REF: Page 351 TOP: AONE competency: Knowledge of the Health Care Environment

9. To help staff nurses adjust to using research in practice, what strategy would the nurse manager use? a. Attendance at a regional research conference b. Formal classes in electronic search techniques c. Establishing a journal club d. Issuing reports on the adverse consequences of outdated practices

: C Journal clubs provide opportunities for engagement in reading research and considering how it might be applied to clinical practice problems, which is considered very effective in behavioral change. REF: Page 402 TOP: AONE competency: Communication and Relationship-Building

17. Elizabeth is an example of a(n): a. Early adopter. b. Late majority. c. Laggard. d. Resister.

: C Laggards prefer keeping traditions alive ("We have always done it this way") and openly express their resistance to ideas (speaking out against the change). Late majority individuals are openly negative but will engage with new ideas when most others adopt the change. REF: Page 311 TOP: AONE competency: Leadership

25. Edith has been vocal about her negative concerns related to a new charting system and frequently expresses the view that keeping the "old system" would have been "just fine." In facilitating change, your best approach to Edith would be to: a. Put her in the pilot planning group for the change. b. Determine if she has considered retirement. c. Schedule her work assignment so that it coincides with those of two staff members who are confident with technology and the change. d. Avoid discussion of the change, and trust that with sufficient training and information, she will change.

: C Laggards prefer keeping traditions and openly express their resistance to new ideas. Having a group of change agents and innovators on board to champion an idea builds what Patterson et al. call "social motivation" and "social ability." This group can help staff, such as laggards, who are less adept at change. REF: Page 311 | Page 315 TOP: AONE competency: Leadership

3. The nurse manager decides to change staffing schedules in the intensive care unit. She chooses to use Rogers' diffusion of innovations theory to implement the change. What are the elements of Rogers' diffusion of innovations theory? a. Assessment, planning, intervention, and evaluation b. Preparation, validation, synthesis, and evaluation c. Knowledge, persuasion, decision, implementation, and evaluation d. Scanning, diagnosis, decision making, application, and reevaluation

: C Nurse managers disseminate research findings to patient care team members. Rogers' diffusion of innovations theory provides a useful model for integration of evidence into practice. REF: Page 391 TOP: AONE competency: Leadership

21. Team Member A and Team Member B engage in heated disagreements on a frequent basis in team meetings. Their behavior is characterized by insistence on their points of view and refusal to back down or to negotiate alternative solutions once their ideas have been expressed. This behavior is characteristic of: a. Autocratic leadership. b. Constructive conflict. c. Dualism. d. Creativity.

: C Our society tends to be dualistic in nature. Dualism means that most situations are viewed as right or wrong, black or white. Answers to questions are often reduced to "yes" or "no." As a result, we sometimes forget a broad spectrum of possibilities actually exists. Exercising creativity and exploring numerous possibilities are important. This allows the team to operate at its optimal level. REF: Page 337 TOP: AONE competency: Communication and Relationship-Building

2. The chief executive officer asks the nurse manager of the telemetry unit to justify the disproportionately high number of registered nurses on the telemetry unit. The nurse manager explains that nursing research has validated which statement about a low nurse-to-patient ratio? It: a. Promotes teamwork among healthcare providers. b. Increases adverse events. c. Improves outcomes. d. Contributes to duplication of services.

: C Studies related to staffing and patient outcomes suggest that patient outcomes are improved with a low nurse-to-patient ratio and especially with a low registered nurse-to-patient ratio. REF: Page 376 TOP: AONE competency: Knowledge of the Health Care Environment

2. The nurse manager used a mediator to help resolve conflicts on the unit. During the mediation process, the nurse manager saw signs of potential team-building. One key concept of an effective team is: a. Conflict. b. Task clarity. c. Commitment. d. A designated leader.

: C Team-building involves moving toward a common vision, which requires commitment. Conflict and clarification of tasks are components in the development of this commitment. REF: Page 323 TOP: AONE competency: Communication and Relationship-Building

16. The SBAR system of communications is one of the most used communication systems in health care because: a. It deals with all aspects of communications in patient care except communication with the physician. b. The nurse is on the same communication level as administration. c. It honors a familiar, structured transfer of information among health professionals. d. It honors an unstructured transfer of information.

: C The SBAR system was developed by professionals in the California Kaiser Permanente System and involves direct, respectful communication skills among professionals with the aim of quality patient care. REF: Page 328 TOP: AONE competency: Communication and Relationship-Building

20. A private ambulance clinic established a few streets away from Pacific Hospital markets its services as quality-based within caring relationships. The clinic is part of the __________ environment. a. Public b. Macro c. Competitive d. Social

: C The competitive environment includes factors and elements that compete for resources with another organization. These elements may include those in public and macro environments. REF: Page 299 TOP: AONE competency: Business Skills

13. At the beginning of its strategic planning processes, Pacific Hospital learns that nursing staff members are uncomfortable with utilizing evidence-based practice, and that staff perceives that only limited resources are available to support translation of evidence into their usual practice. Because this issue has been identified, the management team determines that a plan of action is required to address the issue. Who should be involved at this point? a. Nursing unit managers b. Staff nurses c. All staff d. Administration

: C The first phase in planning is conducting an assessment of the external and internal environments. Assessment of the internal environment should include all levels of staff and facilitates feedback on effectiveness of strengths and weaknesses of systems and processes within the institution. REF: Page 294 TOP: AONE competency: Business Skills

8. While making rounds, a night supervisor finds a unit with a low census and too many staff members. The night supervisor is performing as a statutory supervisor when he or she: a. Assigns nurses to care for specific clients. b. Develops a protocol for unlicensed personnel. c. Recommends transferring a nurse to another service. d. Teaches a nurse to use a new piece of equipment.

: C The night supervisor is acting in accordance with the National Labor Relations Act, which would enable the supervisor to assign nurses to care. REF: Page 349 TOP: AONE competency: Business Skills

8. From your observation of planning activities in the organization, you have noticed that planning and decision making occur at various levels of the organization. The nurse manager has overall responsibility for which of the following? a. Sending out postcards b. Organizing a telephone follow-up c. Monitoring ongoing activities and projects d. Listing the patient population

: C The nurse manager should articulate organizational decision making for the criteria model adopted by the organization and monitor ongoing implementation of the plan. REF: Page 295 TOP: AONE competency: Knowledge of the Health Care Environment

5. A nurse is explaining the pediatric unit's quality improvement (QI) program to a newly employed nurse. Which of the following would the nurse include as the primary purpose of QI programs? a. Evaluation of staff members' performances b. Determination of the appropriateness of standards c. Improvement in patient outcomes d. Preparation for accreditation of the organization by the Joint Commission on Accreditation of Healthcare Organizations (JCAHO)

: C The primary purpose of QI is improvement of patient outcomes, which relates to prevention of error, quality patient care, and patient satisfaction. REF: Page 362 | Page 367 TOP: AONE competency: Knowledge of the Health Care Environment

12. After searching the literature, Tara, the unit manager develops a table that outlines the findings of studies on management of incontinence. She then examines the studies in terms of risk and whether the findings fit within her practice context and for her population of moderately to severely cognitively impaired patients. This is which phase of Stetler's research utilization model? a. Preparatory b. Validation c. Evaluation d. Application

: C The third phase, comparative evaluation and decision making, involves making a decision about the applicability of the studies by synthesizing cumulative findings; evaluating the degree and nature of other criteria, such as risk, feasibility, and readiness of the finding; and actually making a recommendation about using the findings of the studies. REF: Page 386 TOP: AONE competency: Knowledge of the Health Care Environment

8. The oncology clinic manager and the educational coordinator asked nursing staff to complete a brief written survey to assess their attitudes and knowledge related to having used the new infusion equipment for 6 weeks. The stage of change in this situation is: a. Developing awareness. b. Experiencing the change. c. Integrating the change. d. Perceiving awareness.

: C This particular initiative assesses the success with which the change has been integrated into everyday practice after it has been experienced, or the degree to which staff members have accepted using the new infusion equipment. REF: Page 308 TOP: AONE competency: Leadership

2. The home health agency hired an expert in financial management to evaluate and propose a plan for reversing growing expenses and decreasing revenues. The expert is well respected, both personally and professionally, by members living in this small community. To be effective, staff will need to perceive this change agent as: a. Trusted, quiet. b. Flexible, informal. c. Credible, enthusiastic d. Communicative, personable.

: C To influence the decision, the expert must be seen as having knowledge of what matters to the people that they lead and of the change area itself. The expert also must be enthusiastic and communicative and have referent power. REF: Page 311 TOP: AONE competency: Leadership

14. After staff meetings lately, Sharon, the head nurse, observes her staff in small groups, having animated discussions that end abruptly when she approaches. Sharon reflects on this observation and realizes that: a. Two very outspoken members tend to dominate discussions in meetings. b. This behavior is indicative of a high level of communication among her staff. c. Staff members are very committed to the team and have strong opinions. d. Ongoing discussion outside of meetings is conducive to creativity.

: C When team communication is dominated by a few members, leaving others uninvolved or bored, disagreement is not expressed openly. As a result, team members "stuff" their feelings and wait until after meetings to voice their opinions. REF: Page 323 | Page 324 TOP: AONE competency: Communication and Relationship-Building

24. The executive team at Hospital XYZ develops a polished strategic plan that includes a mission statement, goals, and objectives.. The executive team involves the marketing team, so that the plan can be communicated internally and externally. After 2 years, no significant progress has been made toward any of the goals. What has been omitted in the plan? a. Planning b. Marketing c. Integrated financial planning d. Customer focus

: C Without the inclusion of a business plan with the strategic plan and integration of financial planning and allocation of resources execution of the strategic plan can become paralyzed, even with the most effective communication of the strategic plan itself.. REF: Page 293 TOP: AONE competency: Business Skills 1. A clinic nurse developed objectives for a diabetic education program. The characteristics of well-written objectives include that they should be what? (Select all that apply.) a. Achievable b. Understandable and specific c. People-oriented d. Manager-oriented e. Means-oriented f. Measurable ANS: A, B, F The S.M.A.R.T. acronym describes the attributes of objectives: specific, measurable, agreed-upon, reasonable (achievable), and time-bound. REF: Page 295 | Page 296 TOP: AONE ¬¬competency: Business Skills

1. A nurse manager is experiencing conflicts between herself and staff members. She had tried to develop a team by using a shared leadership model to empower the staff. Staff members are functioning: a. As a team. b. Independently. c. Interdependently. d. As a group.

: D A group is a collection of interconnected individuals working together, with a high degree of interdependence, for the same purpose. A team is a unified group that is committed to a common purpose, performance goals, and approach, for which they hold themselves mutually accountable. The conflict indicates that the staff may not be united in a common purpose. REF: Page 322 TOP: AONE competency: Communication and Relationship-Building

9. An example of one strategy to improve participation in the change process by staff fitting the behavioral descriptions of laggards, early majority, late majority, and rejecters is to: a. Encourage teamwork. b. Transfer to a different unit. c. Require attendance at staff meetings. d. Delegate the roles and tasks of change.

: D According to Rogers's work, the individual's decision-making actions pass through five sequential stages. The decision to not accept the new idea may occur at any stage. However, peer change agents and formal change managers can facilitate movement through these stages by encouraging the use of the idea and providing information about its benefits and disadvantages. REF: Pages 309-311 TOP: AONE competency: Leadership

10. The staff development educator developed strategies to help nurse managers actively listen. Guidelines for active listening include which of the following? a. Speed up your internal processes so that you can process more data. b. Realize that the first words of the sender are the most important. c. Be prepared to make an effective judgment of the communication sender. d. Cultivate a desire to learn about the other person.

: D Active listening means suspending judgment about what is about to be said and listening to all that is said (and not just the first or last words). It is motivated by a genuine desire to learn about the other person. REF: Page 335 | Page 336 TOP: AONE competency: Communication and Relationship-Building

14. Marie is a long-term staff nurse on the rehab floor. Her unit manager has been eager to adopt evidence-based recommendations related to family-centered care on the unit. Marie's response has been that she rarely has time to provide care to patients, let alone families, and that there is no good reason to do anything different than what she is already doing. An approach that may gain Marie's support of the idea is to: a. Invite Marie to review the studies for herself. b. Suggest that she does not need to provide family-centered care. c. Avoid discussion of the idea with her until she initiates it. d. Secure the support of her closest colleagues on the unit.

: D As a skeptic, Marie, who is a late majority adopter, needs pressure from colleagues to move her towards support of the recommendations. The translation of research into practice requires that nurse leaders and managers understand group dynamics, individual responses to innovation and change (such as the response of late majority adopters), and the culture of their healthcare organization. REF: Page 392 TOP: AONE competency: Communication and Relationship-Building

1. The staff members in a local Emergency Department are experiencing stress and burnout as the result of excessive overtime. The staff decides to unionize to negotiate for better working conditions. The increase in unionization within health care may be attributed to the: a. Movement from being "blue-collar workers" to being "knowledge workers." b. Excess profits in health care. c. Level of risk that exists for health care. d. Number of people who are involved in health care.

: D As technology replaces unskilled workers, fewer workers are available for trade-union organizing, which has led to declines in union memberships. Nurses represent a large pool of workers who may be available for union organizing in the face of the declining pool available elsewhere. REF: Page 355 TOP: AONE competency: Business Skills

19. Within a multisite healthcare system, the most appropriate strategy for translation of research would be: a. Widespread development of protocols using EBP at unit levels. b. Dissemination of EBP and recommendations to individuals, units, and the organization. c. Development of the skills of individual managers on how to build guidelines based on EBP. d. Establishment of an interdisciplinary center to guide and lead the translation of research findings into practice.

: D At a systems level, the most appropriate approach would be establishing a center that leads in, guides, and promotes EDP across and at various levels. REF: Page 395 TOP: AONE competency: Knowledge of the Health Care Environment

22. The unit manager was addressing nursing students in the lounge area and was discussing team leadership and team effectiveness. She stated, "One can agree to disagree with another team member's perspective even when one doesn't necessarily see that perspective as being the correct one." In being creative, what did she mean? a. Championing one's own opinion. b. Being compassionate c. Being flexible d. Committing to resolution

: D Caregivers must listen to the other person's perspective, listen to the message accurately, identify differences, and creatively seek resolutions. REF: Page 336 TOP: AONE competency: Leadership

15. Elizabeth, an RN with approximately 15 years of service on your unit, walks away from one of the learning sessions on IV care and you overhear her telling a colleague that she thought the session was a waste of time because "the unit has been using heparin for years and there has never been any adverse effects." You follow up with Elizabeth and discover that she is really quite angry about the information sessions because she feels that you are implying that "what she has been doing all these years means that she is incompetent and doesn't care about her patients." Your response to her indicates that: a. Elizabeth will never adopt the change. b. Elizabeth is insecure in her practice. c. Elizabeth requires more information about the practice. d. Change involves emotions.

: D Change, whether proactively initiated at the point of change or imposed from external sources, affects people. Responses to all or part of the change process by individuals and groups may vary from full acceptance and willing participation to outright rejection or even rebellion. It is critical to be able to "read" people and to recognize that communication should involve people's emotions and feelings. REF: Page 309 TOP: AONE competency: Communication and Relationship-Building

15. A method commonly used in Quality Assurance to monitor adherence to established standards is: a. A Pareto chart. b. Brainstorming. c. Patient interviews. d. Chart audit.

: D Chart audits are a common method of addressing process standards. Chart audits over time yield trend charts. REF: Page 375 TOP: AONE competency: Knowledge of the Health Care Environment

11. The nurse manager was upset with the staff nurse and said, "You did not understand what I said." Which element in the communication process was she referring to? a. Feedback between receiver and sender b. A message channel c. A receiver who decodes the message d. A set of barriers that may occur between sender and receiver

: D Problems can occur at any point in communication and result in miscommunication. In this instance, it can be assumed that there was a sender, a receiver, a channel, and feedback. In this scenario, barriers such as distractions, inadequate knowledge, differences in perceptions, and emotions and personality may have resulted in misunderstanding between the manager and the staff nurse. REF: Page 326 | Page 327 TOP: AONE competency: Communication and Relationship-Building

12. As the nurse manager on a rehab unit, you are asked to come to the tub room immediately because two nursing assistants are having a loud disagreement in front of a patient. You ask the nursing assistants to meet you outside and after ensuring that a third nursing assistant is able to care for the patient, you speak with the two nursing assistants. Which of the following would you ask first? a. "How long have you two been working together?" b. "Have you experienced disagreements like this before?" c. "How do you think this patient's perception of her care has been changed?" d. "What happened to bring on this disagreement today?"

: D Conflicts are usually based on attempts to protect a person's self-esteem or to alter perceived inequities in power. When a nurse recognizes upset and reaction, the following steps can be helpful (Sportsman, 2005): Identify the triggering event ("What happened to bring on this disagreement today?"). Discover the historical context for each person. Assess how interdependent each person is on the other. Identify the issues, goals, and resources involved in the situation. REF: Page 328 | Page 329 TOP: AONE competency: Communication and Relationship-Building

17. In the cardiac intensive care unit, there has been simmering discontent about the new nurse manager, who avoids any discussion about her scheduling and practice decisions. The staff have begun to sort into "different camps" depending on how they feel about the manager or the decisions. Which of the following statements MOST accurately describes this situation? a. The tension that has been generated will result in creative solutions. b. Staff will become a cohesive group that takes a stand against the manager. c. The conflict will result in increased dialogue about practice and scheduling options. d. Patient care may suffer because attention and energy is being diverted toward the unit relationships.

: D Destructive conflict polarizes groups, saps group morale, deepens differences in values, and diverts energy from more important activities, such as patient care. Constructive conflict opens up issues of importance, results in solutions to problems, and enables authentic communication. REF: Page 337 TOP: AONE competency: Communication and Relationship-Building

17. After an extensive campaign to ensure that the community surrounding Pacific Hospital has been exposed to the emphasis of excellence in clinical care that occurs through the use of evidence, the marketing team finds that the public perception has changed little. This determination is made possible through: a. Marketing surveys. b. Focus groups. c. Informal feedback. d. Comparison of data against benchmarks.

: D Developing benchmark data at the beginning of a project enables comparison of later data against the benchmark to determine if a marketing plan has met its objectives. REF: Page 299 TOP: AONE competency: Business Skills

21. Which of the following is most accurate regarding evidence-based practice? a. Evidence-based practice replaces continuous quality improvement. b. Evidence-based practice began with medicine and assists in determining which medical models can be applied in nursing practice. c. Effective and efficient care can already be demonstrated, which means that EBP will soon become redundant. d. EBP is generally recognized across disciplines and by policymakers as state-of-the-art clinical practice.

: D EBP is recognized across nursing and other disciplines as reflective of state-of-the-art clinical practice, as it is based on best available evidence. REF: Page 385 TOP: AONE competency: Knowledge of the Health Care Environment

23. Benchmarks in the strategic marketing planning process are useful in: a. Establishing programs. b. Setting targets. c. Rewarding employees. d. Measuring progress.

: D Establishing benchmarks enables measurement of the interim and final achievements of programs. REF: Page 299 TOP: AONE competency: Business Skills

13. Sally (RN) and Melissa (RN) have shared an ongoing conflict since the first day that Melissa worked on the unit. Sally has confided to another colleague that she doesn't even know why the conflict started or what it was about. This is an example of: a. How expectations and objectives need to be made clear in team situations. b. The need to encourage open discussion of disagreements in opinions. c. The importance of involving all staff in discussions in group settings. d. The enduring nature of first impressions.

: D First impressions are lasting and, as Sally indicates to her colleagues, are often an unconscious response. REF: Page 325 TOP: AONE competency: Communication and Relationship-Building

6. To effectively achieve a change goal/outcome in a change situation, the wound care specialist will: a. Preserve the status quo. b. Diminish facilitators and reinforce barriers. c. Weigh the strength of forces. d. Strengthen facilitating forces.

: D For change to be effective, the facilitators must exceed the force of the barriers; thus, strengthening the facilitating forces would achieve this aim. REF: Page 307 TOP: AONE competency: Leadership

22. As the unit manager, you spend a day performing direct patient care and work with a new system that is designed to capture patient documentation at the bedside. During discussions with staff while giving care, you discover that the number of screens that need to be opened during documentation makes charting more complex and time-consuming than traditional manual charting approaches. On the basis of this feedback, you: a. Assume that the system is doing what it needs to do. b. Provide reassurance to staff that the unit has achieved its goals in implementation of the system. c. Ask some of the staff if they have had similar experiences with the system. d. Consult chart audit data and end user consultation reports to determine if errors and problems are occurring.

: D In Kotter's eight-step change model, removing obstacles means keeping alert for barriers in structure and processes that limit the ability to change and then removing those barriers once they have been found. REF: Page 315 TOP: AONE competency: Leadership

7. The nurse gives an inaccurate dose of medication to a patient. After assessment of the patient, the nurse completes an incident report. The nurse notifies the nursing supervisor of the medication error and calls the physician to report the occurrence. The nurse who administered the inaccurate medication understands that: a. The error will result in suspension. b. An incident report is optional for an event that does not result in injury. c. The error will be documented in her personnel file. d. Risk management programs are not designed to assign blame.

: D QM stresses improving the system, and the detection of staff errors is not stressed. If errors occur, reeducation of staff is emphasized rather than imposition of punitive measures such as disciplinary action or blaming. REF: Page 365 | Page 366 TOP: AONE competency: Knowledge of the Health Care Environment

24. As the unit manager, you are interested in determining whether patient autonomy is preserved through informed consent in surgical settings. You determine that participatory action research is the best method to address this question. Which of the following is consistent with a participatory action research design? a. You interview 125 patients who have recently undergone surgery and transcribe the interviews to determine themes. Themes are validated with an expert in informed consent. b. You circulate a questionnaire to patients who recently underwent surgery and ask for their opinions regarding consent. Data are analyzed and the findings distributed to administration and other groups. c. An audit is undertaken of signed consents for treatment, to determine if the consent is properly witnessed and signed. Findings are used to inform changes in policies. d. You meet with a patient group to determine which questions should be asked about patient informed consent and what issues might be encountered and addressed during the research.

: D In participatory action research (PAR), the members of the community being studied are integral members of the research team and are involved in identifying the questions and addressing the issues involved in the implementation of the research project (Chevalier & Buckles, 2013). REF: Page 390 TOP: AONE competency: Knowledge of the Health Care Environment 25. Volunteers in a study are assigned randomly to groups. Some of the volunteers receive an herbal supplement that is reputed to control nausea, and some of the volunteers are assigned to a control group where a placebo is administered. This is an example of a(n): a. Longitudinal study. b. RCT. c. Meta-analysis. d. Appraisal tool. ANS: B RCTs, or randomized controlled trials, always involve testing of a treatment through the random assignment of subjects in the study to an experimental or treatment group or to a control group that receives a placebo. REF: Page 398 TOP: AONE competency: Knowledge of the Health Care Environment MULTIPLE RESPONSE

14. Collective action is effective in: a. Ensuring that needs of nurses are placed ahead of other disciplines. b. Defining nursing as a profession. c. Advising patients of the needs of nurses. d. Amplifying the influence of individuals.

: D Individuals may have limited influence in achieving various purposes such as advancement of quality care or of the profession, whereas collective action helps to define and sustain individuals in achieving the desired purposes. REF: Page 348 TOP: AONE competency: Business Skills

24. Sarah, RN, is one of your most enthusiastic staff members and has been to a workshop on preparing educational materials for patients. On the basis of this workshop, she would like to develop an information Website for patients who are being admitted to the ward. An appropriate response to Sarah's suggestion would be: a. "That is a great suggestion, but we have no resources for such an expensive undertaking right now." b. "Perhaps you can keep that in mind as we redesign our charting system." c. "We have too many seniors as patients, and you know that they don't use technology." d. "There is a great group here that meets to look at technology pilots. Let's see if you can join them and discuss your idea further."

: D Involving Sarah with others who enjoy new ideas and who are able to try out new ideas in pilot projects enables her to remain on the cutting edge and to try out innovative solutions with the least amount of disruption. REF: Page 311 TOP: AONE competency: Leadership

7. In a nurse managers' meeting, strategies for ways to help retain staff are discussed. One strategy for assisting nurses in developing collective action skills is: a. Accepting the practice of "going along to get along." b. Attending as many workshops as practical. c. Spending as much time as possible in clinical settings. d. Taking the opportunity to work with a mentor.

: D Mentoring facilitates development and adoption of positive interaction and other skills that facilitate good decision making. Optimism, trust, and decision making are important in collective action and shared decision making and contribute to job satisfaction and lower turnover in staff. REF: Page 351 | Page 353 TOP: AONE competency: Business Skills

9. Which of the following represents a well-written objective? a. Hospital-acquired infections are reduced, and procedures to reduce infections are implemented. b. To increase staff satisfaction and to decrease burnout c. To increase the health of the community d. To implement evidence-based practice on nursing units, as evidenced by adoption of evidence-based processes, by June 1, 2015

: D Objectives are specific (begin with word to followed by an action word; specify single result to be achieved; and specify a target date for attainment) and measurable (provide level of accomplishment at the end). REF: Page 295 | Page 296 TOP: AONE competency: Business Skills

22. After a year, the staff at Pacific Hospital is frustrated, and implementation of evidence-based practice has gone poorly. In responding to this situation, the strategic planning team at Pacific Hospital: a. Recommends that the mission, objectives, and goals of the hospital be reassessed. b. Revises the goal statements in the strategic plan. c. Consults an external team on preparation of materials on evidence-based practice. d. Holds focus groups with staff to determine what kinds of issues, challenges, and obstacles have occurred in implementation.

: D On a consistent basis, the strategic plan is reviewed at all levels to determine whether the execution of goals, objectives, and activities is on target. As stated, a sense of flexibility regarding the objectives is important to consider, and objectives may change as a result of legislation, budget changes, and change in structure or other environmental factors. Involving staff enables development of creative methods to ensure that necessary changes occur. REF: Page 298 TOP: AONE competency: Business Skills

17. As a nurse manager, you know that the satisfaction of patients is critical in making QI decisions. You propose to circulate a questionnaire to discharged patients, asking about their experiences on your unit. Your supervisor cautions you to also consider other sources of data for decisions because: a. The return rate on patient questionnaires is frequently low. b. Patients are rarely reliable sources about their own hospital experiences. c. Hospital experiences are frequently obscured by pain, analgesics, and other factors affecting awareness. d. Patients are reliable sources about their own experiences but are limited in their ability to gauge clinical competence of staff.

: D Patients are reliable and motivated sources of their own experience but often do not have sufficient knowledge of clinical procedures to provide feedback about clinical competence. REF: Page 366 TOP: AONE competency: Knowledge of the Health Care Environment

20. Resistance is most likely when change: a. Is not well understood. b. Involves many layers in an organization. c. Involves nonprofessional workers. d. Threatens personal security.

: D Resistance and reluctance commonly occur when personal security is threatened and may involve loss of confidence in abilities or loss of job or financial security. REF: Page 309 TOP: AONE competency: Leadership

13. Awareness and use of power have been challenging for nurses in general because of: a. Incidences of punishment by authority figures. b. Too little time in the workplace to collectively develop power strategies. c. Lack of cohesiveness and unity among nurses. d. A tradition of obedience to authority.

: D Rituals and traditions such as the Nightingale Pledge have emphasized the need for the "good nurse" to be obedient to authority. This prevailing attitude has made it difficult for nurses, who typically spend considerable time in the workplace and who have opportunity through their work in teams to develop cohesiveness and unity, to develop awareness and use of power. REF: Page 348 TOP: AONE competency: Knowledge of the Health Care Environment

7. The wound care nurse decided to involve those to be affected by change early in the change management process. This can positively result in: a. Coordination. b. Resistance. c. Anticipation. d. Participation.

: D Successful change means persistence and advancement of the change, which requires the undivided focus of all team members. Early involvement and participation are critical to capturing the undivided focus of team members. REF: Page 316 TOP: AONE competency: Business Skills

24. At Hospital Ajax, there has been a 20% increase in instruments and sponges being left in patients during surgery and surgeries on the wrong limbs. These are known as: a. Sentinel events. b. Medically sensitive events. c. Nurse-sensitive events. d. Never events.

: D The NQF and CMS define never events as errors in medical care that are clearly identifiable, preventable, and serious in their consequences for patients and that indicate a real problem in the safety and credibility of a healthcare facility. Examples of never events include surgery on the wrong body part, foreign body left in a patient after surgery, mismatched blood transfusion, major medication error, severe pressure ulcer acquired in the hospital, and preventable postoperative deaths. REF: Page 377 TOP: AONE competency: Knowledge of the Health Care Environment

14. Elizabeth, an RN with approximately 15 years of service on your unit, walks away from one of the learning sessions on IV care and you overhear her telling a colleague that she thought the session was a waste of time because "the unit has been using heparin for years and there has never been any adverse effects." According to Havelock (1973), this comment may originate from failure in which phase of the six phases of planned change? a. Generating self-renewal b. Choosing the solution c. Diagnosing the problem d. Building a relationship

: D The first phase of this model of planned change involves building a relationship as a basis for later phases, which include diagnosing the problem and choosing the solution. At this point, as a new manager, the relationship may not have yet developed sufficiently with Elizabeth. REF: Page 310 TOP: AONE competency: Leadership

18. In looking at an organizational chart for her institution, Jennifer notes that nursing is led at the senior level by a non-nurse executive. Jennifer expresses concern that this is a Zreflection of how nursing is viewed within the organization. Jennifer's comments reflect: a. A concern that resource allocation will be made on a business and not a professional model. b. The dissatisfaction that occurs when lack of autonomy is given to nurses. c. Concern with the nonadvancement of nursing practice in the institution. d. An awareness of how organizational culture is reflected in organizational structure.

: D The organizational chart reflects the formal structure of the organization and can reflect predominant beliefs, values, and relationships in the organization. Exclusion at senior executive levels of nurse leaders may reflect institutional beliefs about how resources are allocated, the degree of autonomy given to staff, and involvement of key groups in decision making. REF: Page 350 TOP: AONE competency: Knowledge of the Health Care Environment

14. In implementing the strategic plan for effective utilization of evidence to guide nursing practice at Pacific Hospital, the nursing unit managers take the next step, which is the implementation. Implementation most likely would involve: a. Revisiting the mission statement and objectives. b. Reevaluating external and internal factors affecting evidence-based practice. c. Preparing a budget to support implementation. d. Developing unit-based objectives related to the plan.

: D The previous planning (assessment of internal and external environments, development of a mission and plan, and allocation of resources) has already been completed. The next step involves adapting the strategic plan to the nursing units. REF: Page 298 TOP: AONE competency: Business Skills

19. "I really wish that my supervisor would realize and acknowledge all the things I do well." In nursing, this has been identified as a problem. Which statement is part of the solution? Focus on: a. New staff. b. Care assignments with which the individual is not familiar. c. Making corrections. d. The strengths of the individual rather than the weaknesses.

: D The research of Rath (2007) included many recommendations, one of which was that focusing on mediocre behaviors and on a person's weaknesses will not lead to excellence. Focusing on weaknesses tends to decrease the appreciation, and thus the acknowledgements. REF: Page 335 | Page 336 TOP: AONE competency: Communication and Relationship-Building

1. The chief nursing officer at a local hospital seeking Magnet™ status creates staff development classes concerning translation of research into practice (TRIP). What best describes TRIP? a. Conducting an integrative review of the literature b. Searching the literature for a systematic review c. Providing the results of research studies to practitioners d. Applying strategies that aid in adoption of research in practice

: D The science of how research is adopted is known as translation science, the science of translating research into practice (TRIP). The primary aim of research utilization is to activate the change process to move research findings into practice to improve patient outcomes. REF: Page 394 TOP: AONE competency: Leadership

18. Which of the following is an effective approach in the appraisal of research studies? a. Accept only studies that use a RCT design. b. When ranking research studies, choose RCTs over qualitative studies. c. Select only studies with a large sample size. d. Evaluate the quality of the research against the standards for that type of research.

: D While randomized controlled trials (RCTs) are generally considered the gold standard for research, it is important to assess not only the method but the quality of the study and its applicability to the question that is being asked. The quality of all studies should be appraised against the standards accepted for that that type of research. REF: Page 398 TOP: AONE competency: Knowledge of the Health Care Environment

The nurse manager understands that the three Ps associated with client education are philosophy, priority, and performance. Effective client education programs start with a shared philosophy that such programs are worth the investment. Evidence of a philosophic commitment to client teaching is best represented by: a.Investing time and energy in teaching clients. b.Developing teaching skills among the nursing staff. c.Assuming that clients lack the knowledge they need. d.Having a teaching checklist on clients' charts.

A A philosophy that patient education is an investment with a significant positive return is one of the three Ps of a successful consumer education focus. Money invested in teaching is money well spent.

In designing a program for young adults regarding safe sexual practices, which of the following might reach the greatest number in your target group? a.Web-based applications b.Print-based media such as newspapers c.Television advertisements d.Brochures in kiosks in malls

A Mobile technology is changing the digital divide, with young adults, minorities, those with no college experience, and those with lower household incomes being more likely to indicate that phones are their main source of Internet access (Zickuhr & Smith).

Based on data from the patient satisfaction survey, the nurse manager decides that a change should be made in communication with family members. What would be important for a nurse manager to consider when instituting a change to improve customer service? a.Assess the perceptions of the nursing staff regarding the particular service problem. b.Include community representatives on a planning committee to address the change. c.Involve physicians, other healthcare professionals, and ancillary staff. d.Review all patient complaints with the nursing staff.

A Nurses are the healthcare providers who spend the most time with the consumer and are in an opportune position to understand the issues, structures, and processes that affect patients. The nurse acts as the primary person to be alert to circumstances that may prevent a successful outcome for the patient and to intervene on the patient's behalf. As a nurse manager, it is important to support staff in their use of power to be in control and to make decisions at the consumer-staff level of interaction.

As a nurse manager, you see an opportunity for patients to be well serviced through the medical home concept. You recognize that the concept of medical homes: a.Currently does not include nurses in its vision of multifaceted primary care. b.Includes nurses as part of an interdisciplinary and multidisciplinary team. c.Restricts nurses to services related to direct care and procedures. d.Cannot encompass nurses within this framework.

A The concept of medical homes encompasses the idea of multifaceted medical homes that provide a usual source of health care. Current discussions have focused on physician-directed care even though nurses in advanced practice are well suited to lead teams in this model.

The chief nursing officer is pleased with the nurse manager's strategy of improving patient satisfaction in the pediatric intensive care unit. She decides to implement these changes throughout the hospital. What would be important to consider in implementing a new program focused on improving relationships with consumers? a.Recognition of the nursing staff for excellence in promoting consumer relationships b.Holding the staff accountable for resolving patient complaints c.Selecting a staff nurse leader to implement the program d.Identifying key staff members who have already demonstrated excellence in consumer relationships

A The nurse leader should allow professionals more influence over their practice;give staff opportunities to learn new and varied skills; give recognition and reward for success and support and consolation for lack of success; and foster motivation and belief in the importance of each individual and the value of his or her contribution.

As a new manager, you reflect on what professional development would be most valuable to assist you in taking on this role. Which of the following would you most likely identify?

A workshop on conflict management and communication skills.

Which of the following activities would represent a customer-friendly approach in a healthcare setting? (Select all that apply.) a.Using a local anesthetic before inserting a needle into a child's arm b.Repeating patient history information to the admitting clerk, the admitting nurse, and the ultrasound technician c.Ensuring that birthing preferences are on file and available when a laboring mother comes in d.Providing support to families when a family member is brought into trauma

A, C, D A service orientation means delivering services in a manner that is least disruptive. When possible, services should come to the patient and should be as easy, comfortable, pleasant, and effective as possible. Meeting the emotional, psychosocial, and spiritual needs of the patient is important.

The nurse manager is aware that conflict is occurring on her unit; however, she is focused on preparing for a state health department visit, so she ignores the problem. A factor that can increase stress and escalate conflict is: a. The use of avoidance. b. An enhanced nursing workforce. c. Accepting that some conflict is normal. d. Managing the effects of fatigue and error.

ANS: A Avoidance as a conflict-management style prolongs conflict and has a tendency to escalate conflict. REF: Page 436

Staff at Valley Hospital are concerned that recent staffing cuts will affect their ability to provide quality patient care, and they express their concerns to senior management. The CEO of Valley Hospital makes the following statement: "We need to contain costs because our funding has been decreased." This is a good example of which of the following conditions that propel a situation toward conflict? a. Incompatible goals b. Role conflicts c. Structural conflict d. Competition for resources

ANS: A Conflicts arise in four areas: goals, facts, approaches, and values. Conflicts among goals arise from competing priorities such as the provision of quality patient care and containment of costs. REF: Page 433 | Page 436

In the Emergency Department waiting room, you notice a patient sitting, with his head in his hands, who has been waiting for about 5 hours for relief of his headache. When you approach him to ask him how he is doing, he says, "I can't believe that I have to wait this long for help! Do you know what it is like to be in pain for 10 hours?" Your response to him would be: a. "It is frustrating to wait when you are in pain and when you are expecting to receive relief right away." b. "Don't talk to me. If you are going to be rude, then you will not receive treatment here." c. "We are very busy and don't have enough staff to deal with problems such as yours." d. "Perhaps you should go elsewhere. We do not have time for you here, as many more sick patients are waiting."

ANS: A Empathizing helps the other person to know and feel that he has been understood and is powerful in de-escalating a situation that has potential for aggression and violence. REF: Page 479 | Page 480

Becky, RN, works as a staff nurse in mental health; Sharon works as a data entry clerk in Admissions; Sarah is an emergency room physician; and Donna is a housekeeper in geriatrics. Which of these four is most at risk for violence and aggression? a. Becky b. Sarah c. Sharon d. Donna

ANS: A Healthcare workers, especially nurses, experience a disproportionately high rate of violence, compared to personnel in other industries. Nurses are the primary target of violence in healthcare settings, especially those who work in emergency room, mental health, and geriatrics. Hader (2008) found that nurses experienced their colleagues as primary targets of violence 79.7% of the time. REF: Page 465 | Page 468 | Page 469

3. In keeping with guidelines of the organization, the nurse manager documents staff problems. Documentation of disciplinary problems should: a. Include a plan to correct them and to prevent future occurrences. b. State a detailed history of past problems that are related to the current one. c. Be written at the convenience of the manager. d. Accumulate until the evaluation period begins.

ANS: A In documenting staff problems, it is important to specifically indicate what rules were broken or violated, consequences if behavior is not altered, employee's explanation of the incidents, and the plan of action to achieve and to reach new goals.

19. During unit staff meetings, you observe that Marg rolls her eyes and snorts whenever Julia makes a comment. Your first response as a unit manager is to: a. Discuss what you have observed with Marg. b. File immediate documentation in Marg's personnel file. c. Ask Julie to monitor Marg's behavior during meetings. d. Ignore the behavior, as Marg is one of your strongest nurses.

ANS: A Incivility must be addressed. The initial step in addressing it is discussion with Marg, and if the behavior continues, then written documentation should be filed in Marg's personnel file. Monitoring and follow-up are your responsibility as the unit manager.

Sarah, a staff nurse on your unit, witnesses another nurse striking a patient. Sarah wants to remain friends with her colleague and worries that confrontation with her colleague or reporting her colleague will destroy their relationship. Sarah is experiencing which type of conflict? a. Intrapersonal b. Interpersonal c. Organizational d. Professional

ANS: A Intrapersonal conflict occurs within a person when confronted with the need to think or act in a way that seems at odds with that person's sense of self. Questions often arise that create a conflict over priorities, ethical standards, and values. Some issues present a conflict over comfortably maintaining the status quo and taking risks to confront people when needed, which can lead to interpersonal conflict. REF: Page 433

11. Nathan has been on the cardiac unit for 6 months and has found it difficult to adjust to the expectations of his team. Which of the following behaviors would most likely signal that Nathan is intending to resign from his position on the unit? a. Increased absenteeism over the past month b. Increased attempts to discuss his concerns with his colleagues c. Testing of workplace guidelines d. Frequent defensiveness

ANS: A Many employees increase their absenteeism just before submitting their resignation. If the healthcare worker is experiencing some form of role stress, it might be manifested through absenteeism. Role strain may be reflected by (1) withdrawal from interaction; (2) reduced involvement with colleagues and the organization; (3) decreased commitment to the mission and the team; and (4) job dissatisfaction. Testing of workplace guidelines and defensiveness are associated with immaturity

21. Susan, a new graduate, is upset that so many staff have been absent lately from the unit. She declares to you that all absenteeism could be eliminated with proper management. Your response is based on understanding that: a. Not all absenteeism is voluntary. b. High personal control contributes to absenteeism. c. Direct discussions with employees who have high levels of absenteeism are not recommended. d. All absenteeism is related to personal issues and needs.

ANS: A Not all absenteeism is voluntary or preventable, which means that absenteeism can never be fully eliminated. Absenteeism can result from personal issues and needs, work dissatisfaction, and involuntary reasons such as jury duty.

12. All of the following are grounds for immediate dismissal except: a. Failing to pursue further medical help for a patient; patient dies. b. Selling narcotics obtained from the unit supply of narcotics. c. Restraining a patient in bed for 7 hours, unsupervised, as punishment for hitting a staff member. d. Grabbing the unit manager and threatening further physical harm after a poor performance appraisal.

ANS: A Situations that may warrant immediate dismissal include theft, violence in the workplace, and willful abuse of the patient.

You need to terminate Gregory, who has had a long-standing history of conflict with you and the staff, and who recently was charged with theft of patient belongings. You consult Human Resources, and together, you develop a plan, which includes: a. A private meeting with Gregory, a Human Resources representative, and you to deliver the news and deliver the termination notice and all other documents that are related. b. Planning an opportunity for Gregory to return and be recognized at a staff farewell. c. Calling Gregory at home to tell him that he is fired and that his paperwork will be sent to him at a future date. d. Calling him into a meeting in your office on the ward, where assistance is available, should he become upset or agitated.

ANS: A Termination requires careful planning as to timing, privacy, safety, and how to preserve the employee's dignity and avoid humiliation. Choosing a private location where colleagues are not present, and organizing all documentation that is required to be given to Gregory, achieves these goals and prevents his having to come to the organization at a future date. REF: Page 469

At 3 AM, a man walks into your emergency department. He paces back and forth in the waiting area before he approaches staff to ask if he can see his wife, who is a patient on another floor. He speaks rapidly, his face is flushed, he glances around often, and he keeps his hand in his jacket pocket. A best initial response would be to: a. Assess your situation and your surroundings. b. Ask two or three staff to assist in confronting the individual. c. Ask what floor his wife is on and remind him that visiting hours are closed. d. Remain calm as there is no potential for violence here.

ANS: A The behavior of the individual (flushed appearance, furtive glances, speed of speech) and the hand in his pocket suggest the potential for violence or aggression. The first step is to quickly assess your surroundings for others who might assist and for safety alarms. REF: Pages 39-40 | Pages 81-82

After using a mediator to resolve a conflict between the nurse manager and two staff nurses, the chief nursing officer decides to: a. Observe to make sure the conflict has been resolved. b. Fire both staff nurses. c. Reassign both staff nurses. d. Reassign the nurse manager.

ANS: A The nurse leader should follow up to determine if the conflict has been resolved because, in professional practice environments, unresolved conflict among nurses is a significant issue that results in job dissatisfaction, absenteeism, and turnover, as well as in decreased patient satisfaction and poorer quality in patient care. REF: Page 432 | Page 436

A nurse educator is giving a workshop on conflict. During the sessions, he makes various statements regarding conflict. All of the statements are true except: a. Conflict can decrease creativity, thus acting as a deterrent for the development of new ideas. b. Horizontal violence involves those with similar status but little power in the larger context. c. Interprofessional collaboration reduces unresolved conflicts. d. All conflicts involve some level of disagreement.

ANS: A The opposite is true because research has shown that conflict, like change, increases creativity and allows for the development of new ideas. REF: Page 432 | Page 444 | Page 445

You are part of a multidisciplinary team that is charged with designing a workplace safety plan for your healthcare organization. This team has been established in response to increases in reports of violence and aggression. You begin by: a. Surveying staff about levels of satisfaction with the workplace and management, collegial, and patient relations. b. Offering training sessions in self-defense. c. Developing a policy that outlines zero tolerance for bullying. d. Offering education sessions on recognizing behaviors with potential for violence.

ANS: A Violence and aggression and a toxic workplace can lead to staff dissatisfaction and high staff turnover rates. Surveying staff provides a useful starting place in identifying problems such as employee dissatisfaction, bullying, and other forms of violence. REF: Page 468 | Page 471

Joe and Carol, two of the RNs on Unit 22, are discussing recent incidents on the unit that have involved patients and visitors uttering threats or making demeaning remarks to staff during evening hours. Joe observes that unless someone shoots at him, he is not concerned because "words can't hurt you." Joe's remarks: a. Illustrate common misperceptions about the nature of violence. b. Accurately depict the difference between violence and aggression. c. Are partially correct because verbal remarks do not cause injury. d. Reveal possible issues that Joe relates to violence in his personal life.

ANS: A Violence and aggression involve verbal and nonverbal and covert and overt behaviors, and all forms are capable of producing short- and long-term injury that may have an impact on productivity, work performance, work attendance, and patient care. REF: Page 464 | Page 465

24. The unit manager discusses absenteeism with the unit clerk. She indicates that it is a serious problem on the unit. Which of the following points would they have likely discussed? (Select all that apply.) a. Employee morale is at a high level. b. Care will suffer and standards will be lowered. c. Existing staff have experienced little effect from the absenteeism. d. Replacement staff usually needs little supervision.

ANS: B Absenteeism puts a strain on staff, produces morale problems, and can jeopardize patient safety.

In designing a new healthcare facility, it is particularly important to pay close attention to safety elements related to violence and aggression in which of the following settings? (Select all that apply.) a. Emergency b. Psychiatry c. Gerontology d. Maternal-child

ANS: A, B, C Although the potential for violence and aggression exists in all healthcare settings, emergency, psychiatric, and geriatric settings are at particular risk for violence. REF: Page 468

1. In developing curricula that will address needs based on forecasts for the future, nursing educators need to contemplate (select all that apply): a. Prevention strategies. b. Leadership skills and knowledge. c. Violence de-escalation strategies. d. Strategies for job security.

ANS: A, B, C Future forecasts suggest that health factors such as obesity that are implicated in the development of chronic disorders will increase, as will chronic disease. Leadership skills have been identified as a key competence for nurses of tomorrow, and competence with technology will be needed as technology continues to revolutionize health care. Rather than emphasizing job security, nurses will need to be prepared to be in an environment with many options and episodic employment.

1. Clinical incompetence is one of the more serious problems facing a nurse manager. Joyce, the nurse manager, is not aware of the problems of Sarah, a novice nurse. After she investigates, it is obvious that Sarah's peers are covering for her. Which of the following might Joyce include in her meeting with the nurses? (Select all that apply.) a. "It is a nurse's professional responsibility to maintain quality control." b. "All instances of clinical incompetence are to be reported." c. "It is not considered being disloyal when one nurse reports another for poor care." d. "Patient care is the number one concern. Meeting standards is mandatory and necessary."

ANS: A, B, C, D The nurse leader must remind employees that professional responsibility is to maintain quality care, and thus they are obligated to report instances of clinical incompetence, even when it means reporting a co-worker. Ignoring safety violations or poor practice is unprofessional and jeopardizes patient care.

Nurses entering into the workforce today are faced with which of the following relationships that could create organizational conflict? (Select all that apply.) a. Nurse-physician relationship b. Nurse-nurse relationship c. Nurse-patient relationship d. Nurse-chief nursing officer relationship e. Nurse-auxiliary personnel relationships

ANS: A, B, C, D, E By nature, conflict is potentially present in all interpersonal situations. The nurse manager should create an environment that recognizes and values differences in staff, physicians, patients, and communities. REF: Page 442

Caroline asks family members to leave while she cares for the 16-year-old victim of a recent car accident. The father screams at her and tells her that she has no right to ask his family to leave, and that if she continues to do so, he will "throw her out of the room." Caroline is shaken and tells her head nurse, who tells her that this kind of thing is just part of the job. The guidance of the head nurse: a. Is reasonable. No physical violence was involved. b. Is related to why statistics on violence in health care are likely underreported. c. Acknowledges the deep distress and fear of the family. d. Acknowledges the concern of the nurse.

ANS: B A common perception is that incidences such as these, which do not involve physical injury or harm, but rather threats, are part of the job. Because of underreporting, data related to violence and aggression in the workplace may not be reflective of its true incidence. REF: Page 465

Two staff nurses are arguing about whose turn it is to work on the upcoming holiday. In trying to resolve this conflict, the nurse manager understands that interpersonal conflict arises when: a. Risk taking seems to be unavoidable. b. People see events differently. c. Personal and professional priorities do not match. d. The ways in which people should act do not match the ways in which they do act.

ANS: B By definition, conflict involves a difference in perception between two or more individuals. REF: Page 433

John is a circulating nurse in the operating room. He is usually assigned to general surgery, but on this day he is assigned to the orthopedic room. He is unfamiliar with the routines and studies the doctor's preference cards before each patient. The fourth patient comes into the room and John prepares a site for a biopsy using a Betadine solution. The surgeon prefers another solution. He notices what John has done and immediately corrects him by rudely insulting John. Which of the following is the most appropriate approach to conflict resolution in this example? a. Collaboration b. Compromising c. Avoiding d. Withdraw

ANS: B Compromise involves negotiation or an exchange of concessions and supports a balance of power. REF: Page 439

A group of staff nurses is dissatisfied with the new ideas presented by the newly hired nurse manager. The staff wants to keep their old procedures, and they resist the changes. Conflict arises from: a. Group decision-making options. b. Perceptions of incompatibility. c. Increases in group cohesiveness. d. Debates, negotiations, and compromises.

ANS: B Conflict involves disagreement in values or beliefs within oneself or between people that causes harm or has the potential to cause harm. Folger, Poole, and Stutman (2012) add that conflict results from the interaction of interdependent people who perceive incompatibility and the potential for interference. REF: Page 432

7. A nurse manager understands that the second step in handling an employee with a disciplinary problem is to document the incident. Which of the following is best for documentation of personnel problems? a. Use of the performance appraisal on an annual basis b. Notes made immediately after an incident that include a description of the incident, actions taken, plans, and follow-up c. A tally sheet of medication errors and other specific problems that will be used at annual review d. Copies of reports, placed in his or her file, of all unusual occurrences involving the employee

ANS: B In documenting staff problems, it is important to specifically indicate what rules were broken or violated, consequences if behavior is not altered, employee's explanation of the incidents, and the plan of action to achieve and to reach new goals.

Your healthcare organization places a high value on workplace safety and integrates this into all aspects of administrative and patient care processes. As a unit manager, you thoroughly endorse this direction, and during the selection and hiring of new staff, you consistently: a. Refuse to hire applicants who are pushy during interviews. b. Thoroughly follow up with all references before offering a position. c. Ask applicants during the interview if drug or alcohol abuse is a problem. d. Refuse to interview applicants with sporadic work histories.

ANS: B Determining if current employees pose a danger in the workplace is a critical factor that is often overlooked. In addition to personal and psychological factors, behaviors can be observed in employees that may be related to violence or aggression in the workplace (Paludi, Nydegger, & Paludi, 2006). The most obvious indicator is a previous history of aggression and substance abuse. Screening potential employees through drug testing, background checks, and references can help reduce the risk of hiring someone who may pose a danger in the workplace. REF: Page 469 | Page 470

23. The education consultant for the hospital is presenting a workshop on "Documentation: A Manager's Responsibility." Which of the following points would she not include in her PowerPoint presentation? Documentation: a. Cannot be left to memory. A notation must be placed in the personnel file. b. Should avoid discussion of the problem. c. Should include what was done about the problem when it occurred. d. Needs to include date, time, and place.

ANS: B Documentation of personnel problems is one of the most important aspects of the nursing manager's role. Through carefully detailed and timely documentation of the problem and plan, the manager decreases the burdensome problems that can ensue from improper or inadequate documentation.

15. Nurse Stacey is a self-admitted drug addict and has been a heavy abuser of codeine. Stacey and the unit manager decide that changes have to occur. Stacey enrolls in an addiction program, and the manager has her transferred to a drug-free area. What other strategies might be appropriate? a. The manager could refer Stacey to the Human Resources Department. b. The manager could assist in monitoring Stacey's progress. c. The manager could counsel Stacey if Stacey has formed a trusting relationship with her. d. Stacey needs to be asked not to involve her family in the recovery program because this is a work-related situation.

ANS: B Effective management demands that the organization take an active role in helping employees with special needs. Humanistic strategies that counsel and assist employees are cost-effective and necessary

13. The unit manager on 4E is concerned about the performance of Jean, a staff nurse. She is not involved directly with Jean, so she has not been able to determine whether the problem is one of motivation, ability, or both. If Jean lacks ability, which of the following strategies might the head nurse use? a. Dismiss or transfer Jean. b. Document all problem areas and then discuss with Jean. c. Develop appropriate solutions and make recommendations to Human Resources. d. Smooth over the problems if they are minor in nature.

ANS: B For the employee to change and grow, specific corrective measures need to be taken. Consultation with the employee is necessary, and documentation is key to determining the issues.

A patient who has a history of involvement with drugs and weapons comes up to you in the hallway and asks you a question regarding directions in treatment. When you respond, he moves closer in to you and puts both hands up on either side of your neck. No one else is in the hallway. Your best response at this point is to: a. Yell at him to stop. b. Calmly ask the patient to remove his hands. c. Hit the patient in the midsection. d. Use pepper spray.

ANS: B In a potentially violent situation, it is important to look and behave in a calm and confident manner, even if you do not feel calm or confident. The person that you are de-escalating will notice and take his cues from you. REF: Page 478

4. Before terminating an employee, a nurse manger must: a. Be an expert in all legal aspects of termination and discipline practices. b. Know the organization's specific policies for addressing disciplinary problems and termination. c. Function as a counselor for problem employees. d. Do everything to assist and protect the employee by adjusting standards and policies.

ANS: B It is important to know the policies of the organization to address disciplinary issues fairly and equitably, as well as to know the model that is employed to address employee problems.

The chief nursing officer plans a series of staff development workshops for the nurse managers to help them deal with conflicts. The first workshop introduces the four stages of conflict, which are: a. Frustration, competition, negotiation, and action. b. Frustration, conceptualization, action, and outcomes. c. Frustration, cooperation, collaboration, and action outcomes. d. Frustration, conceptualization, negotiation, and action outcomes.

ANS: B Thomas (1992) determined that conflict proceeds through these four stages in this particular order. REF: Page 434

Two nurses on a psychiatric unit come from different backgrounds and have graduated from different universities. They are given a set of new orders from the unit manager. Each nurse displays different emotions in response to the orders. Nurse A indicates that the new orders include too many changes; Nurse B disagrees and verbally indicates why. This step in the process is which of the following in Thomas' Stages of Conflict? a. Frustration b. Conceptualization c. Action d. Outcomes

ANS: B Thomas' Stages of Conflict include conceptualization, which involves different ideas and emphasis on what is important or not or about what should occur. REF: Page 434 | Page 435

Sarah is involved in intervening when a patient attempts to harm herself on the unit. During the interaction, the patient slaps Sarah across the face. As a head nurse, it is important that you: a. Offer Sarah immediate education and training in self-defense. b. Assist with follow-up documentation and offer access to counseling. c. Provide access to a lawyer. d. Encourage Sarah to see the incident as a normal part of care.

ANS: B Training should be conducted on a regular basis and cover a variety of topics, including policies and procedures for reporting, record-keeping, and for obtaining medical care, counseling, workers' compensation, or legal assistance after a violent episode or injury. REF: Page 477

Delaney, one of your staff nurses, confides that Marjorie, another nurse, has been actively telling others that you are incompetent and do not know what you are doing in relation to patient care, and that you lie to the staff about attempts to get more staffing. Delaney says that Marjorie is derogatory towards her in front of others and tells her "that she better shape up." Through telephone calls and conversations during breaks, she is recruiting other staff to her position. Delaney confides that most of the staff find you fair, honest, and knowledgeable. Marjorie's behavior can best be characterized as: a. Political action. b. Bullying. c. Building alliances. d. Disgruntlement.

ANS: B Workplace bullying involves aggressive and destructive behaviors such as running a smear campaign, engaging in put-downs, and excluding team members from socialization opportunities. REF: Page 465 | Page 473

You note that Unit 64 has had a high turnover rate of staff during the past year. In addressing the staff turnover rate, you are: a. Confirming the high correlation between managerial incompetence and violence. b. Demonstrating awareness that workplace violence, if present, has significant costs. c. Aware that staff and manager experiences contribute to high turnover. d. Aware that violence is a rare but present factor in the workplace.

ANS: B Workplace violence and aggression contribute to staff turnover and toxic work environments. Loss of the organizational investment required to train new staff and departure of experienced staff can increase operating costs and reduce the quality of care. REF: Page 468

8. The nurse manager places a staff member on probation because of reports of chemical dependency. The nurse manager should be aware that which of the following statements is true regarding chemical dependency? a. The chemically dependent employee usually hides any changes in behavior. b. When confronted with the issue, the affected employee is usually relieved to have someone to talk to about the problem. c. The chemically impaired nurse affects the entire healthcare organization. d. Hospital policy, state laws, and nurse practice acts address procedures for the chemically dependent employee in the most general terms.

ANS: C A chemically impaired nurse jeopardizes patient care through impaired skills and judgment. She or he also compromises teamwork and continuity as peers attempt to cover deficiencies in work performance for their impaired team member

14. Nurses generally experience difficulty in identifying behaviors and actions that could signal chemical dependency in a co-worker. Which of the following is not a behavioral change that occurs with chemical dependency? a. Personality and behavioral changes b. Job performance changes c. Changes in educational involvement and pursuit d. Absenteeism

ANS: C A manager needs to be alerted when suspicions of chemical dependency are raised by behavioral changes in the employee. These include mood swings, changes in hygiene and appearance, heightened interest in the pain control of patients, frequent changes in shifts, increases in absenteeism, and increases in tardiness.

10. A nurse manager must be familiar with the agency's policies regarding termination. Termination procedures include which of the following? a. Following specific procedures from other organizations b. Having an attorney present at the termination meeting c. Having adequate written documentation to support the action d. Having a friend present during the termination meeting

ANS: C All steps should be followed, including full appropriate detailed documentation and following the procedures of the organization.

Mrs. Hill, aged 68, was hospitalized after a stroke. The speech therapist recommended that oral feeding be stopped because of her dysplasia. During visiting hours, Mr. Hill fed his wife some noodles. The nurse noticed this and stopped Mr. Hill from feeding his wife, telling him it was the doctor's decision. An hour later, the nurse returned and found Mr. Hill feeding his wife again. The nurse tried to stop him again. Mr. Hill refused and claimed that the clinical staff was trying to starve his wife; he also threatened to get violent with the nurse. The nurse decided to walk away and documented the event in Mrs. Hill's chart. According to Thomas' Four Stages of Conflict, in which stage could the nurse have been more effective? a. Frustration b. Conceptualizing c. Action d. Outcomes

ANS: C By walking away, the nurse is engaged in an action or a behavioral response, which is the action stage of conflict that is outlined in the four stages of conflict (Thomas, 1992). In this stage, the nurse might have used more effective strategies, such as clarifying Mr. Hill's views on feeding his wife and engaging in dialogue with Mr. Hill to clarify his concerns and attempt to reach a common goal. REF: Page 435

You note that Unit 64 has had a high turnover rate of staff during the past year. In selecting the appropriate action, it is important that: a. All documentation is reviewed. b. Usual processes for discipline are followed. c. Confidentiality is assured. d. An incident report is filed.

ANS: C Confidentiality is important if an employee fears intimidation or retribution from a manager. REF: Page 471

A nursing instructor is teaching a class on conflict and conflict resolution. She relates to the class that conflict in an organization is important, and that an optimal level of conflict will generate: a. Creativity, a problem-solving atmosphere, a weak team spirit, and motivation of its workers. b. Creativity, a staid atmosphere, a weak team spirit, and motivation of its workers. c. Creativity, a problem-solving atmosphere, a strong team spirit, and motivation for its workers. d. A bureaucratic atmosphere, a strong team spirit, and motivation for its workers.

ANS: C Differences in ideas, perceptions, and approaches, when managed well, can lead to creative solutions and deepened human relationships. Work on conflict suggests that complete resolution of conflict is counterproductive to the achievement of organizational goals, organizational change, and cohesiveness of employees. REF: Page 432

1. The most important approach that a nurse manager can take with an emotionally troubled employee is to: a. Act as a therapist for the employee. b. Adjust the standard of care to assist the employee. c. Assist the employee in obtaining professional help. d. Adjust the employee's work schedule to decrease stress.

ANS: C Emotional difficulties are usually beyond the scope of skills that a nurse manager would normally employ. A referral needs to be made to a professional who is specifically prepared to deal with this kind of difficulty.

As a nurse manager, you realize that your unit has become a toxic environment in which horizontal violence and incivility has become common. In addressing the problem, you decide to implement which of the following? a. Increased education in clinical skills b. Follow-up of all reports of violence c. Training in conflict resolution and team-building d. Posting of the institutional policy on violence at the nursing station

ANS: C Encouragement to report violence in all its forms is crucial to understanding the root of the problem and implementing plans to eradicate it. Acts of good faith by organizational management in supporting staff include a policy of non-retaliation for reporting. Making sure that reporting is easier and doing an impartial investigation are critical. Addressing a toxic culture also requires training in conflict management, leadership, communication, and team-building. REF: Page 471 | Page 473

You note that Unit 64 has had a high turnover rate of staff during the past year. In investigating this situation, an important source of data might include: a. Employee evaluations. b. Level of experience of staff. c. Exit interviews with staff. d. Selection processes and decisions.

ANS: C Exit interviews may assist in identifying issues such as workplace violence, bullying, and intimidation by managers. REF: Page 471 | Page 472

During coffee and other breaks, Rosalie, the new RN, is shut out of conversations with the other staff. When she approaches other staff on the unit to ask questions, they turn and walk off in the other direction. The behavior of the staff is characteristic of: a. Dislike. b. Lack of trust in Rosalie's abilities. c. Horizontal violence. d. Cultural incompetence.

ANS: C Horizontal or lateral violence and bullying are terms used to describe destructive behaviors towards co-workers, such as the "silent treatment" and shutting others out of socializing. REF: Page 465 | Page 473

While working with an aggressive patient, it is important for the nurse to: a. Speak firmly. b. Call the individual by name. c. Place herself between the patient and the door. d. Ignore threats against her.

ANS: C In situations where a patient may become aggressive, it is important to ensure that you are not trapped in the room. REF: Page 479 | Page 480

Jane has transferred from the ICU to the CCU. She is very set in the way she makes assignments and encourages her new peers to adopt this method without sharing the rationale for why it is better. This is a good example of a process and procedure that creates which type of conflict? a. Organizational b. Intrapersonal c. Interpersonal d. Disruptive

ANS: C Interpersonal conflict transpires between and among nurses, physicians, members of other departments, and patients. REF: Page 433

In which of the following situations would you, as the head nurse, be concerned about potential safety issues? a. Jordan comes to your office to complain about inadequate staffing on the unit. He says that he is concerned because he attributes a recent incident to the staffing levels. b. Henry, a long-standing RN on the unit, has begun to miss work regularly. He calls in but is vague about his reasons for the absences. c. Carla, RN, has just ended an abusive relationship with Jake, RN, and he will not leave her alone. You are meeting with Jake today because colleagues on nights have reported that Jake seems to have been intoxicated last night and the previous night. d. Sarah is very quiet and says almost nothing in team meetings. Lately, she has been much more animated since becoming friendly with a couple of other RNs on the unit.

ANS: C Jake seems at most risk for violence because of his alcohol use and history of aggression. In the other situations, Jordan is expressing a legitimate concern and is behaving assertively; Henry may have health concerns or other issues that are private and interfering with his work life; and Sarah's change in behavior is likely related to a higher level of comfort with work and colleagues. REF: Page 470

Lee, the head nurse in ER, has attempted to meet Jillian, one of her staff RNs, for several days to discuss concerns about Jillian's relationships with her team members. Lee hopes to offer Jillian coaching so that Jillian's relationships can be more satisfying for Jillian and her team members. Each time Lee and Jillian set a time to meet, Jillian phones in sick. In this situation, Lee and Jillian are demonstrating: a. Similar conflict management strategies. b. Escalation of conflict. c. Avoidance and compromise strategies. d. Competing and compromise strategies.

ANS: C Jillian is demonstrating avoidance by staying away from meetings to discuss her team relationships, and Lee is demonstrating compromise by offering coaching in return for Jillian's being able to engage in more satisfying relationships. REF: Page 436

Kala, a unit manager, in discussing a role the CEO would like her to perform, makes the following statement, "I will sit on the hospital taskforce on improving morale if you send me to the hospital's leadership training classes next week, so I can further develop my skills and thus be more effective." Which of the following conflict management styles is Kala using? a. Collaborating b. Avoiding c. Negotiating d. Accommodating

ANS: C Negotiation involves an exchange of concessions (membership on a committee in return for attendance at a workshop) or trading. This strategy supports a balance of power. REF: Page 439

17. Arrange the strategies from Question 16 in the order in which they should occur in progressive discipline. a. A, B, C, D b. B, A, C, D c. C, B, A, D d. C, A, B, D

ANS: C Studies have shown that following this sequence provides a fair and effective plan for discipline and remediation.

Jenny tells you that she is always able to tell when others are about to become violent because they yell. Your response to Jenny is based on your understanding that: a. Her perception is accurate. b. Yelling is more likely associated with aggression. c. Violence is signaled by a variety of behaviors. d. She is mostly accurate in her thinking.

ANS: C The STAMP Assessment Components and Cues outline a wide variety of verbal and nonverbal cues that might signal the potential for violence. REF: Page 476 | Page 478

5. A nurse manager understands that the typical first step in handling an employee with a disciplinary problem is a: a. Verbal reprimand. b. Written reprimand. c. Reminder of employment standards. d. Day off without pay.

ANS: C The progressive model of discipline advocates that the first step of the disciplinary process is the informal reprimand or verbal admonishment. The nonpunitive discipline model advocates reminding the employee of the employment policies and procedures of the agency.

A safety and security plan is important to a healthcare organization because it: a. Lays out preventive measures in relation to violence. b. Provides direction as to changes in facilities that protect staff. c. Establishes expectations in relation to behavior and tolerance of violence. d. Establishes policies and practices that guide prevention of violence and expectations in the workplace.

ANS: D A safety plan provides overall direction in relation to what is expected, how violence is prevented, and what will occur when violence happens. REF: Page 471 | Page 472

20. Ellen is a novice nurse on your unit. Even though she has come to you highly recommended, as her supervisor, you have noticed some knowledge and skill deficiencies. These deficiencies have been noticed by her peers as well. Which of the following is likely to be the greatest asset to Ellen in improving her performance? a. Giving her a book on nursing related to your area b. Having her spend time with the hospital's manager of education c. Sending her to a conference d. Making arrangements for practice time for her in the hospital's skills lab

ANS: D Active learning and engagement in learning, such as lab practice, are useful in assisting Ellen to improve her skills and knowledge

22. The nursing director calls a meeting with one of the new unit managers. She is very concerned about a report of substance abuse on the manager's unit, and she reviews the procedures involved in dealing with chemically dependent staff. Which of the following statements would not be included in the discussion? "As a manager, you: a. Need to be aware of ADA issues." b. Should check with Human Resources regarding chemically dependent employees and employment practices." c. Check the nurse practice acts for the state in which the nurse resides." d. Should realize that the nurse is a professional embarrassment and should be kept out of sight of other staff."

ANS: D As a manager, you need to be familiar with state and professional regulatory and reporting requirements regarding chemical use and abuse, as well with Human Resource practices and guidelines

Which of the following best exemplifies the predominant style of conflict management for staff nurses? a. Sarah and Jonas, two RNs, disagree about the best approach to assisting a family that has complex needs. They decide that they will consult with family and together will decide what is best. b. Jennifer needs to switch a shift to attend a family function. She arranges to trade with Nancy, who wants a day off next to a 3-day break. c. Lindsay asks Melody to stay late for the third day in a row. Melody refuses, stating that she has already helped out for two days by staying late for Lindsay. d. Lara asks Stacey to switch shifts with her because Lara wants to attend a concert. Stacey would prefer not to but does to enable Lara, who is new in town, to be with her friends.

ANS: D Avoidance and accommodation are the predominant conflict management styles of nurses. Accommodating involves neglecting one's own needs while trying to satisfy the needs of another. REF: Pages 438-440

18. When progressive discipline is used, the steps are followed progressively only for repeated infractions of the same rule. On some occasions, rules that are broken are so serious that the employee is: a. Transferred to another unit. b. Suspended indefinitely. c. Asked to attend a union grievance meeting. d. Terminated after the first infraction.

ANS: D Behaviors that include violence, theft, and purposeful abuse of a client are sufficiently serious to warrant immediate dismissal with the first incident.

The head nurse and a staff nurse are having a conflict over how to use and apply a new procedure for dressings in the medical/surgical unit. The staff nurse wishes to use the new procedure based on newly released nursing research. The head nurse wishes to use a protocol that has been used in the department for a number of years. The head nurse later makes comments to other staff on her unit about the credibility of the staff nurse. This behavior is associated with: a. Lateral violence. b. Horizontal violence. c. Confrontation. d. Bullying.

ANS: D Bullying involves aggressive or destructive behavior or psychological harassment of a recipient who is in a position of power differential with the perpetrator (the head nurse). Bullying is closely related to lateral or horizontal violence and involves such behaviors as incivility or intimidation. REF: Page 439 | Page 444

Jill is the head nurse on a unit in a large hospital. Two of the staff nurses are constantly arguing and blaming each other, and a resolution has not occurred in months. To solve the existing conflict, which is the most creative conflict resolution? a. Avoiding b. Competing c. Compromising d. Collaborating

ANS: D Collaboration, although time-consuming, is the most creative stance. The collaboration technique involves both sides in the conflict working together to develop an optimal outcome. This results in a win-win solution. REF: Page 439

Which of the following exemplifies the predominant conflict management style of nurse managers? a. Elizabeth, the head nurse on neurology, finds that Tom, the RN nurse on nights, is irritable in relation to any suggestions or new ideas, and so she comes in to work after Tom leaves the unit. b. The technology committee has recommended a clinical system for implementation on the nursing unit. Staff is anxious about the change. Tim, the head nurse, asks staff for ideas on how to meet the technology goals and to meet staff needs. c. During management meetings, George, the head nurse on nephrology, dominates meetings and decisions. Lee, the head nurse on the cardiac step-down unit, begins to miss the management meetings. d. Ann, RN, asks her head nurse if she can go on the permanent evening shift. The head nurse, Rajib, agrees, as long as Ann agrees to be involved in assisting to mentor evening staff in the use of the new clinical information system.

ANS: D Compromise involves trading and negotiation and is the predominant conflict management style of managers. REF: Page 439 | Page 440

Factors that influence the ease with which conflict is resolved include all except which of the following? a. Level of interdependence of the parties b. Interprofessional collaboration. c. Expression of one's own needs and ideas. d. Avoidance of the issue or concern.

ANS: D Conflict involves a level of interdependence and is a condition for conflict but not necessarily for continuance of the conflict. Expression of one's ideas and concerns is considered assertive and effective in resolving conflict if the concerns and needs of the other are also considered. Interprofessional collaboration has been shown to be effective in resolving conflict. Avoidance tends to prolong and sometimes escalate conflict. REF: Page 432 | Page 436 | Page 438

16. Incivility is a disruptive behavior or communication that creates a negative environment and interferes with quality patient care and safety. The manager can implement steps that help to alleviate uncivil behavior on a unit. Which of the following would not be an appropriate first step? a. Suspending the staff member from work b. Providing written admonishment that is discussed and placed in the employee's file c. Providing verbal admonishment d. Terminating the staff member

ANS: D Dismissal does not enable the present organization to attempt remediation of the behavior and is not consistent with first steps in progressive discipline.

Linda, a staff nurse on nights, yells at Ali, another RN, and tells Ali that she is stupid and can't get anything right. In responding to this situation as head nurse, it is critical that you: a. Require that Linda attend anger management classes. b. Investigate to see if Ali did anything to aggravate Linda. c. Call both immediately into the office to discuss the situation. d. Respond to Linda in a way that is consistent with organizational processes and with similar situations.

ANS: D Erratic or arbitrary discipline, favoritism, or behavior that undermines the dignity of either individual undermines efforts at curbing workplace violence. Disciplinary actions must be proportionate, consistent, reasonable, and fair. REF: Page 475

The nurse manager decides to use a mediator to help resolve the staff's conflict. A basic strategy for truly addressing this conflict is to: a. Identify the conflicting facts. b. Be determined to resolve the conflict. c. Schedule a meeting time for resolution. d. Have a clear understanding of the differences between the parties in conflict.

ANS: D It is important for each person in the conflict to clarify the conflict as "I see it" and how "it makes me respond" before all the persons involved in the conflict can define the conflict, develop a shared conceptualization, and resolve their differences. REF: Page 435

Which of the following is NOT a factor in patient- and/or family-generated violence in healthcare settings? a. Feelings of vulnerability b. Anxiety about treatments or diagnoses c. Feelings of powerlessness or loss of control d. Staff rudeness

ANS: D Unlike in other settings, hospital violence differs in that it is usually the result of patients or their family members feeling frustration or anger. This is usually related to feelings of vulnerability, stress, and loss of control that accompany illness. Other factors such as the location, size of the facility, and type of care provided also increase the risk for violence. REF: Page 468 | Page 469

In trying to achieve Magnet™ status, the chief nursing officer establishes a shared governance model to help nurses experience job satisfaction. However, some nurses who have enjoyed working with less autonomy resist this change and begin to criticize and make rude comments about managers who embrace this model, as well as colleagues who support it. The comments are largely ignored because those who are making them are well established nurses who are often vocal about their displeasure with the organization. Organizational conflict is arising from which of the following? a. Staffing practices b. Increased participation in decision making c. Allocation of resources d. Tolerance of incivility

ANS: D Organizational conflict arises from discord related to policies and procedures (such as staffing policies and practices and allocation of resources), personnel codes or conduct or accepted norms of behavior (such as incivility), and patterns of communication. A major source conflict in organizations stems from strategies that promote more participation and autonomy of staff nurses. REF: Page 433 | Page 434

2. The nurse manager knows that the most serious effect that absenteeism has on the nursing unit is that: a. Using replacement personnel with new ideas may be beneficial. b. Salary costs are lower because personnel are fewer, and outcome is favorable. c. Absence on the part of the rest of the staff is decreased. d. Unacceptable patient care may result.

ANS: D Reduced staffing adversely affects patient care. Employee morale suffers, care standards may be lowered, and additional stress is placed on working staff

Sarah is a nurse manager in a surgical unit. She is concerned about a conflict between Lucy (a staff nurse) and one of the maintenance personnel. Sarah explains to Lucy that unsatisfactory resolution of the conflict is typically destructive and will result in: a. Decreased frustration between the maintenance worker and her. b. A good relationship with the maintenance department. c. Eventual resolution of the problem without further intervention. d. Decreased productivity on her part.

ANS: D Research by Saltman et al. (2006) determined that productivity decreases with destructive conflict, whereas constructive conflict strengthens relationships. REF: Page 435

Mrs. Hill, aged 68, was hospitalized after a stroke. The speech therapist recommended that oral feeding be stopped because of her dysplasia. During visiting hours, Mr. Hill fed his wife some noodles. The nurse noticed this and stopped Mr. Hill from feeding his wife, telling him it was the doctor's decision. An hour later, the nurse returned and found Mr. Hill feeding his wife again. The nurse tried to stop him again. Mr. Hill refused and claimed that the clinical staff was trying to starve his wife; he also threatened to get violent with the nurse. The nurse decided to walk away and documented the event in Mrs. Hill's chart. The outcome as depicted by Thomas' conflict stages can be considered to be: a. Compromising. b. Confronting. c. Constructive. d. Destructive.

ANS: D Resolution was absent because the nurse did not have time to effectively deal with the issues in the conflict. This can lead to negativity, increased frustration, and further distancing between individuals or groups, including between patients and nurses. REF: Page 439

Which of the following healthcare employees is MOST at risk for violence? a. Becky, working in a well-lit area with stable psychiatric patients and other staff members nearby. b. Sarah, who works in a busy emergency room. Access to the emergency patient units is allowed by security staff and alarm systems are in place in patient units. c. Sharon, who works evening shifts. Workstation is behind a shatterproof glass, and an alarm can be reached easily from her computer. d. Donna, who works the evening shift, cleans rooms each night in the administrative wing and business offices, which are largely empty. The wing is near an outside access door.

ANS: D Staff who work in isolated, poorly lit areas are at higher risk of violence, which would describe Donna's work conditions. In addition, if the business office contains money, this could make it a desirable target for criminals. While Becky and Sarah work in high-risk areas (mental health and the emergency room), the nearby presence of colleagues, the availability of alarms, and good lighting help to reduce the risk. REF: Page 468

9. The chief nursing officer reviews the policy about "progressive discipline process." The progressive discipline process includes which of the following? The manager: a. Is a counselor and friend to the employee. b. Should reprimand and suspend the employee as a last resort. c. Should rehire the employee after a reasonable length of time. d. Should terminate the employee if the problem persists.

ANS: D Termination can be defined as the discharge of an employee who is performing at an unsatisfactory level or who is not a good match for the organization. Termination is always considered to be the last resort when dealing with poor performance.

6. The chief nursing officer understands that clinical incompetence is best prevented by a: a. Flexible protocol for evaluating competency skills. b. Standardized clinical skills checklist. c. Newly established peer review process. d. Formalized competency program with established standards for practice.

ANS: D The competency program with established standards of practice outlines what the nurse must do to achieve desired competencies in her current position. Competency assessment and goal-setting should help the nurse identify how to excel and which competencies the nurse wants to achieve in the future.

As a senior manager, you notice that there have been several resignations on a unit where a new charge nurse has been hired. You suspect that the new charge nurse may be demonstrating bullying behaviors, but staff say little about their relationship with the charge nurse. Your decisions about intervention would be based on which assumptions? a. The staff nurses would tell you if the charge nurse was engaging in relational violence. b. It would be unusual for leaders to engage in violence and bullying behavior. c. Bullying is primarily related to feelings of marginalization and jealousy among peers. d. Initiating confidential exit interviews will assist in determining if leader violence or bullying is occurring.

ANS: D To understand if violence or intimidation is a reason for leaving, organizations should conduct exit interviews with the assurance that the information will remain confidential if an employee fears retaliation. This is an important step in gauging if the problem is bullying or intimidation by managers. Johnson (2009) found that 50% of respondents indicated that they were bullied by their manager or director. The researcher suggested that when management is part of the problem, victims have a harder time feeling they have adequate support to end the negative cycle of violence. Lack of support leads many victims of bullying to decide that the best alternative is to leave the organization and to give this advice to others who find themselves in similar situations (Johnson, 2009). REF: Page 471 | Page 472

Residents in a new long-term care facility attend a large dining hall for meals. In reviewing reports of aggression and violence, you note that behaviors such as hitting, or attempting, to hit staff are increasing. Further investigation suggests that this behavior occurs most often at mealtimes. A possible intervention would be to: a. Seat residents with the highest potential for violence next to those with the lowest potential for aggression. b. Feed residents earlier in the day. c. Restrain residents who are violent or aggressive during mealtimes. d. Establish a smaller dining area that is away from the main area that is for residents who have potential for aggression/violence.

ANS: D Violence and aggression are more likely during times of increased activity, such as mealtimes. Reducing activity levels through interventions such as a separate dining area may reduce incidents of violence and aggression. REF: Pages 468-469

Sarah is a nursing graduate of 5 years who is very confident in her clinical skills. She has taken some certificate courses in leadership and management and has considered beginning a graduate degree with this focus. She is excited about being able to use her knowledge and interest by being hired as a nurse manager. Before beginning her new position, Sarah spends time with her nurse executive to clarify the executive's expectations of her and of the unit that she has been hired to manage. Sarah's actions are important in avoiding role:

Ambiguity.

The nurse manager of a unit was demoted to staff nurse 6 months ago. Because of being near retirement, the former nurse manager wanted to be employed at the hospital and was offered a position on the same unit. The former nurse manager complains often about how infrequently the current nurse manager is available on the unit and argues with physicians and co-workers. Clients have complained about the attitude of the nurse. The behavior of the former nurse manager can be best explained as being caused by:

Anger as a stage of the grieving process.

In addition to providing coaching, a nurse mentor may provide counseling to the mentee. For counseling to be successful, the mentor must:

Assure confidentiality.

Technology is integral to a cardiac ICU. Sue, the nurse manager, implements a patient-centered approach that focuses on the meaning of the experience for the patient and family, primary nursing, and a health literacy approach. Sue is: a.Using high-tech-low-touch approaches. b.Using high-tech-high-touch approaches. c.Providing products. d.Providing tangible products of satisfaction.

B The provision of humanistic care within a high-tech environment is characteristic of high tech-high touch approaches and reflects the idea that the more that high-tech is used, the more patients also desire high-touch.

The nurse manager has to develop a patient satisfaction survey. What is one of the critical elements in selecting a patient satisfaction instrument? a.Being able to use the same instrument for all clinical units b.Including items that are important from the patient's perspective c.Being able to administer the instrument before a patient's discharge from the hospital d.Being sure that the reading level is no higher than third grade

B Because satisfaction is a measure of service and service is a measure of perception of what matters to the patient, to measure satisfaction, surveys must include items that reflect the perspective of the patient. The quality of human contacts becomes the measure by which the consumer forms perceptions and judgments about nursing and the health agency. Consumers may not be able to evaluate the quality of interventions, but they always can evaluate the quality of the relationship with the person delivering the service.

In orienting new staff nurses to a pediatric intensive care unit, the nurse manager asks the staff nurses to answer the following question: "What is an important consideration in providing information to parents of a critically ill child?" a.Making sure that they receive complete information during each encounter with a member of the nursing staff b.Assessing parents' preferences for the amount of information desired c.Allowing parents to observe key aspects of their child's nursing care d.Making sure that patient education brochures explaining ICU protocols are readily available

B Consultation with the parents regarding the amount of information that they desire reflects a service orientation, in which preferences and needs of the consumer are placed first. The other answers reflect nurse-directed decisions in which the nurse decides what information and how much information is needed and how it is to be delivered.

The nurse manager analyzes the data from the patient satisfaction surveys. What can a nurse manager do to strengthen service recovery and improve consumer relationships? a.Post comparisons of patient satisfaction scores with those of other units on a monthly basis. b.Involve the staff in resolving consumer issues quickly and effectively. c.Ensure that staff members apologize to patients when they complain about services. d.Ask that patients with complaints about services place them in a written format.

B Consumers need to be treated with fairness, given explanation, and provided with information about how errors will be prevented in the future. Staff can be assisted to respond to patient concerns through scripting, support, and an atmosphere that places an emphasis on learning and solutions rather than on blaming.

A 27-year old woman is admitted to your ICU in a coma, following an accident. The family of the patient, who is a Native American, places a medicine pouch in the bed with the young woman. As the nurse in this situation, it is important to: a.Explain to the family that the medicine pouch may contain herbs that may bother other patients. b.Ask the family about the significance of the medicine pouch for them. c.Remove the pouch when the family is not present. d.Put the medicine pouch on the shelf beside the bed.

B Diversity encompasses more than differences in nationality or ethnicity and may include a variety of ways that patients are different from their healthcare providers. Nurses need to recognize the culture of their work setting, realizing that it may differ markedly from the culture of the consumer, and move beyond ethnocentrism to provide culturally competent care. This competence includes cultural knowledge, which involves actively learning about a community; cultural sensitivity, which entails valuing and respecting beliefs, norms, and practices of the people being served; and collaboration within a community (Flaskerud, 2007). In this instance, it is important to understand the meaning of the pouch for the family; removal from the bed without discussion does not demonstrate respect for the values and beliefs of this family.

As a head nurse, you are concerned about the service orientation on your unit. Which of the following findings and approaches might provide useful information for you? a.Patients find nurses friendly and accessible; data are aggregated for the institution as a whole. b.Data indicate that nurses are responsive to requests for assistance; data are available for the unit and the institution. c.Specific questions related to management of comfort are included for the institution as a whole. d.The survey asks for a range of responses for the unit and the organization, with a focus on facilities, such as cleanliness and responsiveness of administrative services.

B The National Database of Nursing Quality Indicators (NDNQI®) is a national repository for unit-based quality data that can be used by organizations to benchmark the outcomes of care against those of other institutions (ANA, n.d.). Unit-based quality indicators, including satisfaction with nursing care, are a key feature of the NDNQI®. In addition to hospitals being provided with their own and comparison data, researchers are able to access de-identified data in order to answer important questions about nursing care quality. Pain management, the discharge process, and post-discharge patient callbacks are specific areas where nurses can make significant improvement in patient satisfaction. Because patients and nurses may differ in what they see as factors that produce satisfaction, total reliance on nursing perceptions of patient satisfaction may not provide a complete picture.

John is an older adult patient who comes regularly to the multigroup practice in which you are a nurse practitioner. He says that he doesn't understand what he is supposed to be doing about his medications, because every time he comes to the clinic, he sees someone else who has different ideas. John's experience represents what aspect of the current consumer experience? a.Nurses are well-trusted members of the healthcare team. b.Fragmentation of care results in lack of respect and trust. c.Care providers often have conflicting ideas about care. d.The public does not trust care providers other than nurses.

B When consumers visit a multigroup practice, they do not have the option of selecting a specific healthcare provider, and thus, there is less opportunity to build a trusting relationship with a provider.

During review of a patient's progress, the healthcare team determines that a patient requires treatment that is generally accepted at that time in the usual illness trajectory of a patient. The patient is unable to pay. As the head nurse, you persist in ensuring that this patient receives the treatment. You are: a.Empowering the patient. b.Avoiding litigation. c.Advocating for the patient. d.Supporting the clinical pathway.

C Advocacy means defending the rights and interests of others and, in this situation, the right of a patient to receive care, as determined by standards utilized in a critical pathway.

Which of the following actions best exemplifies advocacy? a.Developing a list of agencies that will provide free services for the homeless b.Working in a needle exchange program for individuals in an inner-city environment c.Acting on behalf of a patient to promote end-of-life wishes to an ethics committee d.Working in a free clinic for immigrant workers

C Advocacy means making known and defending and protecting the rights and interests of others, as well as ensuring the dignity and respect due to others. Simply being employed in an environment where this might be a focus of practice does not necessarily ensure that advocacy is actually occurring.

As part of performance appraisal, the nurse manager designs strategies to acknowledge staff members. What practices by the nurse manager best acknowledge staff accountability and contribution? a.Providing new and varied learning experiences for staff members b.Fostering group cohesiveness through standardization of unit activities c.Allowing professionals greater influence over their practices d.Giving recognition for success and support for failure to staff members

C Facilitating greater control over practices implies trust and acknowledges expertise and performance.

The complexity of the healthcare environment for consumers is increased by: a.Falling levels of education among the public. b.Increased levels of poverty. c.Complex compensatory systems and a variety of delivery systems. d.Increased numbers of uninsured or underinsured.

C The complexity of options in health care and the processes and policies involved in funding health services for patients, as well as fragmented relationships with a growing variety of healthcare providers, contribute to the complexity of the system for patients, especially for those who might need it most, such as the poor, uninsured or underinsured, and homeless.

In writing the patient satisfaction survey, the nurse manager is aware of the education levels of the families. What is the most critical element in the concept of health literacy? a.Providing instructional materials at appropriate reading levels b.Facilitating access to translators for persons with language barriers c.Obtaining, processing, and understanding basic health information so appropriate decisions can be made d.Knowing that most people have limited health literacy skills

C The definition of health literacy used by the federal government is "the degree to which individuals have the capacity to obtain, process, and understand basic health information and services needed to make appropriate health decisions" (U.S. Department of Health and Human Services, 2010).

A nurse is admitted to a psychiatric unit. The staff expresses frustration with her because they have explained several times her medication regimen, and yet, when she goes on passes, she fails to follow it. The staff believe that, as a nurse, she should be able to understand what is expected. The nurse's failure to follow the regimen indicates: a.Early cognitive impairment. b.Lack of motivation. c.Lack of health literacy. d.Worsening health state.

C What is evident from the response is that she lacks health literacy or the capacity to obtain, process, and understand basic health information and services. Using a Health Literate Care Model involves weaving health literacy strategies into care by assuming that patients do not understand their health conditions or what to do about them, and then, subsequently assessing patients' understanding (Koh, Brach, Harris, & Parchman, 2013). For example, a nurse who is an expert clinician in a specialty practice area, when diagnosed with a serious chronic illness, may not have the appropriate background to make informed healthcare decisions.

Who of the following might be the BEST mentor for Becky, a new nurse manager on the cardiac unit who has 4 years of previous clinical experience?

Courtney, who has been a nurse manager for 3 years. Her staff and supervisor value her skills and her leadership acumen and championship of innovation.

Which of the following exemplifies a service orientation? a.Staff members on the unit are encouraged to chart details about family support networks. b.Chart audit reveals that details related to assessment of family history are missing. c.The palliative care unit organizes a "tree of light" fundraiser each year to highlight the importance of palliative care. d.A children's preoperative holding area is initiated in response to requests from families and nurses who care for them..

D A service orntation needs to translate caring into appropriate, timely action that meets the needs of patients. Activities such as documentation of details or promotion of services may remain at the technical or conceptual level without a commitment to caring interactions. The institution of a holding area where families can be with children represents a caring action that arises out of interactions and knowledge of patient needs and thus, is service oriented.

The chief nursing officer decides to establish a client advocacy position in an oncology unit. Advocacy is best represented by: a.Establishing private and professional networking systems. b.Asking social services to handle clients' concerns. c.Identifying community support groups. d.Empowering others by promoting self-determination.

D Advocacy involves empowering and promoting self-determination in others.

Corrine, a student nurse, often hears that nurses are gatekeepers and wonders what that term means. As a nurse leader, you explain that this is a reference to the: a.Assessment and admission of patients into care. b.Orientation of patients to services once they are admitted. c.Function of controlling which patients see the physician and which do not. d.Coordination of care, services, advocacy, and access for patients within the healthcare system.

D As gatekeepers to the system, nurses advocate for and coordinate care, services, and access for patients across all providers, settings, and levels of care.

You are involved in designing a clinic for women in an inner-city neighborhood. A goal of this clinic might be: a.Development of services that are identified in various studies as important for this target group. b.Partnership with area city councilors and health professionals to provide services that are consistent with their vision and funding sources. c.Provision of immunization and addictions services and health screening services for women in the area. d.Development of services that have been identified by the women and neighborhood advocates as necessary for their health care.

D Involvement of persons in their own health care is a cornerstone of healthcare reform and is important for improving health outcomes and patient experiences. While studies and partnership with key informants are important sources of information in the design of services, engagement of the women through their neighborhood advocates may increase activation and resulting behavior. Patient activation refers to patients' willingness and ability to take independent actions to manage their health and care.

An example of a factor that would impede a patient's learning is: a.Poverty. b.Limited health insurance. c.Being an older adult. d.Heavy sedation.

D Nurses need to be sensitive to the teaching needs of those at risk for disparities in health care: persons of a different race or ethnic group, women, children, older adults, rural residents, and those with limited or no health insurance, low health literacy, and/or low socioeconomic status. It is important that lower expectations for persons who are disadvantaged, have a low literacy level, or have limited English proficiency are not unintentionally communicated.

A nurse makes a medication error that is not serious and does not cause harm to the patient. As the head nurse, your best action would be to: a.Call attention to it by posting the critical incidents report at the nursing station. b.Include the mistake on the nurse's performance appraisal. c.Apologize to the patient for the error, and indicate that discipline has occurred. d.Educate the nurse on how to provide an apology to the patient.

D Service recovery ensures responsiveness to the patient, and as part of service recovery, it is important to address an error in the most productive way, which also includes the nurse who made the error offering an apology to the patient.

The nurse manager on a pediatric intensive care unit wants to evaluate patient satisfaction. The nurse manager understands that ultimately, positive relationships with consumers of care are evaluated by the: a.Cultural sensitivity of staff. b.Cost-effectiveness of care delivery. c.Economic value of service. d.Outcomes for clients and their perceptions of care.

D Trend-setting organizations such as MagnetTM organizations need to demonstrate excellence in outcome data such as patient satisfaction. Patient satisfaction and perception of the quality of care are affected by the quality of the nurse-patient relationship. Valid measurement of patient satisfaction is an evolving science; nurses do not always accurately gauge what factors are most important to patients. Satisfaction measures are often skewed in a positive direction with scores clustered at the top of the scale.

After several months in the role of manager of a dialysis unit, Maryanne finds herself still questioning the gap in expectations between her and her staff and management and is also questioning if she can reconcile her concerns about quality care with the strong cost containment orientation of the facility. At this point, Maryanne is in which stage of role transition?

Discrepancy

The unit leader on an inpatient psychiatric unit of a large hospital has been in the position for 3 months. The unit leader is frustrated by how little time is available to work with clients and how few changes have been implemented in that time. The phase of role transition being experienced is the role of:

Discrepancy.

Sarah is a nursing graduate of 5 years who is very confident in her clinical skills. She has taken some certificate courses in leadership and management and has considered beginning a graduate degree with this focus. She is excited about being able to use her knowledge and interest by being hired as a nurse manager. Before beginning her new position, Sarah spends time with her nurse executive to clarify the executive's expectations of her and of the unit that she has been hired to manage. Sarah finds that she has begun to think negatively about the way nursing care is delivered on her nursing unit. She often wishes that she was back on her old unit and in her familiar staff nurse role. This behavior:

Diverts energy from internalization of Sarah's new role.

Sarah is a nursing graduate of 5 years who is very confident in her clinical skills. She has taken some certificate courses in leadership and management and has considered beginning a graduate degree with this focus. She is excited about being able to use her knowledge and interest by being hired as a nurse manager. Before beginning her new position, Sarah spends time with her nurse executive to clarify the executive's expectations of her and of the unit that she has been hired to manage. After beginning her new position, Sarah finds that she spends a great deal of time in direct patient care. Her staff begins to complain that they are never able to find her when they need her, and that some aspects of her responsibilities fall behind, such as scheduling. Sarah is most likely:

Experiencing difficulty in unlearning old roles.

Sarah is a nursing graduate of 5 years who is very confident in her clinical skills. She has taken some certificate courses in leadership and management and has considered beginning a graduate degree with this focus. She is excited about being able to use her knowledge and interest by being hired as a nurse manager. Before beginning her new position, Sarah spends time with her nurse executive to clarify the executive's expectations of her and of the unit that she has been hired to manage. Sarah finds that she is comfortable with the expectations of staff and her supervisor regarding her management role and responsibilities and has been able to effect a strong commitment to quality clinical care on the unit. At this point, Sarah has likely attained this role:

Internalization.

An interviewee for a nurse manager position asks for a copy of the organizational chart. Organizational charts provide information about the role component of:

Lines of communication.

The new nurse manager feels pulled between the expectations of staff, the demands of hospital administrators, and family obligations. According to Hardy (1978), unrelieved role stress and strain will lead to:

Low productivity and performance.

During an employment interview for the manager's position in a home health agency, the applicant asks questions about the medical director and about retention of staff. The nurse executive assures the applicant that the agency has few personnel problems and receives excellent support from the medical director. The applicant knows that the agency has a 50% turnover rate and has had three medical directors in the past year. The nurse executive is:

Minimizing the challenges of the position to make a positive impression on the applicant.

Seth is hired as the nurse manager for a surgical unit. After a year, the hospital reorganizes, and his position is lost. In leaving the unit, it is important for Seth to:

Negotiate a reasonable settlement.

A nurse manager notices that Nathan, an RN who has been on the unit for approximately 3 years, has a particular interest in technology and seems to be very enthused about working with software and hardware at home. She speaks with Nathan and asks him if he would lead investigation of software applications on the unit. This is an example of:

Opportunity.

The new head nurse on telemetry has been in the position for 3 months. The head nurse and the administrator disagree on how much time the head nurse should allot to various aspects of the role. Staff members on the unit complain that the head nurse is unavailable for clinical concerns because of being off the unit while attending meetings. To facilitate the process of role transition, the head nurse should:

Schedule a series of meetings with staff and the administrator to clarify expectations.

The chief nursing officer develops a mentoring program to help new staff members adjust to their new jobs. The main purpose of mentoring is:

Promoting staff retention.

Sarah is a nursing graduate of 5 years who is very confident in her clinical skills. She has taken some certificate courses in leadership and management and has considered beginning a graduate degree with this focus. She is excited about being able to use her knowledge and interest by being hired as a nurse manager. Before beginning her new position, Sarah spends time with her nurse executive to clarify the executive's expectations of her and of the unit that she has been hired to manage. A strategy that may help to make the transition to her management role and to respond to relationships and situations in her new position is:

Recognizing her strong commitment to care in the management process through journaling.

John, a new nurse manager, complains to his colleague that he feels very uncomfortable with the conflict between what he thinks he should be doing as the manager and what his supervisor thinks he should be doing. According to Hardy's role theory, John is experiencing:

Role strain.

In assisting new graduates to make the role transition to graduate nurse, Ted, the unit manager initiates which of the following?

Self-check list to assess competencies that have been strengthened

A nurse manager in one hospital values a colleague who is a few years older and has more experience in nursing management. The colleague works in another hospital, but they meet for lunch once a month. In these meetings, they share their feelings about nursing management and their lives. The function of a mentor that is missing in the relationship is:

Sponsorship.

A nurse is interviewing for a manager's position. Which of the following actions is considered a role preview?

Touring the unit

Sarah is a nursing graduate of 5 years who is very confident in her clinical skills. She has taken some certificate courses in leadership and management and has considered beginning a graduate degree with this focus. She is excited about being able to use her knowledge and interest by being hired as a nurse manager. Before beginning her new position, Sarah spends time with her nurse executive to clarify the executive's expectations of her and of the unit that she has been hired to manage. Sarah finds a mentor, Amy, who has been in the role of unit manager for 3 years and has a similar interest in clinical excellence. During their frequent meetings, Amy provides assistance with learning aspects of the manager's role, including technical aspects, such as how to interpret budget printouts and to achieve budget outcomes. The success of Amy's coaching depends on:

Willingness of Sarah to receive feedback.

As a result of Amy's coaching, Sarah, a nursing graduate of 5 years, completes a ROLES assessment. This assessment is helpful in (select all that apply):

b. Role development. c. Areas of conflict in expectations. d. Expected work time commitments.

17. The primary difference between a résumé and a CV is that a résumé: a. Reflects your skills, knowledge, and background in relation to a specific position. b. Offers a detailed listing of positions held and where positions were held. c. Includes a long and detailed explanation of academic and work experience. d. Provides contact information and focuses on your background, in general.

ANS: A A résumé is a short, customized overview of your professional life that relates to the qualifications of specific positions and how you are able to match your background to the qualifications that are desired. Provision of contact information is common to both the résumé and the CV. Résumés are more effective if details of particular positions that have been held are highlighted as compared with a detailed listing of positions held.

1. A new graduate is seeking a new position in nursing and wants to "sell" herself effectively. The best strategy is to: a. Create a résumé. b. Practice interviewing. c. Call the personnel offices. d. Create a curriculum vitae.

ANS: A A résumé provides a customized overview of details of your professional background that relates specifically to a position for which you are applying. Résumés advertise your skills to a propective employer.

21. "Stress-buffering" behaviors can be elicited to reduce stress. All of the following behavioral coping responses can be used by nurse managers to reduce and manage stress except: a. Distancing oneself from work. b. Using cognitive reframing to change irrational thoughts. c. Journaling and keeping an informal diary of daily events and activities. d. Exercising regularly.

ANS: A Achieving balance between work and leisure is a useful strategy for stress reduction. Distancing, however, can be a sign of depersonalization that includes negative attitudes as well and is a characteristic of burnout.

12. Knowing your professional strengths is important to: a. Finding your fit in positions and a career path. b. Maintain a professional status. c. Act in a manner that is legal and ethical. d. Understand the role expectations of a position.

ANS: A Being aware of your strengths is critical in determining what you will bring to a position and can be used to find your fit and possible career path. Knowledge and experience are important in maintaining the privilege of belonging to a profession and of behaving ethically and legally.

16. Sources of occupational stress in nursing include all except which of the following? a. Authoritarian leadership b. Concern about moral wrongdoing by colleagues c. Multiple changes in a short time d. Job insecurity

ANS: A Ethical distress, complexity compression, job insecurity, high acuity levels, rotating shifts, and workload are all sources of work-related stress for nurses.

12. In a job interview for a nursing position, Marley can be assured that which of the following will occur? a. Both eustress and distress b. Only eustress c. Only distress d. Neither eustress nor distress

ANS: A Eustress is defined as stress that is pleasant in nature, and distress is defined as stress of an unpleasant nature. One can assume that every interview has both of these stresses.

18. A functional résumé focuses on: a. Experience and skills gained in positions. b. Positions held and specific roles in the positions. c. Academic qualifications and achievements. d. Relating skills and experience to qualifications in a specific position.

ANS: A Functional résumés highlight skills and experience gained rather the details of specific positions. As with résumés in general, skills and experiences are customized to create an image of an individual in a particular position.

13. Which of the following will require greater attention in the future? a. Chronic disorders b. Obstetrical outcomes c. Prevention of hospital-based errors d. Team conflict resolution strategies

ANS: A Lifestyle choices, obesity, and an aging population will lead to an increased emphasis on prevention, personal accountability, and innovations such as robotics in the management of chronic illnesses.

20. Rama, a psychiatric nurse, is assigned to four patients. The patient that would be at greatest risk for psychological compromise is the patient who has experienced: a. The death of a spouse. b. The death of a distant friend. c. A recent job layoff. d. A divorce.

ANS: A Most studies that have been concerned with "life stressors" have found that the death of a spouse produces the greatest stress.

3. Which of the following actions is most representative of how health care of the future might be delivered? As a nurse leader, you: a. Refer families who require immediate help to a local food bank. You also work with local agencies and families to establish a mothers collective in which mothers learn about nutrition and prepare low-cost, nutritious meals that are shared with the mothers in the collective. b. Work together with a local agency to set up a free clinic in which addicts and the homeless can receive free health care and prescriptions for immediate needs. c. Ensure that individuals who are admitted to your unit are asked about their smoking history and that preoperative and postoperative planning takes into account how smoking will affect status during and after surgery. d. Address the health of those who are overweight and obese on your unit by ensuring that hospital meals offer nutritious, healthy food choices that are satisfying.

ANS: A Referral to a food bank addresses the needs of a specific population, while exemplifying an important leadership strength: thinking long-term, acting short-term. The project that involves mothers will teach mothers about nutrition and engage them in preparation of low-cost, healthy meals while promoting longer-term changes in healthy eating.

23. A necessary leadership strength for nurses of the future is: a. Inspiring others to work their best to create the future. b. Understanding the nuances of fundraising to make up funding shortfall. c. Guarding the tendency of other professions to encroach on nursing roles. d. Adapting work life to an aging nursing workforce.

ANS: A Senge said that all leadership is really about is people working at their best to create the future. Predictions of the future highlight the importance of interprofessional teams and of a shift towards understanding the importance of health care.

2. The nurse manager of a unit has lost many staff members, and the unit is now staffed with a large number of agency and traveling nurses. She knows that the agency and traveling nurses are all contracted to stay on the unit for the next 3 months. One way to improve morale and decrease stress in the unit would be to: a. Plan a social event and include the agency and traveling nurse staff members. b. Plan unit-based social events for your remaining permanent staff members. c. Request hospital-based "floating" nurses to substitute for the temporary staff. d. Implement team nursing.

ANS: A Social support, in the form of positive work relationships, can be an important way to buffer the effects of a stressful work environment. Including all staff in the social event enables those who are not normally part of the team to experience this support and provides an opportunity for the staff as a whole to develop supportive relationships.

8. The staff development educator presents a series of programs on stress management to the nurse managers. Research has indicated that an individual's ability to deal with stress is moderated by psychological hardiness. Psychological hardiness is a composite of: a. Commitment, control, and challenge. b. Commitment, powerlessness, and passivity. c. Commitment, control, and passivity. d. Decreased isolation, challenge, and passivity.

ANS: A Some people have the capacity to accept changes in life with good humor and resilience, which, in turn, influences behavior that prevents illness. Hardiness involves the capacity to manage time and stress, to reframe situations positively, and to commit.

17. Mr. T. Jones and Mr. R. Smith are both going to become residents in Sunny Haven Lodge. Mr. Jones views it as an opportunity to socialize and meet new friends. Mr. Smith views this as abandonment by his family and is worried that the care will be inadequate. Each senior perceives the situation differently. This is a good example of stress that is: a. Both a positive stressor and a negative stressor. b. Occurring only because of age. c. Positive in both cases. d. Harmful in both cases.

ANS: A Some researchers have determined that stress is a person-environment process in which the persona appraises the situation as taxing or not. Appraisal is an important concept that explains why two people react in different ways to the same situation.

14. Steady state styles would be most likely in which of the following situations? a. Small hospital, in an isolated rural setting, with limited hierarchy b. Large urban teaching hospital c. Health network with several organizations d. Travel nurse agency

ANS: A Steady state career styles (career-long commitment to a particular position) are more likely in rural settings, where commitment to the community is high and alternative career opportunities are limited.

5. A hospice nurse has been feeling very stressed at work because of both the physical strain and the emotional drain of working with clients with AIDS. She tries to walk 1 to 2 miles three times a week and to talk regularly with her husband about her work-related feelings. One reasonable stress management strategy would be to: a. Start taking yoga lessons. b. Make an appointment to meet with a psychiatrist. c. Start jogging 5 to 6 miles every day. d. Plan to go out for a drink with fellow nurses after work every day.

ANS: A Stress relief techniques include 30 minutes of exercise five times a week, as well as techniques such as yoga that relieve mental stress.

Which of the following would be most in line with Hersey and Blanchard's concepts? a. The team of caregivers on day shift are familiar with their roles and with the patients. The nurse manager decides to work on the unit budget in her office. b. After a year of working on the unit, Shari, an LPN, is still hesitant about many policies and procedures. The charge nurse decides to challenge Shari with more difficult patients. c. The nursing supervisor asks one of her charge nurses to lead a technology integration project. The supervisor continuously demands involvement in decisions that the charge nurse is making in the project. d. Team members complain that Alysha, an RN, is unmotivated, and that she refuses assignments that are complex or difficult. The charge nurse suggests that Alysha is relatively new and that she needs time to adjust.

ANS: A The Hersey and Blanchard framework suggests that when ability (skills, job knowledge) and willingness are strong, the involvement of the delegator is less.

3. The nurse executive at the local hospital began working there 20 years ago and has risen to the highest designation in nursing. This career style is known as: a. Linear. b. Spiral. c. Steady state. d. Entrepreneurial/transient.

ANS: A The linear career style, as described by Friss (1989), refers to vertical advancement in an organization and is of interest to those with a desire to gain organizational knowledge and different perspectives on nursing.

8. As a team, you and the staff have determined that there is a need to reduce medication errors on your unit. Together, you developed the questions that you would like addressed and searched the literature for relevant research studies. Based on the evidence, you suggested a change to your practices and now are involved in implementation of these changes. Today, there was a major study released that would significantly change what you have decided to do. What are you and your staff experiencing? a. Compression complexity b. Distress c. Information lag d. Technology advancement

ANS: A This situation exemplifies complexity compression, a term that means many changes are happening almost simultaneously and before one practice can be firmly implanted in our minds, we are already addressing some other new change. This compression can be distracting or useful.

The nurse on the 7-7 shift is assigning a component of care to an unlicensed nursing personnel (UNP) employee. The night nurse should remain: a. Accountable. b. Responsible. c. Authoritative and liable. d. Responsible and task-oriented.

ANS: A When a registered nurse delegates care to a UNP, responsibility is transferred; however, accountability for patient care is not transferred. Thus, "accountability rests within the decision to delegate while responsibility rests within the performance of the task" (Anthony and Vidal, 2010, p. 3).

MULTIPLE RESPONSE 1. High levels of work-related stress affect all but which of the following? (Select all that apply.) a. Job satisfaction b. Absenteeism and turnover c. Nurses' health d. Client welfare

ANS: A, B, C, D Many writers and researchers have found that these work-related areas are adversely affected by stress.

8. A nurse is applying for a new position. This position is one in which she will serve as a liaison between a hospital and a school of nursing. The nurse has to update her résumé to include her teaching experience. The goal of creating a curriculum vitae is to: a. Have a listing of facts about your professional life. b. Create an opportunity to be interviewed. c. Respond quickly whenever a position becomes available. d. Be certain you can recall facts for a prospective position.

ANS: B A curriculum vitae provides an all-inclusive but not detailed listing of your professional life.

9. To develop a curriculum vitae, or résumé, a nurse must develop a personal data file. The goal of a personal data file is to: a. Create an opportunity to be interviewed. b. Have a listing of facts about your professional life. c. Have a tool in place for marketing yourself. d. Create a document that lists your skills.

ANS: B A personal data file enables storage and recall of career-specific details that can be retrieved and shaped for a specific purpose using cut and paste approaches rather than creating whole new documents.

You ask Evelyn, a new UNP, to check what is left in Mrs. N.'s inhaler when Evelyn makes visits to Mrs. N. and also to check whether Mrs. N. is receiving any positive effect from the medication. Evelyn reports for 3 weeks that Mrs. N. is using the inhaler and that there is enough medication left in the device. The day of her last visit to Mrs. N., Mrs. N. is admitted to the hospital in severe respiratory distress. When she is admitted, she tells the physician that she has not been using the inhaler for 4 weeks. Determination of Evelyn's educational preparation and certification is related to the concept of: a. Accountability. b. Authority. c. Role performance. d. Assignment.

ANS: B Authority refers to the right to do and may be designated by law, educational preparation, or job description.

22. The education consultant at St. Joseph's Hospital is giving a workshop on Cognitive Reframing. The consultant explains that Cognitive Reframing reduces stress by: a. Aiding individuals in identifying positive stressors. b. Helping people realize that negative thinking causes emotional distress. c. Eliminating negative stressors. d. Replacing positive self statements with negative irrational beliefs.

ANS: B Cognitive Reframing is a therapy that aids individuals in discovering that their irrational thoughts can be replaced with responses that are more rational. It enables individuals to gain a sense of control over the situation and can change "I'll never ..." to "I can ..." or "She always ..." to "Sometimes she ..." It is an approach that allows individuals to replace negative thoughts and statements with others that are more realistic and helpful.

20. A number of changes are introduced to a unit, including changes to familiar clinical procedures and the use of PDAs to enable bedside documentation. You, as unit manager, anticipate which of the following will contribute most to complexity compression? a. Meaning of the change b. Pace of changes c. Previous experiences with change d. Confidence of the leader in the value of the change

ANS: B Complexity compression is a term that means that many changes are happening almost simultaneously and before one practice can be firmly implanted in our minds, we are already addressing some other change

During staff development programs, staff nurses verbalize their frustration about their workloads and having to delegate so many tasks to others. One of the main reasons that delegation has emerged as an issue is because of: a. The amount of paperwork required to complete care. b. The complexity of client care. c. Earlier discharge practices. d. The numbers of other disciplines present on a given unit.

ANS: B Complexity of client care, a multilevel nursing model (registered nurses, mixed with LPNs/LVNs, and UNPs), and community-based care provide many challenges in determining the care required and outcomes desired and/or mandated, and in matching needs with various abilities and authority of regulated and unregulated healthcare providers. The nurse manager should ensure that staff is clinically competent and trained in their roles in patient safety.

23. To enhance team leadership skills for your team leaders, you arrange opportunities for: a. Certification. b. Continuing education. c. Graduate courses. d. Volunteerism.

ANS: B Continuing education provides systematic learning opportunities that augment existing skills and knowledge for delivery of quality care and advancement of career goals. Graduate and certification courses provide advanced knowledge and skills.

Functions such as "delegates tasks to assistive personnel" that are outlined in a position description for an RN Team Leader would be considered: a. Active delegation. b. Passive delegation. c. Passive accountability. d. Active responsibility.

ANS: B Delegation of functions that are normally considered part of or an essential part of the practice of a licensed person through a position description is considered passive delegation.

The day shift nurse asks an LPN/LVN to complete a component of care for a client. The day shift nurse is engaging in what function? a. Delegating b. Assigning c. Sharing d. Authorizing

ANS: B Delegation refers to transfer of responsibility for work; the day shift nurse retains accountability for the outcomes of patient care.

With delegation, responsibility and accountability remain with the: a. Physician. b. Professional who delegates. c. Individual who receives the delegation. d. Individual who previously performed the task.

ANS: B Even though the delegatee (the one who receives the delegation) receives direction from the professional who delegates a task and must have the authority to complete it, the delegator retains accountability for the overall outcome and completion of the activity. The delegatee has responsibility (obligation to engage in the task) and authority for the task.

14. An example of role conflict occurs when: a. The director of ICU and the manager of the surgical unit wish to hire the same new employee. b. Two part-time staff members are hired to work in a unit, but the job expectations for them are not clear, and the head nurse expresses disappointment in their performance. c. The nurse manager for ICU believes he does not receive as many resources as the nurse manager for nephrology. d. Line managers believe that support staff use their technical knowledge to intrude on their authority.

ANS: B Failure to comply with expectations can lead to role conflict. Role conflict and role ambiguity are major sources of conflict for nurses.

9. The chief nursing officer understands that a nurse manager can exhibit stress that is related to management mistakes. An example of a management mistake is: a. Achieving excellence on the job. b. Following others' expectations. c. Organizing the desktop. d. Organizing the calendar of events.

ANS: B Following the expectations of others is attributed to role strain. Role strain involves a subjective state that occurs when social stress is present.

3. As a nurse manager, the one activity you should not overlook is: a. Posting the yearly rotation schedule. b. Reviewing vacation requests. c. Scheduling staffing for holidays 6 months in advance. d. Anticipating staff sick days.

ANS: B Free time and vacation time are needed for individuals to recharge. If time for work is more than 60% of wake time, or when self-time is less than 10% of wake time, stress levels increase.

7. As a nurse manager, it is important to become a "future thinker." Which is an example of a "future thinker"? a. Keeping traditional practices b. Moving toward evidence-based practices c. Finding less need for more knowledge d. Believing that macromarketing will be a necessity

ANS: B Future forecasts include evolutions in power, structure, and knowledge; rapid change in the healthcare system; the demise of macromarketing; and increased evidence-based practice.

7. The chief nursing officer listens to nurse managers verbalize their feelings of internal stress. One common source of internal stress seems to be: a. The death of a loved one. b. Perfectionism. c. Getting married. d. Losing a job.

ANS: B Losing a job, the death of a loved one, and getting married are examples of external stress.

5. Your organization is in the middle of re-designing patient care units, with decisions based on best practices and various other sources of evidence. In the middle of the transition, there is a temporary halt called to the transition because of a re-design of the health care system and greater emphasis on primary care. What would be a healthy response to this situation? a. Salvage as much of the original planning as possible so as to reduce expenditures. b. Engage in consultation to create innovative solutions that bridge the existing plans and the new directions. c. Abandon the current planning in favor of addressing the new trends. d. Continue with the current planning because trends come and go.

ANS: B Stability and total chaos are the ends of a continuum. Moving in some way between those two ends suggests that we live in a constant state of disequilibrium in which we strive toward stability while recognizing we experience chaos. As we continue to move from "traditional" practices to evidence-based ones and from a heavy focus on tertiary care to one that values primary care, we can assume that we might experience more chaos. Chaos can lead to new learning and new, innovative solutions. As nurses, it is important to be able to function in an evolving environment.

18. Jeff, an RN in his 30s, has lost a parent, just purchased a new home, and is laid off with 6 months' severance pay. A the same time, Jerry, an RN in his 50s, is financially secure and is asked to take early retirement with a buyout. How will the two men react to the emotional and physical influences and the sequence of stress? a. The younger man will feel more stress. b. The two men may or may not feel the same amount of stress. c. The older man will feel more stress. d. Neither man will experience any stress..

ANS: B The amount of stress is unpredictable. Responses to stress are affected by factors such as age, gender, personality, culture, lifestyle, health, and life experiences

11. Your unit has a number of patients who have undergone limb amputation. In working with the clients, you begin to think beyond therapies such as pharmacotherapeutics and surgery and you explore biomechanics, robotics, mind-body approaches, and cognitive behavioral therapies as possibilities in working with these clients. You begin to amass information in a number of areas with which you previously had little familiarity. According to the Wise Forecast Model©, you are a. Acting wildly. b. Learning wildly. c. Engaging in interprofessional care. d. Increasing your complexity compression.

ANS: B The first step, learn widely, means that we must extend our sources of knowledge beyond our role and clinical areas of interest. In fact we must extend our learning beyond nursing and health care. Widely might encompass another discipline such as architecture or engineering.

1. Nursing professionals in the twenty-first century will accomplish most of their work: a. Through teams of internationally prepared professionals. b. In teams and through group work. c. Through long-term, secure jobs. d. In competitive environments and work groups.

ANS: B The future is about teams and group work. Competition will be out and collaboration will be in. Job security will be out and career options will be in. Our brightest and best may leave more often than they do at present to pursue career options internationally.

You ask Evelyn, a new UNP, to check what is left in Mrs. N.'s inhaler when Evelyn makes visits to Mrs. N. and also to check whether Mrs. N. is receiving any positive effect from the medication. Evelyn reports for 3 weeks that Mrs. N. is using the inhaler and that there is enough medication left in the device. The day of her last visit to Mrs. N., Mrs. N. is admitted to the hospital in severe respiratory distress. When she is admitted, she tells the physician that she has not been using the inhaler for 4 weeks. This incident is an example of: a. Incompetence of the UNP. b. Failure to follow through. c. Skills but no motivation. d. Lack of accountability.

ANS: B The nurse should maintain open lines of communication and seek information, and the UNP should know how, when, and what to report. Communication of delegation of tasks includes specific information about what is being delegated, expected outcomes, and deviations (which includes what immediate action needs to be taken). This two-way communication and follow-through allows patient care to be altered, if necessary, in a timely manner.

5. A young male nurse began in nursing as a staff nurse at a hospital. After 3 years, he moved to a home healthcare agency. After 2 years, he was promoted to a managerial position and now has returned to another hospital as a new director of nursing. This career style is known as: a. Linear. b. Spiral. c. Steady state. d. Entrepreneurial/transient.

ANS: B The spiral career style, as described by Friss (1989), involves an in and out and up and down approach to opportunities.

12. The starfish analogy is exemplified in which of the following? a. A unit manager resigns after continued tension between the administration and her regarding implementation of primary nursing. The primary nursing project dies. b. Nurses try to establish a clinic that provides ambulatory care to parents and young children in an impoverished neighborhood. Community members advocate for funding from political leaders and insurers. c. Alana, a new graduate, promotes continence care based on evidence. When she presents her ideas, senior staff refuses to consider it. d. The head of a community health service moves on to another position. Programs are disbanded.

ANS: B The starfish analogy points to the connectivity that we have with one another and how we influence and are influenced by others all the time. This affords many opportunities for leadership that are dependent not on formal titles but on opportunities to shape the work at hand.

During a fire drill, the nurse manager becomes very assertive and directive in her communications with staff. This type of situational leadership depends on: a. Supportive behavior by the leader and immature followers. b. The development level of the followers and the type of behavior of the leader. c. Well-developed followers combined with a strong leader who acts quickly. d. The leader's ability to evaluate personnel and communicate that evaluation.

ANS: B When abilities, relationships, and/or time is limited (as in a crisis situation), the leader assumes a bigger role in guiding and in making decisions (Hersey and Blanchard and "telling" behavior).

19. You ask Evelyn, a new UNP, to check what is left in Mrs. N.'s inhaler when Evelyn makes visits to Mrs. N. and also to check whether Mrs. N. is receiving any positive effect from the medication. Evelyn reports for 3 weeks that Mrs. N. is using the inhaler and that there is enough medication left in the device. The day of her last visit to Mrs. N., Mrs. N. is admitted to the hospital in severe respiratory distress. When she is admitted, she tells the physician that she has not been using the inhaler for 4 weeks. Before assigning Evelyn to Mrs. N.'s care, the most appropriate action of the care coordinator would have been to: a. Determine Evelyn's educational background and preparation for this role. b. Ask Evelyn if she has worked with inhalers before and to describe what she knows about them. c. Advise that if Evelyn has any questions about what to do with the inhaler, she should come to the coordinator. d. Advise Evelyn that working the inhaler is not really complicated and that she should ask the patient how to check medication levels in the inhaler.

ANS: B When delegating tasks, in addition to specifying the task to be completed, outcomes, priorities, timelines, deviations, report time frames, monitoring, and resources, asking the delegatee to give examples of each is helpful in ensuring that communication is clear and has been understood. Preparation of UNPs lacks consistency; therefore, the safest practice is to determine the knowledge and skill level of the UNP in relation to the skill and the patient before delegating.

Which of the following indicates safe delegation? a. The nurse supervisor for a large urban acute care department asks the unit manager to accept two new acutely ill patients, which the manager does. The unit is short two staff, and the replacement is inexperienced. b. A unit manager agrees to release a staff from her unit to Unit B. The staff member she agrees to release is experienced on Unit B and is agreeable to the change. The unit manager's unit is fully staffed and patients are stable. c. The nurse supervisor asks the head nurse for Unit A to make do without a replacement for an ill staff member because Unit A was originally overstaffed anyway. Patient acuity levels are very high on Unit A and two staff are orientating. d. The nurse supervisor asks the charge nurse on Unit B to cover Unit F, which is two floors up, because the charge nurse for Unit F is ill. The charge nurse for Unit B is an experienced manager but has no experience with the nursing care required on Unit F.

ANS: B When span of control (number of individuals for whom a manager is responsible) is compromised by geographic factors such as lack of proximity, instability in patients' conditions, or lack of experience, the span of control that is being delegated may lead to unsafe care.

10. An experienced staff nurse applies for a distinct position of patient advocate at a new clinic. Based on the various tools available to her, which of the following should she bring to her interview to best present her skills? a. Résumé b. Annual evaluations c. Portfolio d. Patient advocacy project

ANS: C A portfolio enables prospective employers to view evidence of significant achievements and responsibilities that would be pertinent to the position.

21. A well-written letter of resignation is critical to: a. First announce your intent to resign. b. Formally signal discontent in your current position. c. Maintain a positive relationship with your former organization and colleagues. d. Fulfill your legal obligations as a departing employee.

ANS: C A well-written resignation letter outlines your intent to leave the organization and appreciation of the organization but should follow an initial meeting with your manager to first discuss your intention. A well-written letter maintains a positive relationship with the organization.

16. Which aspect of our tradition and history in nursing may impede our movement towards future-oriented thinking? a. Lack of confidence b. Focus on the discipline of nursing c. Focus on details in the everyday practice d. Mistrust of trends and new evidence

ANS: C Because of our history of attention to details, we may need to challenge ourselves in developing our ability for leadership. Moving from micromanaging to focusing on setting expectations for those for whom we are accountable may feel uncomfortable. However, that movement reinforces our ability to deal with longer-term issues.

10. The chief nursing officer has been developing her portfolio for years. What is the chief nursing officer modeling? a. Her clinical expertise b. Affection for tradition c. Her employability d. Her busy professional life

ANS: C Being employed is no longer sufficient; we must be employable. A portfolio outlines achievements and experiences that communicate employability.

2. A nurse manager has been employed in the same facility for 20 years and has held the same position. This career style is known as: a. Linear. b. Spiral. c. Steady state. d. Entrepreneurial/transient.

ANS: C Career styles that are marked by selecting and staying in a role throughout a career are characterized as steady state.

6. A unit manager recently graduated as a clinical nurse specialist with a focus in gerontology. She applied to take a certification exam. Certification is designed to recognize: a. Basic knowledge in a specified area. b. Advanced practice in functional roles. c. Special knowledge beyond basic licensure. d. Continued competence as a registered nurse.

ANS: C Certification is an expectation in some settings for career advancement in advanced practice or in specialized areas and goes beyond basic preparation.

24. Nathan tells you that he has selected nursing as a career because many jobs are available and he will have job security. Your best response to Nathan is: a. "With many young people going overseas, many jobs and options will be available. Stable jobs and job security will be part of the nursing employment market." b. "The job market for nurses will be diminished with funding cuts to hospitals." c. "The employment prospects for nurses are positive with many options to choose from. Flexibility and adaptability are essential to income security." d. "It is unlikely that nursing will survive in the long term with funding cuts and a declining population of seniors."

ANS: C Employment for nurses continues to be positive, although roles will change in a rapidly changing environment, which will increase options. With the number of options available and sporadic work opportunities, nurses will need to be flexible and able to adapt rapidly. Job security will be out; career options will be in.

20. During an interview for a manager's position, you find the supervisor and staff unfriendly. Responses to questions are met with vague responses. After the interview, you decide not to pursue the position. What follow-up, if any, is most appropriate? a. There is no need for you to do anything further. You likely will not be offered the position anyway. b. You should file a complaint with Human Resources about the supervisor's lack of interviewing skills. c. You should send a thank you note to the interviewer, indicating appreciation for her time. d. You should call and leave a voicemail, indicating your disinterest in the position.

ANS: C Even if you are disinterested in the position or think that the interview has gone badly, an appropriate follow-up is a thank you note to the interviewer. This recommended follow-up creates a positive impression and may leave open the possibility of future interactions.

Ali, an RN on your unit, is consistently late to work and makes remarks such as "Do you really want me to do that?" when patients and care are assigned to her. You have spoken with her frequently about her: a. Ability. b. Skills. c. Attitude. d. Personal issues.

ANS: C Hersey and Blanchard explained followership in terms of ability and willingness. Attitude refers to willingness or reluctance to perform work.

An RN colleague, who is a long-standing and collaborative member of your team, is performing a complex and novel dressing for the first time for the patient to whom she has been assigned. Which of the following would be the most appropriate communication with her? a. "How do you usually do this kind of dressing?" b. "The dressing needs to be done today and tomorrow with the supplies on this cart." c. "Here is what you need for the dressing, and I will show you what needs to be done." d. "I know you know what you are doing. Let me know if you have any problems."

ANS: C If a situation involves a new task and the relationship is ongoing (two individuals who will usually continue to work together), the delegator explains what to do and how to do it. Hersey described the leader's behavior as explaining or persuading, which, is characterized as "selling." The RN who is assigned to the patient is an experienced nurse and team member, but is new to this specific situation. In situations where the nurse is experienced but the task is new, explain (and demonstrate) what needs to be done.

The unit manager is working in a large metropolitan facility and is told that two UNPs are to be assigned to work with her. Delegation begins with: a. Acknowledging the arrival of the second UNP on the unit. b. Providing clear directions to both UNPs. c. Matching tasks with qualified persons. d. Receiving reports from the prior shift.

ANS: C In delegating to the UNPs, the nurse must consider what cannot be delegated, as well as the factors of safety, time, critical thinking, and stability of patients.

15. On your curriculum vitae, which of the following is the recommended approach for listing employment and educational history information? a. 1979 RN Diploma 1985 BScN 2002 MN b. 2002 Mount Rush Health Center Staff Nurse 1997 Cedar Falls Clinic Staff Nurse 2007 Kilkarney Rehab Center Case Manager c. 2007 MN 2005 BScN d. 1997 Sturgeon County Hospital Head Nurse 2002 Sturgeon County Supervisor 2007 Sturgeon County Director

ANS: C Information that is included in the body of the curriculum vitae should always be in reverse chronological order so that the most recent and, presumably, most relevant job information appears first.

11. During performance appraisal interviews, Joanne's nurse manager notices Joanne's excitement when she talks about how she has helped patients on a rehab unit understand the complexities of their regimens. When Joanne's nurse manager asks her about her career path plans, Joanne says that she wants to become a nurse administrator. The best response to Joanne would be: a. "Nursing administration is rewarding. What experiences would help you along this path?" b. "You do not appear excited about nursing administration. Unless you are excited by that career path, I wouldn't advise going in that direction." c. "You seem to find teaching others very rewarding. Have you considered that as a possible career path?" d. "You are too inexperienced to consider administration. Work for a few years, and then consider administration."

ANS: C Joanne evidences excitement about teaching patients, and although administration could be a rewarding path for her, education might be a better option for her to consider. Looking at job aspects that are rewarding is helpful in determining which career direction to pursue.

24. As a unit manager, you chair the unit meetings. For each meeting, you consider and establish the purpose of the meeting. Second, you prepare an agenda. Arrange the following steps in an order that would make the meetings productive and successful. 1. Distribute an agenda. 2. Control the flow of interactions. 3. Select team members. 4. Start on time. 5. Keep the meeting focused and directed toward accomplishing the set objectives. Select the correct order from the following options: a. 1, 2, 4, 5, 3 b. 4, 1, 2, 5, 3 c. 3, 1, 4, 5, 2 d. 3, 4, 2, 1, 5

ANS: C Planning, organizing, and keeping the group on task are critical in ensuring that meetings are productive and that time is managed well.

24. Professional associations: a. Set regulatory requirements and establish entry requirements for nursing. b. Offer graduate programs for clinical and career advancement. c. Provide opportunities for career networking and support. d. Are open to all individuals who meet the criteria for membership.

ANS: C Professional associations are frequently, although not always, voluntary groups whose members provide leadership in issues and policies of interest to nurses. Professional associations also offer networks of nurses with similar interests.

Which of the following exemplifies accountability? Karen, the nurse manager on 5E: a. Consistently submits her budgets on time. b. Gets along well with her staff and with other managers. c. Outlines her rationale for reduction of RN coverage on nights to the Nursing Practice Committee after serious patient injury. d. Actively solicits ideas regarding scheduling from her staff.

ANS: C Reliability, dependability, and obligation to fulfill the roles and responsibilities of the nurse manager are consistent with responsibility. Accountability refers to being answerable for actions and results

6. Your organization is in the middle of re-designing patient care units, with decisions based on best practices and various other sources of evidence. In the middle of the transition, there is a temporary halt called to the transition because of a re-design of the health care system and greater emphasis on primary care. As a manager in this situation, your staff experience a gap between what they expected (the original re-design of the units) and what is actually happening (a need to integrate primary care in some way). According to Selye, the nurses on your nursing team are likely experiencing what? a. Eustress b. Distress c. Stress d. Compression

ANS: C Selye described stress as being on a continuum between stress that is positive (eustress) and stress that is negative (distress). Because individuals perceive the same event differently, from the information that is given, it is not possible to determine whether the nurses experience the events as eustress or distress; what is most likely is that the nurses are experiencing stress, which is what occurs when there is a gap between expectations and what is actually occurring.

17. A stroke unit experiences numerous changes related to implementation of new technology, a changed nursing care delivery model, and staff turnover within a period of 6 months. Staff members begin to show signs of reluctance to implement any more changes. This exemplifies: a. A poor relationship between leaders and staff. b. Lack of knowledge regarding the importance of changes. c. Striving to achieve stability in the midst of great disequilibrium. d. The importance of chaos in promoting adherence with established practices.

ANS: C Stability and chaos are at opposite ends of a continuum. When chaos is present, change occurs but life may seem uncontrollable. Resisting further change is a move toward establishment of equilibrium.

19. A business condition that may assist forecasting but add to the complexity of change is: a. Consulting with professional groups about change. b. Researching about trends on futuristic sites. c. Asking patients to examine options for change. d. Building profit projects and sustainability into planning.

ANS: C Stalk and Butman suggests that asking the customer for feedback on options may assist with forecasting. Adopting this strategy runs counter to current practice and would increase the complexity of healthcare planning and forecasting.

1. As a nurse manager, you observe a staff nurse who over the past few weeks has become withdrawn and has had several absences due to minor ailments. Your best action would be to: a. Ask the nurse if she is okay during report. b. Refer the nurse to the employee assistance program. c. Ask the nurse to meet with you for a few minutes before she leaves for the day. d. Write a note to the nurse advising her that her work attendance must improve.

ANS: C Stress can lead to emotional symptoms such as depression and a variety of ailments. Meeting with the nurse privately may assist in identifying stress and possible solutions.

11. Which of the following statements would best define stress? Stress is: a. The comfortable gap between how we like our life to be and how it actually is. b. Everyday life, both the highs and the lows. c. A consequence or response to an event or stimulus that can be positive or negative. d. Identical to distress.

ANS: C Stress is defined as a consequence or response to an event or stimulus. It is not inherently bad (distress), and whether highs and lows are seen as distress or eustress is dependent on each individual's interpretation of the event.

2. Based on studies of workplace environments for nurses and future projections, the workplace of the future will be: a. Less intense because of more technology. b. About the same as it is now. c. More intense because of more technology. d. Fluctuating between intense periods and less intense periods.

ANS: C Technology will continue to revolutionize health care and contribute to complexity compression. In addition to access to knowledge, electronic records, and current applications of technology, technology will include robotics, which will change how chronic disease can be managed, and bioengineering will make possible interventions that do not yet exist.

21. In order to plan long term, you consider what the client of the future will look like. Which of the following client profiles would best capture shifting demographics and trends in health care? a. Younger, knowledgeable about health options b. Female, uses emergency care services for parents and children c. Older, one or more chronic disorders, diverse background d. Male, various occupationally generated disorders, diverse ethnic background

ANS: C The client base is aging and more people will be living with chronic disorders. Persons will travel more and there will be increased need to speak two or more languages to address the needs of a mobile global population.

4. You notice that wait times in your Emergency Department are growing longer, because of factors such as increases in the numbers of persons with chronic disorders, discharge of patients into the community at a higher level of acuity, and limited resources for transfer of inpatients. You begin to think about an application that would use your knowledge of the Emergency Department but also software and business applications and wonder if this would reduce wait times. You have not encountered anything similar to this idea. According to the Wise Forecast Model©, you are in what phase? a. Wild thinking b. Act widely c. Think wildly d. Learn widely

ANS: C The second step in the Wise Forecast Model© is to think wildly. Step two is designed to create connections among disparate thoughts. This thinking might be seen as the start of innovations

7. Which of the following situations is most likely to result in a productive, whole work situation? a. Amy, RN, 5 years of experience in the emergency department. Amy accepts a position working with older clients in a home health agency because she has relocated and this is the only full-time position available. b. Adam, RN, 8 years of experience in various nursing positions, including that of a nurse manager. Adam accepts a new nurse manager position because he has a family and wants more regular hours. He is most comfortable working in direct client care. c. Louise, RN, 10 years of experience in an emergency department. She accepts a position as a case manager in home health care, working with older clients. She especially enjoys working with older adults and wants to take on leadership and management challenges. d. Courtney, RN, a new graduate. Courtney is getting comfortable with delivering nursing care as an RN. She is offered a position on surgery as a permanent team leader. The unit has had a great deal of turnover recently, and only limited mentorship is available.

ANS: C The whole of any work situation is composed of two elements: person and position. A productive, whole work situation results when a person's talents and strengths are successfully blended with expectations of the position. Of the situations described previously, the one most likely to result in a productive, whole work situation is that of Louise, who although her experience has not been in home health, is interested in both the roles and the responsibilities of the position, as well as the target group being served. She also considers the group being served as an area in which she demonstrates strength.

The nurse manager is setting up the room assignments for the unit. She has one critical patient on the unit, who is going to require more care than the others. Before delegating a task, a nurse manager should: a. Delegate the admission assessment to the LPN. b. Review the employee's performance assessment for the most recent period. c. Assess the amount of guidance and support needed in a particular situation. d. Create a task analysis of critical behaviors for the individual.

ANS: C To delegate effectively, the nurse manager must assess the abilities required in the situation and the abilities that staff have to anticipate the amount of direction, monitoring, explanation, and independence that can be assumed.

County Hospital has position descriptions for all staff, including RN Team Leaders. Sarah, a team leader on the rehab unit, assesses the needs of the patients in her area, assesses the skills and backgrounds of each of the individuals on her team, and then assigns and delegates the appropriate care provider to each patient and task. Sarah's activity in the example described is termed: a. Passive delegation. b. Passive accountability. c. Active delegation. d. Active responsibility.

ANS: C When a position description contains functions that are considered to be the normal practice of the person in that role, then it is considered a passive delegation act. When Sarah decides what is best for the patients in her care in terms of who should perform the care and then holds the person accountable, she is engaging in active delegation.

County Hospital has position descriptions for all staff, including RN Team Leaders. Sarah, a team leader on the rehab unit, assesses the needs of the patients in her area, assesses the skills and backgrounds of each of the individuals on her team, and then assigns and delegates the appropriate care provider to each patient and task. Sarah provides Colleen, her RN colleague with details regarding the patients to whom Colleen has been assigned on the day shift. This is an example of: a. Accountability. b. Responsibility. c. Assignment. d. Delegation.

ANS: C When an RN assigns care to another RN, it is termed an assignment and not delegation, because both accountability and responsibility are transferred.

2. An example of a career is (select all that apply): a. Employment in short-term contract jobs in business, nursing, and whatever is available. b. Involvement in an area of practice that is regulated. c. Continuous employment in the same position and the same arrangement for 20 years. d. Moving into and out of nursing positions in various cities while pursuing travel and education that develops understanding of global health.

ANS: C, D Career refers to progression of skills, consistency, knowledge, and/or status. Career styles can be defined as linear, steady state, entrepreneurial, or spiral. Deepening skills in one area is an example of a steady state career style, whereas moving into and out of positions in various cities can characterize an entrepreneurial style. Involvement in a regulated field defines a professional interest but not necessarily a career. Work opportunities that are expedient define an interest in employment but not necessarily a career path.

MULTIPLE RESPONSE 1. Thomas has been a nurse in your ICU for 10 years. In facilitating Thomas' professional development, you would focus on (select all that apply): a. Certification for the ICU environment. b. Discussions about how Thomas can fit with role expectations and relationships. c. Possible specializations within the ICU environment. d. Encouraging him to lead changes that leave long-term impacts after his retirement.

ANS: C, D Thomas is a mid career professional. As such, you would expect him to be interested in honing areas of expertise (such as leadership, or developing a deeper expertise in a particular area of ICU nursing) as opposed to gaining skills necessary for his work environment such as certification or becoming comfortable with his role and relationships in ICU, which would be critical to an early career nurse. Legacy building is characteristic of mid career professionals.

You are a member of a team assigned to care for 15 general medical/surgical clients. You have all worked well together in the past in this same type of care. If you are assigned to coordinate this team's work, your best strategy, based on the Hersey and Blanchard model, would be to: a. Have a list of tasks to be accomplished and tell each member of the team what he or she must do. b. Encourage people to discuss their frustrations in providing this care. c. Ignore them—they've done it before. d. Provide minimal direction and let them come to you with questions.

ANS: D According to the Hersey and Blanchard model, when ability (skills, job knowledge) and willingness are strong, the role of the delegator is less ("delegating behavior").

13. You are offered an opportunity to take a temporary leave from your position as a nurse manager to lead a technology implementation project. Which of the following reasons for accepting the opportunity is most consistent with developing a solid career path? a. You are pressured to do so by your supervisor. b. The organization has no other suitable candidate for the position. c. You have limited knowledge of information technology and no real interest, but this will increase your knowledge. d. Accepting a position outside of your established skill set may establish you organizationally as an innovative, adaptable leader

ANS: D Although giving into organizational or supervisory pressure may bring an enhanced learning and organizational profile, what is to be gained needs to be assessed against your career goals, interest, and aspirations. Increasing and expanding your skill set within defined career interests is a valid reason to consider a chance opportunity.

22. In preparing for a fair interview process as a hiring manager, you should: a. Put water out for the candidates. b. Ensure that you know the names of all candidates. c. Dress comfortably and professionally. d. Prepare a schedule of questions to be asked of all candidates..

ANS: D Although providing water, knowing names, and dressing appropriately sets a professional and respectful tone for the interview, developing a schedule of questions to be asked of all candidates is important for gathering comparative data and ensuring equitable treatment

6. The nurse manager is implementing a shared governance model to help with communication and decision making. Although staff members like the concept, change is difficult. Staff nurses feel: a. More empowered. b. More communicative. c. Less stressed. d. More powerless and devalued.

ANS: D Change can lead to feelings of being overwhelmed and powerless, especially if complexity compression or rapid, intense changes have been involved.

18. A stroke unit experiences numerous changes related to implementation of new technology, a changed nursing care delivery model, and staff turnover within a period of 6 months. Staff members begin to show signs of reluctance to implement any more changes. The phenomenon experienced by the staff is termed: a. Eustress. b. Care process. c. Stereotypical thinking. d. Complexity compression.

ANS: D Complexity compression refers to many changes occurring simultaneously before time is sufficient to assimilate the change.

In delegating to a UNP in a home health setting, which of the following represents the most appropriate delegation communication? a. "You will be taking care of Mrs. S., who needs assistance with her bath." b. "You will need to help Mrs. S. get into and out of her shower. Ensure that you check the condition of her feet, and let me know if you have any concerns when you check in." c. "I am not sure that you know how to do this, but I am giving you Mrs. S. She is quite obese and needs skin care." d. "Mrs. S. needs help to get into and out of her bathtub. Her bath will need to be completed by 10:00. When you are helping her to dry, please check between her toes and toenails, and phone me by 10:30 if you notice nail discoloration or redness."

ANS: D Delegation communication includes what is being delegated (and what is not), outcomes, specific deadlines (if applicable), specific reporting guidelines (what, when), and who may be consulted. Communication also includes conveying recognition of the authority to do what is expected.

15. A staff nurse approaches the unit manager and indicates to her that because of her father's death in the previous month, she is now finding it very difficult to do her work effectively. This would be considered a(n) ________ stress. a. Internal source b. Familial c. Burnout d. External

ANS: D External stress is outside and removed from the work setting, but it is considered work-related stress because of the impact it has on the worker.

The night nurse understands that certain factors need to be considered before delegating tasks to others. These factors include the: a. Complexity of the task and the age of the delegatee. b. Potential for benefit and the complexity of the task. c. Potential for benefit and the number of staff. d. Complexity of the task and the potential for harm.

ANS: D In delegating tasks to others, the nurse considers factors such as stability of the patient, safety of the situation and of the patient, time and intensity involved, and level of critical thinking required to achieve desired outcomes.

22. You have just hired a recent graduate. The graduate is thrilled with the opportunity that she has been given, as well as with the idea that "working means no more essays, tests, or assignments!" She indicates that she has no intention of touching a book, journal, or health information Website for a "long time." Which of the following would be your best response? a. The expectations and design of educational programs means that new graduates are better prepared for the workforce. b. The new graduate will still have to learn, but it will be job-specific learning. c. An ideal learning plan for nurses emphasizes followership. d. The amount and intensity of knowledge demands lifelong learning that includes assessment of relevancy of knowledge for practice.

ANS: D Knowledge will change dramatically, requiring that we all be dedicated learners. With or without state law, continuing education will be mandatory and essential. Knowledge will evolve from the intensity of the current information evolution so that we will access content with meaning and applicability for our work

19. Social stressors are considered a major factor in the stress nurses experience in the healthcare system. Which of the following is not considered to be a social stressor? a. High amounts of stress in the nurse home environment b. Changes in the current healthcare system such as nursing strategies c. Disruptive behavior coming from physicians and other healthcare workers d. Personal stress triggers such as self-criticism and overanalyzing

ANS: D Personal stress triggers such as self-criticism are considered intrapersonal stressors; environmental factors such as change, work environment, and interactions with others are considered social stressors.

4. A nurse manager has decided that she must institute some personal time management steps to survive work and home life. Her first step should be to: a. Determine what takes up so much of her time and energy. b. Organize her personal and work spaces. c. Purchase a handheld personal digital assistant to help remind her of important meetings. d. Determine her personal and professional goals.

ANS: D Personal time management refers, in part, to "the knowing of self." Self-awareness is a critical leadership skill, and being self-aware and setting goals helps managers determine how their time is best spent.

14. Which of the following strategies is most important in developing a strong vision? a. Seeking out evidence to support trends and out-of-the-box thinking b. Spending time with others with whom we discuss ideas c. Setting up focus groups to provide information on current realities d. Being honest and open about what we think for the future

ANS: D Regardless of how we go about developing our vision (gathering evidence, testing ideas with others), honesty and openness are foundational to a strong vision.

13. Which one of the following statements has been proven to be true? a. Recent research has found that women do not have a unique physiologic response to stress. b. Both men and women interpret the same stressor in the same manner without regard to past experiences. c. Stress influences the immune system in one complex manner. d. Stressors that are identical do not have the same effect on each individual.

ANS: D Research has shown that what is perceived as a stressor is unique for all individuals.

4. Before beginning her own nursing agency, a nurse worked with other temporary nursing agencies in nine states. This career style is known as: a. Linear. b. Spiral. c. Steady state. d. Entrepreneurial/transient. .

ANS: D The entrepreneurial/transient career style, as described by Friss (1989), is appealing to nurses who enjoy variety in experiences

A key advantage that a nurse manager has in terms of delegating is that: a. Clients receive less attention because too many staff make it difficult to coordinate care. b. Nurses report less pressure to perform necessary tasks themselves. c. Administration can predict overtime more accurately. d. Team skills can be used more effectively.

ANS: D The use of multilevel healthcare providers enables healthcare organizations and nursing to provide patient-centered care, with a focus on abilities and skills that can be employed to perform "what is needed now." As tasks become more complicated, delegating skills to others enables the nurse to effectively deliver a complex level of care.

15. To move beyond stereotypical thinking and toward thinking about the future, which of the following would be most consistent with thinking wildly in the Wise Forecast Model©? a. Listing everything that we know about our current situation b. Defining which practices will remain unchanged and which will change c. Asking someone with a great deal of experience to share ideas about best practice d. Challenging current and future practices with questions of "what if?"

ANS: D Thinking wildly includes creating wild questions. Sometimes they are what lead to a wild idea.

10. In helping nurse managers to manage their time, the chief nursing officer suggests that they: a. Maintain a perfectionistic attitude. b. Set up a complaint list. c. Have good negotiation skills. d. Have good information literacy skills.

ANS: D Time can be saved by using information technology effectively, as it assists with effective data retrieval and information gathering and with communication related to a variety of needs in the management setting.

23. Time management is very essential for the nurse manager. Which of the following is not a good time-management technique? a. Decide what not to do. b. Learn to say "No." c. Learn to delegate. d. Break down your workload into large manageable tasks.

ANS: D To manage time successfully, it is important to break down your workload into smaller, manageable tasks. Developing PERT and Gantt charts will aid in dealing with larger, complex projects. Both charts can be used to outline how an individual will approach a large project.

19. A cover letter and a résumé together should be no longer than ________ page(s). a. Two b. One c. Four d. Three

ANS: D Two pages is recommended for a résumé and one for a cover letter.

You are working in a home health service and have three unlicensed nursing personnel (UNPs) assigned to your team. You have worked with two of them for 2 years; the third is new. The two experienced UNPs have patients with complex illnesses for whom they provide basic care. The third member of the team has been assigned to patients with less complex illnesses. Your best approach to supervising their care is to: a. Remain in the office and ask each UNP to check in with you upon arrival at their first patient care site. b. Ask another RN to supervise the two experienced assistants so you can be with the new person full time. c. Meet the new staff member at the first patient care site and ask the others to call if anything is unusual. d. Meet the new staff member at the first patient care site and call the others with questions to determine whether anything is unusual.

ANS: D When ability and willingness are strong, the involvement of the delegator is less (Hersey and Blanchard).

16. Which of the following needs revision on a résumé or CV? a. John Jones 87 Highway Drive City, MI 79110 [email protected] b. M. Howes Anyway Highway City, MO 77700 (H) 777-777-0000 e-mail: [email protected] c. Dr. L. Jones 99 Carway Drive City, NY 84003 (H) 999-999-0000 (Cell) 999-000-9999 d. Tanya Jones 67 Honeywell Drive City, MO 66907 [email protected]

ANS: D When including Web or e-mail addresses, it is important to use addresses that are not overly casual or that communicate personal information.

Leslie, a UNP, transfers a patient while using improper technique. The patient is injured, and as a result, a suit is launched in which both Sarah (the delegator) and Leslie (the delegatee) are named. Sarah is named in the suit because she: a. Retains accountability for the care of the patient. b. Worked the same shift as Leslie. c. Has passive accountability for delegation. d. Retains accountability for the outcomes of care for the patient.

ANS: D Whenever care is provided by someone other than a registered nurse, accountability for care remains with the manager/delegator even though others provide aspects of care.

9. Nursing research has indicated that the foundation for becoming a nursing leader is the ability to: a. Write effectively. b. Speak two or three languages. c. Focus on day-to-day priorities. d. Think futuristically.

ANS: D Whether you are a leader, a follower, or a manager, being able to visualize in your mind what the ideal future is becomes a critical strategy. A vision can range from that of an individual to that of a group or to a whole organization.

Sally is an experienced nurse on the unit and is very experienced with ICP monitoring. She is assigned David, a patient who has been admitted with a severe head injury. In communicating with Sally, you would: a. Provide a detailed explanation of what she needs to do with ICP monitoring. b. Tell her when she needs to provide an update about David's status. c. Ask her to tell you what she knows about ICP monitoring and share expectations about reporting. d. Advise her that you are available if she needs you.

ANS: D You and Sally have a well-established relationship and Sally has the expertise to work effectively with David; therefore, you would need to provide little guidance but would need to communicate that you are available if needed. Hersey refers to this leader behavior as "delegating."


Ensembles d'études connexes

Quantitative Reasoning Module 9, Section 11.4 Homework

View Set

IB Biology: Unit 2.9 Photosynthesis

View Set